From e9246165d38377ef7e51ad1cfc25cea2e8a7ca71 Mon Sep 17 00:00:00 2001 From: wieerwill Date: Mon, 7 Feb 2022 13:09:33 +0100 Subject: [PATCH] weitere 100 Fragen --- ...die Neurowissenschaften - Fragenkatalog.pdf | Bin 173629 -> 180282 bytes ...die Neurowissenschaften - Fragenkatalog.tex | 250 ++++++++++-------- 2 files changed, 139 insertions(+), 111 deletions(-) diff --git a/Einführung in die Neurowissenschaften - Fragenkatalog.pdf b/Einführung in die Neurowissenschaften - Fragenkatalog.pdf index 56c4ebd99b90c25bad790b9870f1fad409058860..0affc99ef73ef5cfdfa0e3e81400923bc3f4390a 100644 GIT binary patch delta 64669 zcmZs>Q*dU{wl*Bww(-U`I=0iXZJTdw+vy}7+qToOt&VNu>%GtUFaD}i>t@xun6+xm zF~>6>bWg%{1i;rt1%PpIB^&(!0Gl(V9X2Fz!mi&i`Bj95rzM;{X-!Govs;8!@c{|k z&=YvF;HoKqh7}&p-=`$9wdGin_2sO%eTVnnZ*)cgnvj6DEE1otw#*M-Y0thQdTC8Z z7h(#42nFnXj3mt5t~stRQf;=*#Hukbxt1qa=nos^FZNtzi9yBJzuliAzyX8S)J?lT zG#MMMr3+Ku0IO~}1$Dh~J(W%gft9pLJUK1L#K5|o3eetaVpq&c zL05qXzICbvjqkP$$(`Tf++fk+N?6h<<;GA#dB{+AXG?94(-7zaoIhqCQBE877ffb1 z&OvS)D#R(Kj(tjSq{B_axn>8OVs}zVUQjr0cG6sqeXVo7BD(M5lkenmH#cuRoR2XW zlZ?RN_x0KXrX0dn1>a5nxr5LLMXC)5KNLi>ryN8pVo=1fESYwC`@xNLv?)yQ&g63Z zQs4F+cI?rLFHSR~ zIaAJrP&Kt4{&D!k&ZYz?i8Lq}^Mo2*r0#f;T20L%Wy!)z3!sW75fJr z7HiZwwp;iEZRc&=XK~Fr3R3a(j3maMeK$&bbG-e8vnL#cD{3bpV7E$7YG`Z+ zp1fs;mV1zPb-@9IV!2}&N}ri|?cHBH%0d{_#N4PrM)yah38ZAm*soZKc7=X1#o#`b zEBjJx%7jslfC~#P44Vbbz6KUn=bJr(T-UR?<#PEvGH9@a&|^-n5l)dJI7h4xG!0Ap z?AWAPzSK{_mv3#?hUJ-VWqthW{ZiisI#kMUk6o_mP=dYPV6t15SR$Q z%%7=)_cV0Y+HiEWEZAA4uFb>IG7|`%*|q>>j#L9#}#+oD}Hoit#IdDz^@!8dG z|2>CFMH*p3kZCe`r3nKpVOd!(q>xS#Z2Q5B&vb2W2y`)$aDh~AVu`JtFU<)`i@&Gz zmG2d_6-^fw4Q2PVKa1&-Yvq4z(`7ALrPFPsRv1L-_fE?NHZ3*3H&p9)e69Gzs2p#u zZ;#7lyr_s2Y}%oNuom(pH$($P(!#W0EYm}0h0pubc9GBEn^HLWMX2K6GdD-X?+u^G z$qpWJyV3Ls)E-U``Siq{wR#?j%MfIIdb`U)MAYO3aK8Kl?XTUW41G0Vk_h;M;vJM` z7%XRapF379Mc3WpSA)4*6|=LWvINIC>|Ew3iw?}sYj^hKd)T8codHiMVkKjP8y+<> zg=Q&Y8Wxf7Kqr@{3$(@vQhf|9*T#q_2+FnyfCWHW%9Sl%a7eXYyW-K-InIloir8U0 zue*ci{hS^WRXRmN3mBNhAFnBcGTv~a>i4u~f;P3cwE++gX*}tFs460$j+jHptF-if z^4EZxZj4}!Li_q1Lov{Ct$}t|D3lT+<@nAmGVmqn9?urfO+ZaRMnG-uNadIVDN55? z8kvlcDf&=UheYOD6=t*h4;aR+{0~9%xsVJenzTTtNq~8q4 z{B+AssLG7i=Q4Mqt_ImKV+{3c(`e`k)lnI653W>0lMq;;TNe%dU1o@?%S*=HY|~2= zMu(HX_tJQ=oP3Y5tn*ruue}uAE6Gye6(*#MLRm)v=P2kmiaUYM8zmh8)0;R*rf^Il zk8%hOcrE(%0TeVc1&`=n!9KqduIO}YZ2D<>TXFGx^>wK2H*Z`0(5D|+GKb7O^oX8I zSRNuO>2bdgk|I$y(;@FFIe#ZJMH}#EI^h);B+(!Yw=J7=K}U1ooy8VTp9G2)V?bUQ zJq4S;uLDBr=AD~<&(u1KU|N4CsJP@MZ?WmQ`@$C50T^%pAXo9RNW2@;4{S8m94V}- z5Rd}NtHceDR?_^LaZW4b2Fn)$C7uz# zfJriNeNY#Xd^_P4#mc!W9(t|*2vvkPf~(yMJmm&xxs6@34CG41Avl3y zu&u4Q39@aw{>7Roa-knGU6=yhQiNLdaQ}Q9FvMBkJs0v)TU7lC?P#;bVT#iaYYMNB z@j7Sm?K3>HAhvq2;cozaytQ?+xF3fbh<`7Ufxmn+@7T47&F$ z_brHURuaiRn?fII0!TgqN&n?v7+C82yg7th--v@3k3&7qXoh*V%fv>_MFqsgzSj{H z;I&KMrK7^8jKJ|l@4;rD4UKLdlKrV}_x*kBW)yvHK`Z{{1$>6lAIM8ZwkU5XFP2iH zJqsu*l%z0}#a_Ljolzs$(xSYh2qR)*ArRIm?{~-~@fkiX=_uVZrtZ&9C%n`7;Ntk8W!z zC*5BVu_brrRH*R~T{l8@e}}3nV5|C}pCi{*reva1zs0{yBRPMM-f$QQ=ZNJ2>~>s# zT+8~X4qnY(L-yc(uDj~k4wrZAe;pBVOeT%7Lf^yrf~G;I0qFoydpo-IhYugguadY4 z$6@NiD-IV3gVg9JtYY6`bg7iJSXgJPXe zj*St()NYTDBZLXWVPhD;3HaFx1Vo|0_54pAsgnb6SL4VYDR$?aNlSFo;nx(}mAHBg z(p}W>DlITDRFCNCx;DqFU4xmxcmq|YE|Z&mZ*EGonkpEpN#_s`JMKlcVyMJRb;QZH zxKU2L_YHxezfWNeP zZ6KL3Pz7^M zEBu+-y4kI=vmOO=m{G5(TBvS`sq}FtbZv;UdpXOE)oRx(zMX#)`q3Rtr|aWUpW<%P zdAe=XpI7}96anUTD1FvwoOj$S+aR^u{PnF>mL!KC*dWB#lV(X4wQsYZSd&TKr1$&# zmF0^?W39UUm#^R7o4ZS3#fDCpV7M!BZkAQT~baf%8D2NT&=czaF zlfyJR9Ic~3+o$}m+#mRO0WnNdue@ARZX7z3i#jVG#3rB@X{&UeuP~db$I(~!gTDR+mJm=wa$h-=1jT>Zz+`D zn#cNq)*cWF@9anPoA-F?EV25fYeP@1*kuDrwY((WD}3X?T>M|230U}#z5ui!QJFzd zw6GcYaEcs#fj+?S;{EMVYo+?mSlpU(&7|Kg5p5F{S(qFKy@}4C{8eUu|Feh&GYwJW zA8XM_AD!vE>l&zI93$>r7d;Z@6-hGka4PS7$RLyZ^j7 z7+b@!F?0SeQnMuY577X%#@+uR_0~IG+wbXmlq4LfmS_U%*h`8jX}royvWXy8BUnsm zXo|nf20P)Ac((FKBUCK9Aimhoh;Uu)*q#~|fuG;z&QE8oGF=eGF{lpC$MqnPsQ3Hq z!omUF(pP@>l=)B@cV={Lwy%~k(g)uHGG#N-K^nst=qgjM?We%8MhO8!ASa$=% z$K|>t*R6+Br8ZU(;Bn)K2&!06=ub@(h=-Q4W`u04Uu76$n^i@Kml{rWCP`L@65so9 z%u8dAORKxRg~$e@i+ww5Vw# zXrX*oKR^a@515BeyO2f104`mM`a|!!&PG{`=CONT#Z?YI?YXaBq+X5j(=3IF-9gut zaGtX{_+E6N)k@bAC>*IjJNK=jTdDLP`^osq`+!Lp{a~UQeK=6xEJ9!UNV`Z*D7&U; z%wQT6BXJO?_GmW)6B+dUR~+{_sDV;|ckC{M>AX9R=2kxajIT_SOVNY5cJa z{w*8&F^j=;7<**p=LFwWe`Airq*^s@6V_S5m|1CddRG1fvh-in!+FpPGW^)gIFhi0 z*d&Z$D}Et;D3Uf80txP1J8mHsH&b|IK3w2jlaAmK@A=gR-yimk>zwnVH$P~I3x10meFhWDT_PXv-AIkqln;g*!lbK`==2hLjN!|o{c z4b5H-X3NmC@wu^i4lPw#mX#wq>Pw<_RFwJ-7uQRa2hwzKi7-9l2eD2;5d!npEJ6QKYixZ2dF}H^6Y#UoLTbAy9}qJd0-6u4NR-2ZryF& zZPSS8dL)@vFPf(rjXcDuWR--njgJ@yI`(-L(*MXDLt2*!XkF8kfFjngafkZ46Y>n>I~WY}03<_DyGt zhur5Vt3kn9IY~&64RVqZ9GW3UnC zM#QZJ&&-n&B?SrDU>O=Eiwy8IVb%pCNMIBdhT4VF_{J+(U*x1wH}ID+vXh98zy#<&%LC(J;R)PG- z1NuLWCdQo$Wz?2;-C$wmcw1$f(qe2Ab&22NQF!CNHHg9ANL19t1x!-0l^XZX)F{p8 zn3ef))10H7HAeRU>=P}`uaKlBSs*DqqG{~xJhDcGUzTaMN?kbm1Zg%M?OozSw6fMS^ z)l%`5QnZ)#e0G{Ufq#ET>=|*iVeC;5s08`21XR)zLU4EEfZuCuU%!C$%BsGVy=a$y z?V6Zpx@-B(dGHiY&d~BzO}=SrY6Ou&wzQFP_1HFUgB8d^klJ3vT1AMAW-)by@wNKE zCdbkketv_h;n>@;{~sXD%JyH7=Hg^c-b>L2gB*ZuUOe8#vhUKztpZYi)d36^8wyx9m=EL{!Be8JIv zgmr(|{Rk{CM;jk?P->?X3xpaqNB@WS7N|Y?u{-_=tEUtR0u-!U?su7>Qmd3XWi;+& za0;Sn(+xO&Kp$M)lObf%U_}6?%4`S_$_7&8?^sO{kl}^>(23xSW!y@$rS2gmtl@HV zSPsub39BdPfd1k6xl?4U+^TFrE^vw zLKhz?4GTQHZ^_hptnG56MU-}zZ=p{2Oq-r=p}Q8FuYQa>em*9CFbD&c+&3O5FTa!f zli#3!#-w+^5nwzkR~An88egF4aJp${W&0xssUbh^uEc?FO+Q;wcIdC1?np{>@2~=3 z9xRQQvgnsoBgKR#?}$cB;tksx6AMms3bnGxN>9+mNMRu4@sNYErknaqRVELjPUJ{~ zM;vzB3m~O~1u-os)koEY8rToaxzd?CFL??&Uq-YTA?d(hbOnIVYcmn zkRO?NK1FuWNH3Kck20y|F~qruO*TJ3Q4GO5^vcA_&1~%GUAvFbPi7Sp^r#nAwD(?q zE6eCVS2*ZVYt0npf*V&EflGdPcDge)6F)CjHTM+*^rF|GJAbC;?g{{T^8O zz$OPWvNpN`Yg8jUWwYu-S+fNjzarnJkg%!r&5TxRnuG*gt!AI>%k{XhvPw4!66`5j z!zFme266aXlYPl;S}h3tHu8Jn$rLQbqMX(cgCPi2rda}QfiV!%2OyFCDD?>JY}Xe6 zd=O$Mrv70Yu2 z&M|@N70_XUx+TR^!o><3MC>a>4N86z-o>P2iFl!Q`&1CuFA>2F+>tTj(%a8jXv3YX+HBys)1hbfkeF z;HZ4prfLimbkP~Gb=F*3EP-kSkXZkeZUl`P>^|uWAU2L1<{?SuVP3_9a>aI3$J6xb zCbqc{sQ0glkiyVsjMWc+J$a+r_Qkg=;_qG0SR@EM8-Q$~iQRCv?nnUj)= zkSx)w-eeHCaz~cw=2?nZ&FK-b1{O4bvUf@-6QN&*W^25J6JNv#En9v47`}Ch;&gLc z?Ml$>z#R7Hwfb@tcVnSD2((_OyZ}dg_|@aiAXGSiqIb+NRS;!LQX9BCYPkCezKbcq zU9-4eK9WkjGiB&DoC4n6HaOwu;M>gQ$AWdS47T?KwX-JJACv_r9$OWqF``jT-w?3r zzaXLgSc;ZeP;*Q5#yT+fK zsHb#5^YHdcfk642Ul7qmK=v1?eg216Nm+}#!-rh`lyEdxefLiU$x$=Gc&7VOf1@p zHb-1=Y?=^KRi;J6=k6N}U5ms>A3Jdt^I;wEnbc?}1I=Ph>P;IbxExHbRaK_${=BDo z7IVU4Mb9OmR#;zkGb^v7gc+;jh*<{{y?z-7H!FkK-rNnvW&I5=Z_W=-t3*9lizPKT|;{l{#ESh#q=NF@JGh&ztax00SNPh?oF6#IJ`vd2LhS1XU z2-kbHh%aZJfgxB(J?itkF|4F@aruL6l2P+uF{MF3Q=X6Nm^ecQGy-8&M?B($C}EUE zqkrF;;$T(=oTHJqBMK2m^y)fPxQ$Q==HroA#-)WGJi9ft6r^u)y4V(oQmS zLPH5${5lYM>E=KXVu-<Gay#Bfk zq4F4cOLml1u4hpqc!%^epP)juEl<0k9@Gj92+8r1d02{tSIc2g!2MAQSCDfNZs8T~ zB>}TR`+*CU0b*E*qujN`;;1z}owjv;mxl^;qI39jHJa}aWuYd5w%=^BzT?}|^Hta7 z{ivuZH?htiNm)7XqPcMP7em{9ho}a*6`Po;_DTF|$J51&|H{DuZfD2e!{g&?-_vUo zTbr1^^L}uuJ|{PIizh>DPkogKru1FI(EIu5$T8^JoKA%8 zILyR!LwSJ}%@qJ`C^}UkS4Q2kf}~^Mg~-%W@~v^TxHVg7D2{RzMT>NEJk`w4WGj7n>eU~Z^E!{_JoHYng5<$&gAW8ewx%}pJXJAxeAZBv ze#W#eNYUl=($hoj(|N(1^fz`Tb(U+^O;)({u}I!7EGUYz=hEJEKPuLv)l`px)YfO0 znbv118Vj&bOdPSitynh<^iz1Bs!-9>jqXVVElze>gIOYEWkn;W7O}syJXOE&n{}-h z>-~;sg-^nhLyWl2;yH)XDTuAUGzFbL{L*~)aq?4VXr6=xd**U%jcJbBOY7o_C%L?v z7^;ktViSjGDRM;4gjP6Ilrl*tG6y9@+v_82jc_Fu8*}tV)>9}!EgC0)<04AgY69WI zYnE}}qia%{d6to*}%BUf{622df^!{C+t> zS+lhu$Ot=m3-P4~J?Y1)T4gp*;7?s+sZ*D2shnV89b;e7lzB>i=T_FR>vC}xGT0JK z9i;<*eSnteYv%u#51{=2RXcW${}KWY_J4#R=AXazzl^{ejM*0w($pxMDnL}^co66>#Jvi|-Iita{_M>2{)#Cd zjA15?2Ki@$q#tI1Vpl7Q*}UHw-4_PPOvS_^z* zOwV1#azT@EIbZSB8SB|@Ip>)g*vcu{vru-3a{h99b$|M|xP!zlhoevsK$tr&IE8tr z5TL2ih)nEM7ueDYDnTzBJkw!;IfPM^BI&3HwbJ+_-ei^=&T|5FT4AIQRL!q98ig8m zf08;VcN76y&yMgQNK5L6i*d>2YMUw?E}-*DIB>7=weqCv3yC2N4iTGPXM@q3qv0hR0 zM9R=TCKANM*c1Ilr9w{>0$!wWlN3@1dYWMVz0bL;1K8csxww>D(uloE+0{hlCNzr| zfQX+G_1~of*YIhZU*0=zGp6r|~JxwuEt{o2nM@UJH}e787M9vIjYjTmu6 z`H%c{`=X+f>fNSMr^5PgVD`*eiHuosow9y$?<2)}YSJM|WndUc0m;7;Oshd-hb6~F zRXgvQ&4|@QA&=1LuAcpf$`bHt@ zM_6$RaPCQAd_~$aW5P3#KpiDl3k)d+QP}Sx%LYp{=v0F|Yg08ZHs4mVw%v!P&7KvK zl`sG7M-LxWSWab308`RCTgsleGQ_pNqTl#M??(7#bQV8q&|}@uQ%_hkqDZn~2!~{b;#p>>H;+Zq!VF>8H%>ggTzPb;s=x`C$$k(L8cvNHE zRt*SDzQ?|Jjui zaFA5$qeak!JUvR+FFdKc7A)GST;{O3rXCgYw{?cya0ypPk)jwg@DdU2VC75{BvRRB zoBEye>DaIC=}JGKslp-L6ZJ9epJD)4xZO0hns!c0SBlJ!-x5|RWNE^=(wW#|^pSgR z8PH-xsK8DB5pub(h``PWsUwn!lFnJWwKBC@ev8bFDn6&Wz6=#(UqBnYkw-VOngvKE zTeE3-D@7_Evh7%x?+aD1rVzyH*PBrwxtQls@bEo(n^%lvMhv7pxmfb5G3wva=s(Gt z89wbvFvr|7jyba}Xf1Kv{58$4QX2MGyM^b-MZgwEx1C&3Sl;-gXc_6zl}jon7joio zHZ-&3hB_OJVK1#!@)iTU^2^C|;V{c2(g_o$utlQ)UR+N>Q6kj;{=7v5t60Rw?#KZT zHBs%o{|;v1_R(2WRq`hxi1FJdZ7(SyQ5tSEbUp={WGQh4V~0Quc_1E|f?l*{Foa4Sog!%Y`IlRq^d$RC0L1 z43Z;v=lh-op^_hr%dy_=f@DE@sAm+6*XlxFRv6+`>r~9w^uDDec%}NadRDs~I9ms{ zxRa6W~j@-tVfNy@`=A0RL$GzV_N12P*nlBo67_GD%wqxf;R61DC`kWNj{XHQIN5prYx894xny%=gk9g$t!5MQCzpv3len9gE+}mL zzO?1)9KX371_ zCHQQ%33kMyAQR^HTZTf4AbJmmBpL-22x49EBh}rw5_O(W24gJas$(x_!f2vv9&-!p zQkdt$9+w%2d+6M}hL{k1=}2k1$^Zw?C9q`F-=N_)z_1MZ_Mg+8*}_8Uw3^%z5NFQJ z)ij>2s%#9b?-emhQKQ5iMbJ_@u)Xx$&yez#(zHC-Dr^RDWwZ`T>4qcbXw-1_xp6U< z2Sc*Qn<8H)ByfsNl{mL+FGObRIPZKHtS6{E-i{O=S;VzG!SNW(=$4ASC4gE2?G&s` z&a@8ECEiX?b^1AdCDluI>gIIaz)ts{C)}~!T#f{#FpaJv#Sn?EuF&rpj%=BnEIKC{ z#Sx8Tu(=nQZ^>1()O(>D6~O~MCzhnM*}QW^Y9SSx_O4u|hySKxjmNQKwtaOogzH77 zk3hrraPiw=W-Kxr+44gEKLzDLyMIH>h%vVEd#UZ~zC>TFAW6KKNrqdH6I;JJs=~fD zAfYC)-8SkSQgJ|T+{EQOG)Z1Bpjo*Gz>Svob{h&Q<9LQj!cX7kC8v`ikBv=3mciXI z)P_Hk=DrQ4G;o1Y2on=*ki>K&&th-z8JvD{z-0nAxkD*MH61c>{9 zI3y?nkO?OCkN!^qlY&I!SUi$S{w7V#+dqxKc(3Kt%batV+I&Pfohf{|k; z-hQKEEZ7&1%lI5!chFZeb#My)b}Y%Z`v%PncH_$`qswA0?y&YE`FS6N7{8m1!sL!) zn*{;#;$^XG)DQf}@8KKAjbJ4x+z*5*1m)urZ{>@`ZK{_yA~5Lg{cOM>ZwwdWqoH90 zp9%?Z=FRD8r&N~Z^`*_KpP0M(>8BB-s@~W43J8&r5R&#am3|QIuEluY-rQqt@|Q47 z9)q59#1bOU?Uh6C$Z%b>gt>~^JG^bO-s@h?mZouX`vH?yx=U`_L^wrnxR3!8n`@2O zPu`H(%3agEs+uk4G4zv)T30x84(p9DJonM9H?!(XpD+FoTL2aI!#ZDosW}`t)3ld^ zMnS4gxf1!wuqam0*D8~!4_{Yn1Z#8e{;RR8c|l2iHtz>y-7mi3|>j6Q}E8cu#!_wS}+XT-@SHUvP6 z!|h1VqzmW|jk}C$Z7QDHElg8W&ytlDo#mv#%6ncylE#G2yA_SdmNu0Z+=~}*o$SDn>OP33B|M} z*mPDtE^yEa#~PqKFO5c;2gfuIyCAq6?ura=Pqct_l&rvwCfZEg7_3^CJ2GWo3*As# zrBkV|+=x4|wXMHn(fmV;d(-U+Ba$V3c?U}RLj}pItIn#6e-Zfg`e_tVqxAD+qLBB? z0!V5M;6HaG*D2#0jf^6j9gi=Qat!JAEe(g{iwCElq@s=D`S}Z6`CX^a!5wwga~jdG zCUR)t$M5z8HStd8yKARHDCsS+MFugz$>bQNsl%TFp%Yy$qe`lfPHjYxMO5b5H3iG5R0fX`(pLI&tA0+X%>dC|8>Ki`+IM!n+aXul@n9|u~|Ld1i{pn6;MwY zquQU3Ee{DPp35y1#z*3cUFk2-y*NU(cOJ~yUZk5A9vQwSmB-4+zh35{QVSv}_%Btn z0OJkBt6paFrFy;&g5Q<;V|NXdNb(*;;R2E7MZ1~PC9mlbQ~gvl3KH9>wBrEHKI2pF zxujVQKh@kcgHZo>xaS=D|HWpw{x>bi^Zy0R*Zym8Tz_I*5OSc`LkKxql`cqYkFI4g zx@TvObpI07+>eNcOL96Wy#_@RQm(L)HuoWS8ti|%{Q>W1vG+1PoayjopquOWnKYts zL1~1;tZjBGnnHc1MkcE-(S+NO&zae|N78J=JI`-o;sfSIAQcT z?dj%ycGv{p^+g(A&5?LbhF#CD-lKmpq4OhjN651pnLCF>er8@U@_3+WI%`^LI-f*A z)*UeO5@k}B8eN>aHBIHjyIj2kuflcvtl5s>ymYXm)pH^2sVMaQc%n1u#J3}!dMdFz zZ|`G9?Dh7>i(lr8ha1w&^djN1kC*D))?r`Mqn zxuQgbB&0WO#IgPC!y&ojM(ETF7|uJg`?I?kugLokCe*Fj#BCnHv$j$jnDn))PF=ji zKPuomx$D-6I~Sw@&K0QaVyH{-PO991MPYd8w=`5Wa&a%K-)cW4HmdN_%TfXj7BE366`67d%6eOak7fmSYr4q?8;95`GZ7->=-Zs^v zO4c6R$9_!?5n*`(Th0zXt0%KL4I<{@vV+fzPv>JRlM_*OBj7$Qf{evwe zG91F#3;V_*wFz2z^I{lX;;!mqZ(^JGuc+J_xuDnHF=HC~2lKExy^ z6vx_bwPoOdte4WEgtFH%9K|F=D}v86E58KJizn&0$;+4li8p=k+9kLMbPUef5JKo} zwC*~wiP#|+K~YwFK7HJiam0O4{5PJ3y#KSorOA?HGt_~UOrE@6*bO?B^mNX|581U) zL<_1LF}aaofC~IY zV8fr5Fdu|e9P(NT3>--o&EP94Cf0=*q>|sp9DWV-V$DAUXhTeQzU*$jQm_a=V7mr&l!0?O)y1iX;7V$F;RiwJrTctA> znQ1qDA6o)z7o*N%6bh47$O^g4I97QgfhIn2ibYbojm#;U%4p6EHVmO!t=2AlbN-_S z>BZKoYEw`uJq=!A>5c68zw529EWh<*3shj(WtEK!;6_-f_~>D}9FtCFLp*bc9c6R8 z{Fy6w|22l!zs4|KQxe*F${jnnXLBN^duW!z06~iq3ikhx}CXIYfWA7S$wQ}ZS(KX0~GnCpi1Y40~9$a z4f75E4VPb`3v*4O{(!cUGsJ$pGP$+imjVFb5&%r_5nz;zLb7|4>?ji?*dLTRii)LR9^i;ssrpBV ze_F_I2XSIfRwu{bFnmt+*RQdSng%4ZgsNOwLjH_5kD?%zn`99|3Go9|9qCmll1L<> zrdW}8OHwtK3hCHhZT^i{e(?Hdkue6v7L*4Cko5_@t%`~_R)gK+EZ+~H3mj}4_RvCw zUuz7ehu}_dmGYW` zHZI!Kluf?M5}SK3dz|9(`NkT5)RG;L6Im)E7cP>9$kcWes~z~&U^S=g2UajqKPmY3 z%JX2%TAhPY(R`$Owx#I6X?VT`jmS@$|GYm`a$o-)L3{eZdwvm)^piZ>@iAYns?!^Z z<A9Ho^Xd|6+4}p1r`x`!VK$v6Gei+gCF~%Ci>^d5tax)dKVF}op1kT{XcwTr3f7k( zr(N?h-c^&p5r~v?)}SA(ZL8~MaGRgGGK563i-&&uK7TTC`5Fe!y@ctJ1T{$ zVbnp7h}1ztnd$1_egSi%{hHwb|Frse^NdF`5xtcj!tm9j({9ZK$nem<)oCwPEFM`e z#=J;B1`V5~-&G1xy3NKu{Z}LCo+s^VKiE8s^a_I~8lXd26TyP8tJ~uZH2bfNk{R zAu;+2kHGbnfk@7hkFT<~c6H+DSLjq7@}y18VxKSN5jj+lYCYK}9!nv;jAzS?;Eo|! zCCAN9F@q2!Uf>A`>PXT8nKxe?5ET}jPBvs#2qjm;XC*5@kfo7wTZ-0%zjQSh{VQ7r zi4pr5M1a0vM03w5{@@greA5ZD*@9>A9?uB=2Qe3g?Da77@ThVX^yM~7G8d!2*;CjL zsgfB&Dpf9u>P?Z+L|+VQ2D8UBTLLIHXd-vpqS}unNw^0=K{3zR_IxOJJC2)izEdj1 znx&ElL6k*u>vo2&!48B`hJrylbc54U9w49^(ep4A{t(A8EqqG&`yFF+vin*pPJ;pn zwFob|8E_OMF}b;^b0v@@`z;PWFb4v=9>+%U00j$8!UlsN)vt&br_i|QV$+#IiSQ0& zMgdYHB*r3;o*ILWL?nZ~xf}#au~h;PfMVL)WcQh+rz+xb+!Ztd2w^Uwd9X?efq&j) z=;I6<+*LBDr1AJ2Gx5$9tqVC9Z2AjM3bpu?i&xB4PqXw*?yh#)m>EqD??{wk(7;5x zgry1sxKRZ|A*vGa34*L6U`)oXP`lf~&Ve~F&7<8Xz`-SJa&5at%6AT9h+WFjoXFMp z-;&5vR=C{1>wcND)Th6gYeevM2J{*j`A* znhRL%K33VFm>z`cridC>@%`SZwZs|Qu)}$-8rd>x?XC6*?>0#`^naV$Od9Orz=8J3 z&T9wtYnMa_xgAuX7G8Z3hmln_W+I8P)wh!ex>%h*3vBfEbK*p;)8%Vy%F1F>_C=oy zr%C^qNYZKP@A3^jL)RZAj@<^4@_c~6j>ILSy%mKCQE=d!yB=f(sZm8VLl%n-q7J@I zn&ozXoN#MOG4>008%Nxm==$B%8dU_r1q}u1sMmxi*y@0 z({UB*H4-nvZ)JOCdfc1g;+e?8AA#VB9(6Z9^&qN3{FJ*v6|knD)r}XutUPV|UOtX( zEJPxU_dd&2rb*vr1LaOJxnzt9FR~ikg=4}A4{~Bh4y>q$H1$ZUFdkuj17F}Oomi{M zq&4W+EUYBVB>#!E^Yg;zmX>!&yA(R07PEO}omOFF;J3c-~ z_t%x#jrrRlndy=I(!V`ijwyR?28U$t7My_a`)Ld$0cZRhC+~&Z`{8+p*Y$3H{Y9!H5u=ZDYx@b%EjzE*I~=Ix%~W8wFjk~BA~)oxhR28ygq&|Psdsybf-Rn)aZMo_34m_0kGZ% z=Z!PKm+gP~yeD7O!^~$x1{)?MzrCkKOdKkk$L_QJ8 zV!!8bkon^WiHHwl?|9n)S&5Gd3k+l!au$mbMgh~+zwVHiTAGF}Nb2@Cu7I1#{o-|gPjjh)I-~yx1 zpJ6cjCYwSbMJh`L2*~!K|J8Gzs1qZcMWeQ)!%1HdpgI0IE@@^XOY6rwvoAf@GBFw5 z6N>7_H&ui6GU7egbhZu1!CtcYDwYQAWju!IxvpM6HMhdO_o>8QDqLhp1SkP7h^{`$ zqI~?Z1G-MGVRmYboGD*yHT-PiCD~McobpT)BkrAq6ium2GvbWqSr|9vkUF^u&u+4d z+k?G7_!n7jw8{ntZ9SP~78b(2=6E}C3#Ubh;A% z4N`)b6jD};8Gbdsn#2VcaHTVY+c?L4bLX|u$+J}Mo<$?h zFGC|tOKJQ0MN+?BN`ewJ#JD99%@B^vS6n#=$1Pxmmh}NvjC>$8S^y_7&7tpowz>r( z)Ob;5VvI1aY*$sV+$@GBe`dchJ@_)cb<&O&b^H_P>=iGOUY9VJpeSUMr0C7w6Pqy{4K1Dcwc~r*8kVNw}J+w z^IQrD5qp4$_Vow=M2+DRyn#Us7u8OE!k*nCTdYUt*~os9za!P@q-6>Piw2S_sy?xC zEsOOU|89%lpf{oo0UyFb%Y%`Mu1z$sojWBM0q}|#8D!6TwzJU$nPNI|?g$yobGy1< zggkg!Pk>MrAD*c-&lE+T8L*Nw^U}qt@+&C$?2J^j!pH_dp0s!YTq0B2K42^Pa6w~qsA6GqCst!KR;mGbP=5VulHP4vtu`3nZUZXzDXET&(m673gvv-q-wvbX(-P+F!P zH85C(om%hog|A*|vtLJ%B75U{x(Z%X>@Rp3PW13d$XY`Or12(ey$&?{oO#5+=1n3q zD3t9&`6ny!yw7+n-ca@kQ)@8@tvp94Gtx?(WDNl#4XyQ`#`*ps^3!wYWpJQfe}i1KSZfwnhphPeeEeqA$k7ehlPC@WZ@hVoy?3v zp#(0n%HVSJ!OMKVp{Bt}jJ6?}h@<7|cQ?lSRq$2^bl65P^vIi*Ctkd{p^l*SULi9d z8FdK2_iWSc{nbnh2Xdlp>~7~^yAMC_sh1N5F4s&*wCVx=+c=7I^bc-IlILU~Gml&Z zH*iF)Gq(noLkvD0oi{pJ0$zM!)GQ_+n(c$Z6GVpxxVhM%R;A6mU-ij`BmRjs?VXIR z&d6U~VUOiVnZO4wrtZ+S!<=Wdump1>c0CjTkE!=?7KpyCItXRkvYDzH9YV|gUzfWd zG^o`nx-RizT)ks57tSjQQaGNfZd(l1fnhVy>Y545UJZYNBM;VN z5T)t*Vvkkv;B@=8v8wIPZAvO5naFFvQad^nQXFI0MV?qqxvmu@&UAF*AcFg=7a8Hv?osT)EZRfpddyccc5^3cwWoZ$K<4O@9I zM{iwb>G(Ss8aFmMI{|GkZ1phnTeQzR2gh`|QD7@vPs!x&MxuXX5wXD~}l-2Flz zUzkq|!U;Q6K@A*r3GfTRWoDS(yM0K$DSinSI^nJLDH=GW74qM3o%{uC1-e&r6%OBI za28!O?nbpg^uR;)ZV~*#3MT=h$Q?_J*oNeJr%XtK#Gk5V2#D(k)(pqY`j24=!ovAK zz7-40|Blb)U5Yud#&6!x9>XlG(R+ujELi1V%(8M_IqVR-eTbrC*ax}L8+Pu>>qE$F zDchJA$cco0h<@@k6UF`|Gq|Zk@%X%fxiCQ)j9^Mo!YM@pm6Lk(b9*}0ML~bx8K(l~ zk}a11Iie||0;Z;X?}-vH{Bo-Zq>A*E5C*y9h6jAP!Z3v~l2Mri91=mdC&7~C)sOtp ztiEXWe+62m1jZ0!Y%`LS{Gb~lpQ4#^#a>6B$;P09nD1T-BK#?0bjc#Sqf}lk%9~*x zyR)GF4IK&V6Lc89j#t{45DF+!)7NJ+e2U+(r+xDa^Xz$ixlT8(Eg`hyH5(PE-OR*T z&j1J(F_^xscvF{6Or^SZW_fujiECF@E9hZV|rqVuA4;8y!JIgftP|1Ectp zAENx3Bn5GV;rHSR>_J&H*pQkYBCPt<*>*q1j$i_Y zf@C(u2D3^FWr9m|gzXeEW)U4jmO{RRTL&D&HR9Lz8IMg(8M(fEYsg>!3j>X=&pu_QDcR(9} zLbJO~CuZXCow<6fA-SZ2M#1JYzC5?uWF_VjR_ZSdrHv$;RmOwy2PuvCUxpk^DLH_- zv^K;mlEj~4s-N6aE{x^8^nK!}q}SZZbe{=?rtNNE%-#L_t&a{`wZjk(YOV2IPD(G8 zDd0&EuLC$Cj4{^#lSYxBSeg})jTfHH;n~v)rnVt+K6IsL$LOMWadGy`QNOAO>0%(< zsYa|D5I*9d*=kwB70k*^t_Q#fNiM-_AcG&oCfV0;4jeJf2`LRQK61P;KERsY{iu+! z@-^9~3%O8qg*K^ofLj|3OznL&sOqv(e_3_|s%?qWRT!5WKuhebN-o4ItTgd-AEt#1da)B2|ZNmH8n<7jv~^{zT|hg zOGI2!k8*0@Vk?Cl-U`4XK6j(*^ZL(lZqk#606*>cPIP#W3dR7|hQ>F-9HV0Bz zq9_^}Uj-&i;w>z8dWM2RyP))e4{=0%(uWe__jo?0DQC4Ct>3ztHQkx&TSBSBLl;u& zcD7Pn7WBPNHNc<$k6j}70xO>1`na?1qPpOb_No{*1Rq@@sR97FP}o{DcnoZ>+| z1vtleE?rO0GMlZJ?OTUl=d`rR3X&(Tp}11ikrnNreFGO!zeTIw^lI?|J1ztW3&SY! zIROn4-IN%`pZMR0!EG&gY`Z5lcOrA8EpCA)V>Rc@uM}VQBHZk??{n0<@(T32fKR2* z()wVN>hn2H!wN8=cw+tOSAA35_2bWK*RI!SQ&=L;P-4Wn|E6E{vp2{_NAoLH&!*Z- z@>o~owUc2I4H#_jLz{pWqK0cMMS|{6a*CX61EU7t$m^kR)VwUd?ah_jHJUa$yw_o? zjeDz2NX@6h1K?V#+1W5;qxw$oPzLLV_9nV9_qVziJOCu!C`Ls=o|tPm!=Zrv`!ooT z@rnMc`CN}6VA4L$@K1K1mG&ZiZ?x;m)W^sjN7UiVL8N2tBG zv0sk>)$zaRVOQVGp_;a50c5xwKnz<2(|Lf^Ma(dQ8ADGij7-jGxcDsa-Dp@|-3dMO z^=Uha7FGK!e3_THu8($y%A+aD`l|cpx>1pG5+`fI0s0234@I{o+x(X@2Lu33NMu_h zTb63ihq&FvW4=EUe?}4f6W|=S5&pjjb}~gB0tnlGw*-)XJA$wTHDEMT)+JjCY5e9J z?GhfJjrRsyLD#)@wNj0|TUVJJk{8l8wv-xr9z|0g<%N;tBFC2Ly} z&3@FZBt8v5_lwA1CG$;`+stX9*V%#ZoVXK5Bg5|g~ zQP|#)B5TyV_u)j_7uErvWMY=LfbW}cVW3K{eDmX?kBYTluy-&fQ~gS$&b8w&E`?d= z4CW58#??Mv8$jyv#jpLC(5F%cxL!R}*bK+1EVu$f%2PB>Hs%!O4Rv_YsWe!RF~;r2 z3j}!y)ICC=eJMY{-6!19Io?I&z;5^^@?vObWjm|Vd)At=tUf0CthLi7jxIa4Yvbf< zF-5(7HLH_*4<>`O&^3V(3SPEO#FtoXNZUDR#M00*51-kIsPnQf+HZiQPg z+NISqG01wG;cDV@P!|4EzRtEY-w|r1vDd9TApcBhO-yC16Xs;vvkbG3MYK=P&A^;T zx-TX2?Sc0;*~!z+Tgc8Y$9e@ahk!1V=UUz+=V#je>*TKKw*Aw1T0PZVU#0!&dp^`c zf(%)B0?8XsM-%X4X-A_CFf$EQ9G#yU^<&`g^#>-nhNOqsQx9GvX z#C|*O*naW*Sv0|kJwsBR;uzs+fQe>CloScK{GST{kYFia8BOMcRWn>24qI=J*Cunn z1a4v|33!Ia{Q2U-c6G{{C(4nd27uKO2C<3_={oBfvnfyvth>#r=6-MIr2Cnf@yf`5 z*Io^ojmM9r&e5}0IBW8T-o}el9M)Ln1xtze7xF-pU=!`b|7QZoAOscP86uIQn4**N?`Fbwk@R>FM z=R=#LmazL*Ag{ERT5lvTF5*F!2;YE^_M?sw&-)A_sswzmu-)s!P1bI`_6m)FDu(E` zczT_c-8LITDqd|v>YH-u7Hix`q%1M(5-Fu>RFePdCW~Sp+IVR$2pb!DypP|Acj3~9 zu)D$wlUHZ9Oh@e|#QA<@4~4`xlUAWh()#T!8IGx`$^X+((vjl&iV zmw20omyQ+7Z$;*FF)UjBaP>7mKQ-9JG-}tC-0zX}2#=95i0H*JK-ZpY({r1LC~i=a zIa7W^z;P8R-piKC``Y1m1z8pUr7yX*`y$dz<+vx6MNDFv+^~O)Nc-3`c!A-v9hD}DaMM!qYpaCprOk10BB89!2Qa^%OFRPx2`xaLTCsDH|j0MlaH*kD6L zkZv^?qwNrb!`~Ui#iea;I@7GF)870i7z^Mpv1Xlu315v6%OGf|=c~8V^Cp|t%F>o@ zl>0Yi9&w@R`^BdzfU3OV<+a=G@NBG zDg z<|cOB-IUZ1-1HwCT+;EV6>4rmMOFEEcUouHsg_3-AK7@DM!9tq`!UW7?U7{$`&#Z~qR;#e+6bgax6cbfTft>!$$-!lNjZx;bi z)Jqi=x~@Ms@?K%ffpck_xI8>1H&TsfMSqXBc07qN(we6u2<_ph(|niqx$z@9l8&^J zqcEQO84I_WXXOCe-1~NKEjeX<4LN1U)91)Oh6}bZS)vl(ARO)D?F>h zNhXm<4)TpvmKyx+oO--AbDaR%OLm57?6hqgkLkvlDtmnw4lNPmmb3@pUSYt&wM{Kj zsRh68mYJa%u@mT=#)QdppNLLs?6*m>9harPN{vl*-vX?CBLv~WX%O0`STyFTh;Z#{S~M6BZSmWWl=t;qQr^`KSubEmf0jUIC{OWS z@-kg)%*hfLf0lx_qs_jjwp}a!+W63|c@n1BW*^OFf@_`2f>V1iu!37dvbtr#%&uw6 zasGrYIxdE~JX5LOo_hvMU*mH=&Dct>i`*ne>9c!#Qn27dlG0kSj)WeP%TO14r#v&MJ_}2ta5xa0$XJ zfj}0LBGpI+?J$XrazhW@eQ)J{H;qZd65CS3lFe_9MlfnthGuD;G;^|UH;W~ieA8Ud zINh{aIl$G=;%0X-nZlg5t2Tz{J?hF1Kbi^=}{y zFY84=el^ljsvUpTmt*gNI29#hNyQ>FM4>@YqwFNHk`-Y~foI7;K(T-XRaz7Bo*=NA z=e=8FQVPVG%l=7|HzVM@+K~6jN%)MT7RXHMaZ9dC&;FQNDnFMV1SG~azaV%GQK-t!%y}kXc=~UDIWo6drI~dz89M9RG z@L6v2240AH``)F6p3C)LZ5N#u=7#&cY+4*i%yDGI_PX#?%D`EPmUMrqW>AoO(}C^}CfkqM zPX~Lh+k!L>1hz*E)%1AvwUx9kT?odZBH*$%9~nzhdnIj*;v`xu49N1yPO%*bc7)}p z7Ahxo_@(jy&8yXhykhP}@ExvtPcsi56g$ExClcO@#|D`T5Y_*#iiOJxh?)5SM~v7y zj%b#&EcyJ?gko?BozkZpHI<5m1|TDK)MQ)$6o@0|BS~Vj^}A=;_xBe&LEaV5A?}Q`C){#ks34#vJJsraik;1W3et|M6ehCciq{p4>V=9} zIxd0ezw_k|reZ1f7)?6TGkJH62vD8IXiK~DChTjY%Mxldp)J;5d%rlF?8~p_xzW7h z?PxGI3$Kq+Y$lH)MQJC6w2eeknkp}lSAuenp&1D_BchH5((up80l%BXhN$BJpm@IU zpx5oT(gdv5ziu~K(^aED{KY@$@}fKS_iJ^3V3=vL{>n1a3|ueISUKPOWbAex-wyQX z@2hOT?B`7q-kIzS^yn&VQ-P&?@{St_p=Fi3G6;Ae3U%XX-7W+mHmB<-5V@;P~HE_y&q+_kRA5PD2tF)d5H#Gl%%*`hA{RhTOr&k0< zZ>F8##Z8x;`B(Ef^u$kJWdT8NhXMwc#RKL_XZ$yy-3lLgm>HCviRnL0Vo(-V_H?Iz z!@*ctxS7%u@qwuTw%#TQWiw4zEp{Jvn{EFd_J3vj&wpxH=Y6|7AJ;C2Vq2cyre@~i zwR)+_%5Q2(C{gR1ofV!u87zw|i_DI|DWfT-*j(6HKya?V=Sxb46w1OF(w^N(!Whc* zfZSA31DYy90}`PJMg|6;E~73-U8-P$<1 zK%QG$TVPu``sX=%Z)e|`CAQWuj*U#<8QL2eAR))-Y~;ryz-R}~RX|RvysDIfPUFJSQx)C%FVj|K>Rv2zc{)_WMZZC(G94nh$_hF z$$=39sAU+4WBszU07*!Ewdl%?W?dEBDH~I0`-zH!*t-`@irxQ_lK3J0ieH{Q6hy&i zBPAUeSzH7EWom721u6f-xI=dJ`x4mpM>)HL=X3QDk?0g~3rzLBpMdhq*!nrXy*f5F zr?8efKxA!aWpH_P37W{p!T7=yQmLt(z2&1GkX~3@8hiS)dh_Eg)rIvv99)q5Cit8A zQ_}35&ivfSy6D&WJ>s%?7{`y&qeE}kIpA4*Wocw_`C0Hm{%8L%AD_Lkz4`95U-lby ztRJ%^T}e{~L#^mLAMCYVdueuUXJl~}ui)|?F+EIy;-~mu=ivOk;QE6h)6WbA(BTgN zcN(XQOWPZ$1^etfl>qMN{hRjuTab~M*awkOM?!%Eh@KS{H6SrH5PD$p_w_^n9!@t8 zcjiXtXZ9_?`0eI(H`3Jl!pIt~x6jz&H@RPw?t5ECK zfUwR3v?|QZh7Qsj*Xbsyv5}xchb?bVc0p;9(3 zr}7Ej##`tM(rK2YO5bN^30y@~A+}1l`HC>|PytO}yWYumj3VRIbX@vOeBlTwTPK!YW z_i`N2yHj7*5?LdcwI)zSh`90qTozyL_|(KIO~G7jPvKYf!iG;sBDIhVwe%)yc>O$v z@=A`Rm#<9wi7fd#W1?I^#khL)ziu9_v&TpGvO)S<`=wae#O?kC?6}LD z_l(~tZ|PREG0b9*as(#W zmRtLGgfq{mAWGS>$jV%8(5@RfnLYUVAP>6F!YPHX4y4Mqg8NS2n5Ttl;2loms28v= zXByfIJ>1+i2^$?>vdHNGz)%Gd-tM_|mJeh-#gY2ijF1-!(sK?LM!?*$UeVRbh2mKE z(CR$XjB}5nMv-81xmuWQVx^s=CF2_~zpC0v4>VhUF~}uM<+(t|c5ShVkL>D^?z%+UO&5 zIHxH{$$`0qhx?jA7xhJ`%sBpShR)QQ^rPA#G}_>L?{yWInH!c2I-` zGQ*}o=Act5QHe2tCV4GWZ+LS*B~KJ<5I^}I@E2PN$(3=;zUkTKRp<3psh{a`ij1<# zdliDQ>BE{z=PSlS|B`z4RThnk2V0eOa%C4*O24v|5a_1A#E|hLTt7HFFrlY4r$mf_ zXr8Tf6|x^f8g%age<=vpa6l1u-R|AW(Vk@Haq(M~q-PqyKL7+RCHCrWun>+4h8{DV z^p^3SNbNK8vec#eytYAJO4d#;?F90f=hOmo=0c__;3gcF%Au`(Le$})iDERW^AYxz zEC~hi_)^^JO+l-t^PVEyaf=gSRboJh?IQPGtY_kYj67#Po#ZFy8$+`j2-TiD=#V?r z`1k~}w+I@*EsIj_`ZSRA7nOuK&N!ZZKH{vL-rk$9P{y}RUMr;Z1yckyt;`O0j?Ado zUKiP8MvHLUXhza++SI}viVORW#xc#|LbvyT`J^-OA$ zO(GdpmVfnm?;n8!9rqqZ#09vXFKg4XitjcK}a&@6!)lSD;Z~s$fEgiF;c(8rurzl<+ z3d@8gtwP3qtYq<}9+p0D=$KO+cR9RF%AL6v(O0e~jKi>4z7-rc7;(-aOkhFSqSg5G zl#~E?>N->=1dCq#)|NAna;GD@!b(Xe5H8x&fL}8eUu@lYRvGXA87y7$VSSP?)F-Cf zAlY8eju}V_ZCL|dAG};h;m~iD7>|JZ?fw$dNNK)sBF&2;6?1BHSOOv-0-6N%*%zfi zto6aOJ{C-D2B~S4U=GcFCy%`U_vj7`7g`_CNjDAOh(p1OD7c1wy6YW!j(f0@swlpK zcC9y=rJ);*ak4o@t=6ZgTdQi4tungq;9n@5L_)Whjq0} zcjAD-_$0wZBKn#w)xLq+qTa|e5v+@Il1y!E9T zlBBG?Tq|x3eT0vLI#&Q2s+ZdLbFfKW$20HN{BWRNXrreN1*<9PaxBOZJ1gL~OKmCW z0LmDcp5*p9Db?kz3aB&$AG`7O5@D=2>X}3dw3^#agjMq)VfJzAG0Irw0pb z&w4Qj|J&eIu3odK4EVXNHV$IEkLp0rMivp2)57^ywkfN6#JNMcWe+|!E`3_Mx!S97;eFzT9FM0e5-CY5%-;du|&=@RcjP=HNqd7njh&70kDD z`oR5WlY+$qVg0VT*ZDVnPUyUMnewK|5D1&dQ@a}}G<&&Rw_iC;S7?jYfkJX%III5K zt>1)K&m!XRFWWHoTsXlY6B4~@hbbNYt@bTOxwAO$@U-rvL#jQtp zxbrritkg-T(e(uz%|3|BDE}~8cRxPp1J`|TAE+~C1~x#MBW2yBn1hVr#p;oEV7Qhl zaHx^um--Ad#fRI-`QyqAcnEDn#HzPfi5QVk_jck=g-Y6~qDx0`u9XLgbJ`(=sxE*1 zeP0jv8}`Lq6B%jo|=u>e8zA7?UgI^yEOPBn@cP1**JS%quecmTGA? z9WS6IpJUA%rUW>Y!>;_&(dShB{n?`l;QT@IrB$ZU0wKnzg0Xb)q&-K%l8jrc_{)rb zIGAK49dA$Y$vBIJdOTlKHF%}uF9U$hz(+IJ9Gs7{(7}wC_p93(tSc&C?Axsn)*ui=(26ENE0{6^y%LX6O&!t zZJ*swUt+DQSma6o4X@ORXm+zv-NMyG%SRBrfZ8?_7yxi9Sq)o!Dbaw3nypFH;i9z9 z>N&$=+?5I!_4GWhc7xculbsi9eh@v`^xH?)lA=x5PPlZtcuJgS+LeVVE8Ux>MYj9v zJ3qxNi@1kW7!$&7POq8Ub3`=f$3L;y)cqmp`}zf=)(@i0U)BXfjEhrz_o%NcSZq8= z9LYj*$^meQg}0|2>yPuh#_sr7Kw_{;qulR}4FneH8i;U-;s7OhW zAr%lH&msI*bUbl!&{yvTGD8hI*A7rRe%-?!nV`|Orq^(wFyWKG*au`I z;;B*Dj-jyWs%unZN!=feveD_2d(mdkr)^8@BBXSALHY`5uY`g6{V&=`=gNB^-Yvl0)Xd&mu3KfF@4;|MdkVm(*(OR{+jV!kI z9fjM6W$Qg3F8QSaSN~pySIGz`6-C_a4cn?d?t!B3?DKE98HXSKZFW&E>-g{)#+V3f zw%dSG1V*maW5VFYYj#=@uIGKth#qZ<$H0>E*cDqeFhZDwi4^$iK}+ zGc=y}VNz~pooSTgr1F^fvH)I)j`iUly=^AVK_!tZ1>4i4<^~h;ooA}j;uPgz!JVG8 zjl?h~3|ADFJQ;jRlb5er}Pw zfK?I$G*XsF5M|cG{fwFi47yrvLHhQB6GB z`=HH<9zB|7O@!WpA{cI1(x{GycrDpz$B2qd=L>=gAAHPI|9e10NK8?0k-CWbhCNX? z5wi{2Br&9&jRKpDifapz$oEG1v!afed{%}9BuEZmCKSth!kU;YPyqP*!g=3@D`y5o zR+2BPfWDbG*nNQnJ}^4rk0vbh?8%QlhcHzj+!qiaj2o&Wuc0$0l-dvM(G_^lM>wOB z8*dB9T|*m5szF>%&@*ln#_R>mR?)=yQ`c!?D!b)~*seQ@0D*5_6X&;|rrz8Yt|MZ| zebkTp+W;cw+pO48xdC(HBA#g+W*YWoE(}l~jX0|}YoW^xmzDq}u`R30@N`3DOOW2= z%8UcA0Mqba379dwV}iVj@`3*mx$ZCzb5w zk&L&2$wb1ezLTFSSeDxhv9a?WO51eG+}}_TS>{>m!QJ)O!{|$Apas*gItAAmStk;z zkSKT+weVWEoB+g^5=VnJNCzeY1xST;O+02nvdMLMEEAfZYu9~uzPjChBuf-)3#zB< z(&@}BX|xPFTCeinj!GmDEdM%=qrF~!v~Elp3+_cs4-iUst)%#gqm_09MZ{NKs|gfg zr(?rS`31Zbs!aS+Y$0fPYIz)DlYHdDHD^7Rm65dIQ^1mIz<@+!(!j!05oj&Bk|a@f znvV6LnwR@xD;TJPek5pdViyTRff5vZL zbf|>WkKl|TGgj9knJEn(OBS#3?C5ag+rU_2#z|Zc!6=8BasLy29R02T!VHE#apX&D z6)lCY!^Z600mNLY&{%}O7~=c>U?VxL9>9ytdI@p%hu%9_9fQ{K_|^z6ZG7zdRd`0p z6F}sNV~20%OSTF`N;w}59nq{Ts{@`0R7SdEuW0+88@nv-Nw1$E0Lj| z)k<}&n=$YRy=%7vs$1WK&q6sgzVSML$9=!usRNLC1}&#!n-mhVx_>RkHOXpscAfL_ z;sUoss9R#@ramw?9?aWCb;yg035S#x0J@LzoSS^}fm*TCW<~>ndA=iR=}~P7{i(=S z8dwd**l&`JV&6g3eBJ%R7s3v0r!!+!-N?`G+qn)YHO9tO9R+b4S(${ZmZ!xN0|6|~ zOr^PA367qwOPZ?THM-PJiHa>R>{K00mVA zI-2((qjnT=8nxdQD#oW5PE0(6^~1*!OX{*G`CcO);$$&s=C+cQ^5~kZT$k-1^cj>C z5%~G4QhU!8mjNKzv=Zi#OR-DUn=`yWFlxEz8B?0j>{k$ZxhFN61f?X}9OfV`VGOv2b^f=`w%MLE<2C zC7<#XdB~uARWDM>K$xK=WXlRr!?5#zBc~Zsr2(W}D@nt>;L3;{M-36oHgMm?Y&`gH zJ6XJ{AxhYh3FXM-N>^OwGtKm!U`xyR6a0O3V5?Mn8smQHjue+w1FzJEt-xP(%#; z$-Ob_DWrs-kPAf2v~?*jKkG zbcoS~g2;r$|IC{td<qXbPve}rjd!cbk6_iTHcdjJ|DeqST?H>^83l>x$89O| zS(gdgs_u%jR9*?id0VnqUy*qf%f?_{GLTQ#KFOQCN;RSvV*kDU>&HD+3wD$9hlZQ)^BqxgKZ< zr(M_W1k9JlH77Owdtxnr@aQ9G&S*1Fh~yi0NmxZTp=__*k73FqLZ;=f3s{UNt)u$< z>2$IZ3A;??woMb1ZT3cWoZSA8Zacz{91a)ipR^v%uuc+c58i45p1e!@O_Mq!lIR0jLeIdcJm zg6GOt>ZC=Q;C6!^dYhE_Wot#ZC^MQi0jLFN&A@XrLVvnEi7AzNM>48dv%?$oU-Abt zcDA$(K7v7IyEPwKR<^ozi=sg!$^EJ!Sl9pNe4C=cujiIANp z6G|(PdVsUEzNgsaC6}Yawlul>8bdqze)08W{tA?vDoUA{^ zU%-sRY-8kUX+rIktrLB4NKGDW1;#@1@YRaEJ9K@S;xhATj==9iUkIHrr^Cp(FUby# zF(vN`kbx`^Iv}Ai-kmYa4Jj{X-PQZ^b}4r93IMZ#tk}3ZJk}dOn*reiqM5dx+z93{ zsoQ$OWzJsx*LdO8lMQ2LP@C&t)aR*zlN)eh7!n?wq2Jh~W;=_05$j&XNf>zPqT&@* zqwRB_m?C*wY}GisfW8yQd8PCS^yvutpE8n39v*Yj&`Z_Lvmo$QSA#pL&~>+1DJR`y z55O+u8lp93I_N#KKn=Kn+GBav(6<4H00w-; za|vj8_02?%hVUSz{Xzh9nuZeUvuhU;2;gR{l4(ga0OQx7Wr)O-{b$Y^5sZ)Q2Ict% zSoJEr#4pOpG4UCkfC+jB5Ak~(ZQ-AL1w;c)egqE+_Kf5|n15GTF^$ZfQ6GGFp6_g--99CFGn59Z+LEyGr2Wgojk^ND)@ zz4B9?j>5fz#T3<(`^gM^e&Es(vegiRzAE)32rro!?*EXh45>#*=XwP3ecXqXlsG#o z4`^fQdU%PGiViOjs+o^iZ_=X(1O%Y9-qG@RPPf{z;;R=PW#vIkKzYtBt@UlYw$cPo zDMsa*8)u4z*Y`Coa;M3R%(g1Ke&Ycm$y+mw6Kqx-x<#cfCgQ4n+|oENc=~K^I!T6U zY|kMsS+z~1HxJ2bD9iv>ISsbgB-K0~R^Qb{j^=$945tT;0YaNofq)UwKkze6@{IzsL`+c+d?} zQYl8-af90{! zrSqv!J5|0gO{KDzMaRoC0kloIB7?uDYzuvM_H~j6vmuT>)aaJNgT4Es&~4`Ko_5|i zv>`_kHvYkbb=ew9PBf89nAg&=85wJKBQ0b7c^aa6^)`Hsp&0Ygx&9yFWl0&Tdwg?8 zHQ3R_U%jda*?zz;D0Vi#$r5`v;nvBx!VheP!6+sM#t ziRGnt%u-q4MI1O)0f6lj`?VWoq`}L!AkU{)Yq#JlSmYJU1IMlP8LN9YG1TsfmL@mE zvs~$z9H`m91>x|Gyy|dQmXco3R-;|N;dKipekEe=EBEy>0y(I;qSC=-xaQ!^3n<8jq~5@nsNvQzSkg_u zRmn#TU<<=*qN|$=k_Nu2kG1PU&NVAc+3)SM$gBcSvdq=IU}QN&!;Ds2QCYVhfp;1ac*h<7INYk_rWxlw)9KcMKRJeQI6LC z{LHVCr}9}O5fBH@Y%N$23y#2Ar(35W7ma2r@+$hwxr056*Ro%4C#TAPMgFP2-XXA$ zNypZ68{!&XDjD336>IZ5p_h&E!5{iww`dGwdhPG^kt^(;y}}vf*V#ixkw%|gtYMsn z(S%!i>n==gV%)AwYUdG=UWGo-a;9CUM=0y^d-~MB^m%Z3^?_?7pa%9?xOyp` z)y!PTcI)CMUBH$LrH&+xb59U<{cG%GzVz{9r_Bi2KxA0ls35WZr9fKjvK@w(4+pRt zM599&ZRRdnw2v5hrN`9>ZKfq_YAy;-C1y*jnSJ%v++*1g+Xc0FsU{3U%Uo`+lI2u% z4FD-7WhqbX$%_}&P~M(G5ZI0{^R%7Q>L_&p-I^axuxyH^6fvPzj?iG^BWO=EDiukwOXXV3I zps6<7k}j24ecPa6#BAC@XK8^)#e_g66&-WklbQSH`8zdvKeFuD@#L|@r%Zj+%d&SM z`@y-Wf@oTvwyiIE$-A@4NX9(rUe&-_dC}{mbgKsgZR5z0B$X&DX(mes6I(VoH-Pj# zdR8py<3g2Q`dS(eu@UcQ4=cTy^mY-t7!Fu%@ckE;rK+~&>A^FRaHZKYJId2GQlN*! zks4AUuVI@l=16$Y^HWA24q`A@_7NDS$lign{5YxlzEcLli({$K;^ABR^bMF^U;sY3; zFNzzbmQ%8(qdg`iG5k%$w_C+4!?p~eLX45}{r22?R4#QaM0i+llMT60Yt6Oc^PIVe zFL8Ej4zLElWpj+#)iW@T@Os`etdse+Ex?5BjKu27BIi<^)4ooom{6 zar737llz^ITlV|1hL9iV)xe{%?-n5gC)(=h_vJd~?gq@n2WP_#51htDgi_bDT4{Txv;au$ z`7%aoeC&N@xY##p}! zQ4dm;^@DgQS5n`=6TmLg!Yi@hJVD6N% zMQeN5=l=&;K&HQ75$g%Mj*N&6tq4{g8=1XY8a4m2lJKq|+%iEuZnMwg$#jK-{Z8GZ zGlu&xU{WrL{-~YL*j`9g^4KF0<2`HYIMnTcvdN695Oh~U0vDdC>qN>v*|1Hlj(3xI zv5+^U9r%Q+p)&*vJUx4V=eEwx)eQMf)}fV7ZeQ%{%Y*3wbyo;z!7YchMyq{L^~^rx z1Q`r;ovF=rAT;l8&9LaOiiDGH`L|qf1e~N11qgmOBgg?b0B@_U>Ua-t@6{cezF{@T z^~ofx^;-j&cwqs>V8X!9C5r-!(m*Ob7HsUKgHy0VQ9pR5orz6k$NA0Vi9t zzoJf$aF}_==K-N<;A)uYUg-g)g87%v=qyISb@(w-H|xqevNWG}+h9eH@{F!(9jH)K ztJ{-7hKnu~pU7c<3)PtWJ6=V0M@Tfr2kKb6@SE-d1{dR9-FsfhaTU>% z7bPf>>@>IUWcGd0O+F>e$vrCJ#a*^#}6NT%#c2SzZvnP8qfo$?tp zteMXQI`G}19MwxK^^ILb)kkVoZNS>a&yV_TFt8@Hi1xXPP0A!5oUN%qk7k!AhvOBR z9CJ3!zt!Mbd{83)XSPnTG>b&6QUiIMIo4?V3Ljs8rDErO#-*?D3Ux?_5+=N5)(DG= z#ml_0;k2(^tDn?hux|0fnXeh!>#(T9gH2LN(1jUgzMk2S1W7!kFpcNRQh%FJDgcyz z`i19Ujnm*x04AOk<0;yP${E5kOVXhLrua?)1Vv2it1|YF#cw<2BlYA|H>spADYd0i zs<-oh7OLgH!1%;z2Z<>)Zg+Xw(;+d?riW;PBWT!V^k;qGa#sYek_imRsCluh_Cv|s zUWEg{E03j6#n(Drn=@oDm*V>5D+&X>&l6gNGusgD_(%o(tHmy2b;{;g9q8Ew;*c`- zNhY>Vg}k`P^5j%yk9KQ$9MOiC3KL9qaQQ@kvj)#;30~&Y#@kCdx_e0fMBzq2=1*rM zu9_`+8uh^BQhj);^q+kI2={c!2nkth)O(Jgcc%OPb2+gn*xF&zzMC5jR3v`qYa;0p z@xd+QPyM-|6JI#>OZ?G(VfYbL9J9$FG8%AE)eVYF0Ut`V3=ll8hVz&O*s;Pbwjv;Z ztx$;Zd+a17X7kx@vqciD!vF$Ici%tlFk z&cu8HL*2H(V)jGrNo7ylZz7B^aWCWyP5dEI>O`kTDX0v(A-%FHJ?eg98O*b&qmR%B zOvMWE;$uBhbEEC|Iz;a-{0X6hI4YrkcGdHbPCmxAMJs095<_$HK1VIi64$+jnl+i6 zp^SwoiyFv@gQZ!Vh|n^*4GTGH>N!4rAhdO1BMj8U7RwDxygJj9uf42AoKS6Zdi_lm zb=rB~mXBpcGq#n|&mcQf<>FR&>?LiD+%5BqWF{b%McOt}wft17<2}L4*}<59p9ppC zj##B|1bwR@f+V07jOUNsz}I9E$O-nJG1**dwC_!5&!54vkg_qehe%GRhHtSW{?_7C zkc)d*R52efqn~Ugw8ZBnHaA(i0ow-g$>ergZ=1hBRh}Fw;BZkd2~uzmibq?4~SSj*6WQi}r9hP2)Zc>oUOVcZZ^LP5o4ub8)`A`;^mKM8}9Ocs`Z+M!BOZPuL8L9YZjX9p4NNQF)ASyF^yEDd|uolofzeMh16 zr{as~#JaQMZ=He)pwNnc!BrJfHo{C%&}N7B`|9t~pSP3%f`Nq{h&KN7u+PD)&2Vrp-aA z^_-+3ox`yT91VRBoT99Th2VK^rJ*A?)`>?XNaR7-tSa)Bq> zc)3KLRwv2^bD{4si5i}JA@F$BPu%nZv6JH z3H|rEz3H%jtZeIVV2RA8inBeElTna6Kd5`R`yX<9{t#+~ggF@3mUVax0;N3u$AlS- z78Rgm2@u;66wJPO-rl3dE^-e<3q$rQd5tNcyaO46XjYNEAt%|C#wWfZpJrb|$~g^P zSe#Ckh-D*}J%N+$7*sp`~q^A}m~l@}&yMg<;dPBjfm z+RVkCAnapi0+k!}#CALzSX@Ge1gdv(}hw<}$FZVVrQM;KMIkyPWaLQjx z8GYt|zi(;I2un7^Us7KwpL=O?97g>02p63fqSAgPr)Set4}JgY^15r-N}9>$Fqb8Y z$rUIm-CSAnR1HOcva@k?Q6w>dncePj6fxpS0$HY64;hy2Fga8_s$!7V2nv?~iz3}B zjQ`=q%=KRA$nkkeRFAZQwexrYcXymKUc~NyjLF})F%2ZDKs&*G@<$MQJlQJE2^l95 z@_78g#^T{@S%?^#6gA;3*+{ag^ikOy0H%UMN7G*2xc%#3@bQcN@2;qvx2KvaL=`UG zt7NfZ&Fa&+r0&3cXwgJUNvJGyfbqsFpHZYuj z7r7Lj531NnqE()>eWV_j3yY2_MVy{xVNYJbI|>Kotm`iFkYf&94^C676j6|tnX$LmgO=}@7PQR_tkn4^ikfmEIq=E ziB)qa>cpL6@!N>dw_b*^Oabr0k%~Zn_skJkK1y8E>%n)7XGL|)74M6{;!t0igph3q zO^r}@L})fTi)^i|`b%yttz-pDWehJC^mr*SUEz^eO@IbUw$@||r_A~t4=(VnMRsI5 z!XUtRJeA6Kic&B1?6G%Oiq%NTay)zK%2@wn&%Esmmh1pQ%Bt(#Z1!C4s zMRu_%nmN&tUR`6R^?-7@-tm(en^a_0CpD^ql?$lt>@DyKzk)9L+NgTpo+5)6}5JN+zpe6M|1nOkj+quz)cjF(@<2CE;_m}wF6?pm6D38vl0!&Ct|IthwMzSfw0zauf&8B^9tJl7mp_K}aqGlv11+s!}k$Sah;NV(v6z*RMc3Dm1W0S!VXQ? zz?-ASLq~B}VGe;ejNCGb^f`ID7xesx|EU(M}YP%ZS)wIQ8*;!*ZJE?V^)Ohu!Y>+?%3+$dK5j=qW39;c3 zon6P|7dno4`!QUm+L*Xo+@Z5GYyx;47vPL1cCoC!mjP4(;X4z5Ip%5w{bfD>1`i~r zul!%X_}wpYKj%6I1X@P)zhfL6DEJedqdKenb)oV$w=R@ORw>r97^&QqKt-wkIW>jh zz|PZzndA%EE0Qe$u2GsBB(IcCPSFND4dSgWc0*>g+O=S1byQRaO)j#@&?E!AFWRAJ zVfJ2;Zs?(HIu*8mWfplKtiVpbLzp_|d)tn`mMe-EgD=^AFPa%R?L~;=-_G8VR+boZ z3_Wxg*>~uDvuZqnO>yATwL18GyMzk9kI&aD(N{XRrmUF_-)Fe;BJYD7`DhRCM&08L z997#jrwiC1Z|AG>uqFvE(RM+8$ihlwBwy|tTSPYnxw(a~Xe1 z9KgoZrUtA2GE7oNMW->j4*s4nWWYvF`>PF}1+yE%Oz3Ap8>XUIvzbi|Svcy4Ju(fK%az%o(N|IaJbY z8u+WZ{1|6>YW3IsrVp8pH=hc?&=rJ}8SEi_Tu6dilyV(Z{iA1_=ROSuQXva{55#gQ zNBl2bb>dAYu_-E(ARNeuXIbIpr0#FKi^8C7iU;j-6P3C8eSe}?(HJ|-UsAiM!&E_e zW6WEB)wv429@5o1Fo5GsL&5J7ybMrnWK`^?<(}D_OZ4ov7eHVUqz8rKuvBf}M5#R% zh!(JH8{#~W4-_UwCthz;X;wLCdnJv$__&#N+(3~X$94Xse#A~dRsS!fqfi+B-;I(m z&l79M&dG`fzl%X*JK?LmbjQr$Th#o<8h$5#uZ6^Zm;G4FDYwxx3#CvNjYIwcR%7|- zr%}H*EmRgwQimxdC)Fi7+o(jcUJTw%iW^3)oAvWHI0SbVaP*1X&hFEX9D zCXqff1xT#Sceh9V8EA?cx2^~|*f&sR7_(=!`0bs(KUym-zGx?3y7WO=3ARG|TX zg}&pNrw=vP2!hGx8Q;Do4U7t;aSMgiY--0t@QKAQOMc6a!0&jxtiw_eO7t+4_pD47 z{27BF<>&Q@C5h~%mgf+9MG3jtg_@SEI0Ix;^SHQOub8OHKrlscz``8CKC^P%tU)WoI6 z1=)jR$&4Go?f%jUXla_2x{=IBnis;`x%lO$bto$K^Zt&B(s;6G%rCVhBpG4KWk_Qo zYHhYzBC~_ub?C%x_&g8jnxA=pw(BaB7G;U7T_64#m|TP{4MskT?Ug?eL>5MBsoKH} z1Og>+4_O*{2t%kPW0=@iRcW_k8ozbjc1UE2*hT1@pdu=QcO?1)%3T5j4XMqVA4V91 z0QaqOESV>bzl2QM$i2P=luu#+7pN)9{kV;88ornTk53dGbXv@a!v9i#3YD_wpQBjR zM7-8gyc?LZ0HW9ig~&OcR@^!QWAZ5;+$}WVga0W1IoXafzRckp1uA z@5bg;l0Rfo)KnLr()$B{jbM5-zVYjCon3g6=-~`bWUW1k7YT?#LEhEef9DoYjXxZ- z4$`XK8zT?UB*G{g4$Czh!8_lQSPP0)zaPM3$~z|28~{@06E`QpJzSpew1NA0uR5!W zie4O^fSgX!*?prQ2aHT^Y_wI40q^m&GEC! zU@!-(qG+dbHU8$1jz_=my#-(5CA|(b0+e1nptSx{lG2p_#M>@F}2QWM>qG|G#f^?Ekv$e>aPqp>VE3jl8t;t-!zNaWN zVpGE}2pPr{a9}d{G4beyNfYZ$3~VRR6;6pxA)tnNrTH_@I7iXhY|ve3_cw3JMwmN& zs;}ArG?c}*MYbv!&p*S*P4+5#wEg)mHELYm$8-U^Un%o{Z|o{J5d5bA!-dQ)Jl81P zA&Ko*=gMKGEz7Ov{K#hT-JPtD82)5+QU24$U99)zl>Jr!Y_kyPiS-}19nmt4P@K}``fs?z@Cwd^%~dx-}a zh)*K2h}mp^8%Gx$F|0ruoo%@IcjuMoP@DA-!68iDU;%E@i(rRW{3cdC6Y(a+E)aol zmAnHXF6=_S`)+q3 zc(BqF&!1;cVXl>^JuYbGSrdbs_R262&nNuv2gGOz9|;VwGqTTt&xxtlz(#91C=|8~ zq#nNYI*&&u$C<4qlpC0Kh4C@>asFG|lb<-1Phm+WprxZ*IO#+|!O}02=z?t7QmM@X-e=v_Tp;I^Sv z_;B#xTC6WJLAbfRJm)$~uRkw(*949NQfIIl*`C+Y!i0LK3Fz_l(z@!bUrhaScQ^^# zGI8DE?RLfiZX|&MF@qrby-RnItO6v#Zs~Y`jb7bw>=!CDCn;~Ts2f=yThjCL(@b)I9?bWuvJt$o{D0xfR3Aaxr03%DfhtF$ik+FD`a~ zfJOq~S+ik{wS|9Q(8&DnhIRtX?8vvJ7);|!pti1Ad-9|jtcjbq-^AT|&;^zv4hidX z*aCvhMSqpk$aiKI&zv~A+*o>&OIXT(!sgz7Kf=knDGJbP6a!dqUOhn&R|ZsxuDH4t zn3aY)C&hohWCNIG-KdGi4TuszX}|8Npn5D&3m*%it{iDIA~>b=*2r(}8W4R#NpPik zEVD>hVpkFtz&CrO7T1`tM%G$!T>eT)(-DWj^w9G2oO!@DbpTAP&*jkW+`+(q=ik6o z)pjGO?Bk6e@%$*dI`@Cx^1zutEUbJdOREkl#u;#*M2?T z;@#j-ak&{pjvUxH)KPp!ccKD+_rlwAhTHDI+P#_=PC!w1 z_TnTPXuM00f(M=ReMiAw(>aIn2uWYr;*=xq#;lQ9bd7=}Mu{Qxf7k5ceY)doOf4Kd zf<(BEa9(>y{Ef1s5c`=t4|s7z6QRgDR(BM8^JN^oBtvs^VQ!a(jLqbKl#qIN6&-tA zKa%lWxAI)3XI)c=rs2J=zv_E2I*g%=oEqUI1BLazi42qR{#5GcMq<fDjhZLM$P9xhC0D{!akJKRduo-$|S}ObIh(UH8)q%W`})v6d{e7L&8)^ngX0 z>X|zljZDIL+K8S*NoZ3{+B7Q^zAyago2KfQ&SjOly+c-l8BKg-t_J(8_9$xH{0>jWOnk@Gn zQu|C&B$jQ1#=IQe;zdH zp*H(}%Cio_xYj;mINu9dM!0TcH+*$&g& zA8E*7CRZk?SJ>j|uG$?A^~*MpEU?>a&{}A68ilogibwIK8*R_9`90b5Kk42+Q>lPW zIZhbe&>7pH$_4R#4GOHr+k}I}DPDo~i>kHZAOQ(jTZgd%ral_j-~RVT;3tpmE&CIH z|AV6WCF1Xp`U1iZdp73cu!c`)+ z#E+<>Ek~Ytw2YJ4hH?atmRL8K2V1kllXbS}4L2DI`un;s51bHHOI}(>H`lfDle(tC zZKJ^^ujb?J7}Omd_(|IIk(iIpF9(o+w5^%9vwdfFR{%VR;pM%)){Zf2k(u$%8rp=X zqlSDS!9%){q^xEZtcl3`;2s^kzL>L>r?2*b`IJweEBMIuOE;Dx_Uaw8osEPz5s;Kc zPGa=y72R3J5;02SQ!>PrV(3AP3cI+9A|yUWjVVcWorr zaK_bLJV`bW^{qRkk@!YLOP~I;QSS`*o!=U)zZlfXxo}t#t0u=!{2%sy9~^b59q89l zT9-%-|42ysdVux?Kd>@6XPB+BOnT?5%$ z)gTAG&yudF%PwMg+lofN&fQ!3OrZuvE^yVvtH+Z&;VC^Ff(l-~d_48wjmS+r>WZm) z%{$DiE{HU9xE3czR*!zZO*A21v^qCet6=6=WF}mEK2(^z{M3+t5^|4Jgfs|5oh(tA zllhzG1wL&1EmYQ7Q32bT7)`YF`1|91qA7PL8s0r6e0m{&kAia1I2-J5GVNc3(ThJw~9U|@O1$h(oEIwvO^%z8QzT~AEfqD9!bm6apGpJ_5t{&pA z2PV(d*nI`|!gd9J#n_`{#ILah%$HbylOpT-h=zR zar$#c9s$^%XYUIc*}mY~ROBtBM9+uSQ?#*_wNQ6WfmbDt|AQxg7zY%eJn~OywLL{Yr0Ec7 z;o32g&PWJ4>WAPl8-Y))VZ0q>1%NN)^M{_IkX?*V?oA&#U|oTz#?^g_V8A)=6mvSYW+lK@6!h zZS(D0K~gt=5AzYab4n5`YQk#c5bO#e{fm$hD&QPKVMf~vMpv5Z?COb-5k>-k?hu8g z?QeOb!t%bCXI_wX#exXZdu>CW+as`>u@3%jph=Wzc2kR2xc2q1m1KOw#EaPZP^6iR zp%hvVzWc^orh>7shX38o3|uXAUht%Tl=XS?KD_UW+B!6IBrEsdxKF0tlNPrhgou?ghtPc)Kd7) z(zkr*oCBq*w!EFOtoHP3(k(f92WHEk%-ib}hCxJ_#kX7vJspsN6Ae$35@iFHFu*=#P@+~(#`_LqtavY2&$ zRUtl1Hz{VR^DLDUzn>rO!UMu*Q+#@15022(Af7G4p3T=~l``Jnxw0yiT_R{FkAmSz zHVfy%@rSgWi>WHs*(qgYFZm-L4MI$<+ozfaV^~P$F3?lW=)Vck0-Z&Me+6TF9{HqH z|D-zK%4aO?#^GLvCkDJ@Bn)3t3}AwP&N`9rs2Ko(P-40&Xe?i)fjK|;&|63d_-D{U z6!%Yev|P0BBOSrNDjEcL{;8YTGR&aD5?+!eYkkW3bZAB)-iNbiWC*rTF93H7P~l&@ z9N5a!v7?`9kp37_jd+5010FU7So$`mS5V09y-|cgH*Q!+{zvR_!|Z~Zz`WppGTNQf z3_J=|h~LCSqw$eBn;+_;YlX^_8=tj|!^~(G1J+;B%RRgsqRBg<8%H4EO$i?<%6wQf zWFVRX8*>>~7M)Zso^&rt^(Ikk2D5#S;VC~Bo)g3FHstjJ(g%kg&9h!F^+&4^%0hxt z^8y?tTX{t^?_!H#WD$Mt3zl<#x8sL&spIpQ({y`w?t^_6306u}wZKE{W@MX+k581R zRtDADUBD~RXJpc;_7R3SFD7;^HHfoJ>Mg2r8dG9S{y;kKJu4dPlT1&w*lTAX5o~f5Zrj~8`R>-!8yhn8XZ~%;3=2P%EbZ!$q?=>m&%ZT zsp-o-p*NZ`2ucvjD{*tK(+$+kZ=PXOmz{8F<0x7Nj%~8FTPhr?!KW#mTBu+=e=b=2 z{eD>B|{{sCsz*J0M2@G^{;k$&p8_px&=iMw57P zSd|+!!El~4RSx0R6#1=8m>{6B?5Z!vt$T7>$s(m?Ef#zK+UmC7S(`Yz>AcAH@uJP3 z<2MdjeX!rBo2So*t!f?d&TT`G{Qx=u(qc7kPGk!3N-Oc`5X)JAR^J{p8I55?Us7tT z7EAI(67I? zq%({~Z-6A<35m5oi~1ze3z__MG$l7*p7S_@*se5fm;ez>3WjMPQrDnD>we~P|IT_) zcBK;fNXiX9aI&{D;XL05c{5G~&xZ#&qhidDz2AtK=YjlxH^9=l7Z+5-5}equ#E&f| zVTkMwcB%>5nmrK9K9}1PD^WgnC-SIe?)x;4!Ns1ce$^x4Ol+3Kz3#Hnx>coB=RIlX z(r!QLr7>rq^ox9ZxkOQqBr-;eWvTil7lYw5L?9-18SLC#Muh#5B2=2bL_K$nA7*su z=sN^P=}p9cWP*Vh`k+I+(4dw-iy(6&{_RF}WD2m@_-lE7+y-eaCP@?rL0Iqz~nT_I?-#hCN&SC{UUQ%tc*FmX; z=uB0ADaKxe8CkyTSMUKpUbU_iQ}4)ofM*JAmaJ1~xfE)ikOj`Mepz+DXa;H(Q{HnM z!dzc(w-Gr1kHr!B8dE0wsHA~VJe~F}s(vS$r@AU!I?WUsL0~E6%&PCf0*`n0)=P=T z|Mb^6&xDU2)qWD5Bf59V}CjcUEEG)Z(twa_BaE zX+a(wt#8HGe@d!Hz6`5gpo)rqy#jLoG6et$q{>y5We@vF*hGe>d#n@f-;eGz27bTc zhAj#H@ol5WW8~%WmZ%e?JU;ed-1?dCnMrQ)&&VcnxbG_xb&UEbbG=f;TP(o(IC$EB zAvBZy)#f?0ZQKLp6Z9d(04oB6zvBFoLaMm$y`k({1>}TygNR-MDFX#uG0sY8y_(3`;`$d z*UEwB^nmK(jz!S}Ed!>lKl(GP{~IiS@~eL3y-G=ck`kpBc9MdmB$VkGz78P%;&RJP zKpAHS=q{#L4T?WaNb5^An$2QdjPIU42Rtikn?4bdlmWuOX3S$Xw1TD|Ubx^0h1&`c`HRE$;Le(6b~Z#>sghgM zYJLWKU91kdN|2h9?`Vbr8dk~A@@cMjX$~3QI)h-i9)e;+jrhV=hPV)PpzW^W7$`OJ z2@}0B>;Ia!$dr21O$u>gTMiBr_vx;nE4X~R*UUUtl7JR?SZ`P1vESu?C51y?2RF4k zbdF;`6!vYuzFNFujluB*gm44>gxf2MIi|!4YU0G?Z!YaOG55t@*Y-CbjXal3uBAh82!J<=8CZeKn=Zdq8ftnw-mBnb57WYj@5uJ;4tv`x zgXXMP13H(X9oT!xFYS1L;terR45%%bjVN4)06``sCFY@18I<+%c$Jax#D^bL5i0yQ z*vi_nraOCovt_HN7q_%WSiEen zp1o3n@$%zm5h0|xU}TfrT$8_7(3;oPvY5ZBh<(N=NZ=L5iTErbkc6AUkSI-s_c==@0Fg z^2j%uZwSJ|xjF)E`CO+`a1B~A4+btgQFlp&0g$cDkyMOUqA$z~L5RB|8+U~JN+J2^ z6}A?S=9ksr%_tW(lx5s4%w@|Xy}$LZ1l7z%LQ>SDvRnCou^TwR&3cz)DNJjtI;gA~ z?cgRTf5hE#-t3=~* zUag4Q&mYV7+G~tf>MHa}(yz`dMw~HHQaEh$WK-tr6hz~SaCm2kLecdX%q*cYHv9`t z`6;0N^5K4e-*kirPfo{?BBi4=Joq2XIn+6|x~=I2&x1fi213d518T{q3rpf`kqnJ` zMEt!+f6&$rW-^}F^^gpxf)725-sh>eXwl|TSeV37a)Y;0t^xg}E=c!{#vHYc!(C7$ z+3#ah`)KjcoDXa@4#9%-;X`p|uulc|d}I6T^V2MUuQ_Q8^}!c+5nUm&giI5wF#}{_ zoY;p3OcQ5s{0~aYZ8}prK{atf!t;BB8cS^<>mRyx7jV|}9=Vuek7c)u%&3@zeOvSG zaQ&T(`k}@4^cGroTO*w&ufz_1=QA!&3Ja1!`&Ao#_mtLXM{`Jb@`Fim`)=J6cpE}J zMn;N%wo`nM@GN@}iwy$^Fw(-8nBOs?@}FC1!a6<80QuqMy`X=zm(dgL>+HPj+c7)KGQ26-+>CjF!AZR{ zO5dol3L>B+L-Ibz_z%5I?$CHFtbv~EF?$GqHeGWMA6ggzJ*5)`N~UK|kOI!Gs!P4) z^7TY5BGPkWm`$e#xhj4gR{XaU*BvDbhy227QGp<~xmA~h{C4cyn8`D}li0=yNbtnW z(moIomFPo*zt?WL-iWLR|2T#IPOo&9NWzB&rK{|+7RU?dO8Y>|IMxYTowRcz-4D-y zd=qnhIzQDj+rsO$gmiF#t;Pybb{Gk-KG zgl@-n)~FGmf}G(j|F+eWCR*(+MpiAZbR1(|81;4vcA5z6mdl7$&;A z727C{#FiXN0_4RkF#t-I4}8+uv>o1Q+4b64|KDO(ipE`R*%%*h6|v1kS&n{ixL5I8 z=L}?ZcXj^O8-oVerSgKG3`vesWSTeKf{ar=>e=hnkmnG{0ll*Vve5$?+7>*2DV$(D z5ZJTWPslI$MjU_^DKfLim2Z+v5+k$GM9m*y6;e5!-X8{fwP~1B(BF-1ox3*K`$BIS z^rzd^7iff-*xQ2;`{xL({@5)PYC4y(w768axpw27;DVjB#RUXBSvl0S+l_AtY5!M= zt;*QIh_cavCLu9fFJ)%PTJM(ckc+vw~YhWf`aU$hq0W)$6&Ly3QvM&oj zCKCufg0g=i?m?1?n#@SatHxMVg8=}?v~ULBxAuIPM3RQ-%YezXPq_b;7W-H#_w9Hd zf3vQ2V=o}hDhPBYvIN^vakKt{&O z8kXA|;Ggzot>`o-Jxo6|I-j4q`mg0chf1)ON`$_65JWA-vw2X_OhPi+S?=6dgaVEs zV6I(`yt$qR5}glYZeKUMJWF*+=atyLcqqaA98|atSs)qTeo-oa@SxTmtCNZfply=p zkb^mbK4iN!AW$G2-1P7@x9NZ{a%(|=4Z9$5g`FHvAP>t$`@Keiq{vmw^SduAoch?8 z@0?*PEf_PT4qdA8Pq>mi$%6wR(O#tc9dIq4UVs{`(C9ylt0f|8;4XrBs}@1bTA-t2=i zk^KES){gAw5}k69^Z>YkYwH4J9jVr*V2)wqQ=Q_o{jFt&MIz8}jPiL;@9@inQZ#@G zN7EJO|F4|-MA=QcWUv2! zt)KfV{7QORZovY&|NmpQ=@r8?!tDsxEH?4Z_vcoUWlBZLveK(}FH1cTXE0uRcD z)}1hA_XdyZeewp!J&A0;oT-M@9h0`nrXoa$f6((kK&O=tH0dn>{6cszQ7rWQ!B;LJ zG@wFRF^QOjL-A>5w#JAw@~tb}H$z{`Hwk}+5lBRL8@q)o3Dj-AftF7B zOmW$V8);uNGvb1INe@lr9Yde%1M`}paNC0_u5*KO4TiC{j14UYvp5^-AG32h-9)~{ zSw9JXurwc~50^<_^XaqxGV$k)3hIiRn<8p|!*l$}9kF9Z!Dj%A4DRLb=S;j7Z zLX?8UjBlwNb6c||W-&6OkGqo>MbFDgbJDSzH=f4Y7dmBQ)z5sIaG4kQP;^7BK!U)< zOeFW`P3ts4hmTdw_(G=ObO2nt*ion^*uPmem1skH&de9^jOnt&Yh&S@276z2^};iU zW9K&o{@1`9y~?;3^t*I)2=!~2SlOV>wIuU4*zZCO=K4vcT`;N_Z=oZowdg_%=zawW!auK} zQpo&tar{C~*o8kQ@vgkg1{39*v#PiH%rVwi_Z2zj{#(p|c}}v@!=dxH)?!_x%&r9m zW7JwgX;rGD)JXGq(RGxq5vv4$#hD5*1nYo7K&UvBay7L|;xgbEe8aj@FnrcRa^nFJ zMoy?=??D{i?Q#?cebdCnm|jtJN=2XP(|0%|ZLoP)5b}MvqpaA9mWVr?LQe?o)~AFm zM={I)MrIm5aZJH}1O0ZV=R3g%Tp8w$|%V++CgIbYCtRoTz4hI&J$>G_oow z@O7a+mD2m;MD9fRDIs`0<(6&H|6-o7)_(s20L}^xP&e4QO2^xfJqM=Pg&y=Kd!i+@ z;Siy_&a_)V?a6(fgKNPMH#`yJ2#jX(}A=+5Q7}fGvP5fHm;!uI_4}C3E07&Y$kx zm)Dt#biy6j{=`RdqRau;VTPeRIemz3@O5dqp*+p(9{CuCD5YZY${0l9gAvFYAl2vz zYoZSC>lTiSe;_xMcZy^)Q&6kQeWI=90pcda3&cQ8YIv?6+uS369SRY?sIWXoz^vQT zcG^{hVqv71E|mK@#b#a~EAkT`6#w?=m${#F+`|uc(e7PrKw-AWGG9GX;P`(gE>AG{ zU~n#|m<%}s&^23QGz-x){JV;2E+9a`=qHC72&4Q#O8u0kG|7f@OXzI z0F-i%mBqWR$0MzOnqTR~bSx(HF;l>o8-^S>^EMJZ#t?LW6cR;7AaWv?hJ}XPJ6WA% zSOeU>8C<|<_kL|WwMxt^2%p3vZOp63&K>T>CzJ+&+OG%RS|(*@%3t_}<&y@G|*>nt*EzKol z&|?;W)d<%OxrO{p{zPv9kQ)i$LKH2O$OugxfxM7^ic3$ecke50O@K``lMJQmTB@&# z>aoA)MQA1PkCgT<7P3MB+m>zw(hQYK5WqYKW%sAi>Uerjr{Vdfp3wC=)tOt_&9%HCx?Kr?>UoO@sWnuR z({^*Mm+$oG=rKjfhQvFj+5LEbzKFUz)J+3s@A(xDr&}C;Ff=b$Hm}%F&LX;wF(3lh zwAkT2p12d9qwB+bp>_iUgk09Ro`1(EiiRUvC&R8|aKHXz@2YDxNlLDWprp7SXUv zY@9(z_RBS2dMxl-{$+n$V~-Wk<+5{AS1>!@?hFP zgDK9}RvOqx*lN$&fOiRKOUN?RA;h>$e9~{rY&|L1dp0s^RRZw&itf%)r<}BROHo5u zSQBOh1u%<)Lk_a&yjgC0{Mh*mSuw}D$9ca^1$$N}7o!&JxVq~yOJ#=} zABP9Vhd9a72`*EBJh@ZhvLoi`5pleZ36^4ExeLQ)4YczL&G zJDAw_ZDCMcYWg6bN}d_4NUiC?Duzy!w~uSnzXP}s+zRgqv{o0XC8FR@!}^_n{{^|7 z3zZ-?>{zX3lr2N~90v>jn`NiZ<{HRe$Vf36>+1_{O-K|z(eIbv(Ne^5dV|`~g5m zg?$B3Bu}#^u8X@au(-RsySux)!vc*vEbh9vyZhp@u(-qG?(WC```){Icke}9)I?5Y zcGholDzdAmb2=+mEaQLOi8}5_GS4<-xB{oBly<=QCW#W5w3P?Y@r{r3HNL%o@=Fg? z|B?p5mEx&2bNEeuCd{7jwhAf_EwMY)22l>2y2s_H!}bq_V!MyrVHjrP9EcghsrcvU zYs@YVwy2I(N(L?PYdb4$ou4nZJTw(gNu(hTpk3;>6}#53Py;PnmxGBRIG(gP`JlUW z#^u0V*&>F}7Rh$pS>y2ohcyU&?UO3Qc180~#b_}bdQd-)bQymL-Sga z;Ea7Gdz_O}w~k^9^JogAG$i|hlI(W# zyowFS0@JeboQ}M}ZPh%lybwj4;5J%PCxOrS!|$^5-}73O2l|GQYKLr=nw`7B#i z9zO*A3cf8q^Zw8|gt7`GME8rX;7lwi|BK8I4zi#w>qUY4(w)W!Mv|zWb+z?pJEXkk z)QZ}@%EUANZ@JQktV$)-fUua@sX{4FN^w}X5LT;yGgRoDHk?@;dVv-I?{ICSk5Zj{}itNhIJea{_ z`D5vLt60bgTm%p&lbcja-7Q6R3B&Vst971M_p`;6Nu&*Nm@$P}o+9jGk|52{2(4J$ z*e}<{q(`D#hLh;r0*RpL5KfnyfoTIW%!&+FStf{730fxDQ5iZVTv_Y|`qmWcu}$Q? z_i;|>P#LK-ZO7F(nQ^SdcqU&LdDN897%1z0=8Z7f`6fKw38(Qa#Nn$_oLw0Fd2wq<#Yi}w6cCJ z;oeR-U(K!$M8#d3VPTs6`yA8#cl9qmYUiOGLy%zeqt6lZ@s;Y4EU>we2pHNpDh11P zTF12eQO%g}WV}+}I(N0h{7y9@khB5%u&&%BI3*|RhA@^LQ~pds%8ZA5Q+BxZINGxj zD>3V^XwTp$VTsx;&&27f4`^VlneqriAQh~wpbyccdgRQ!fruA8^oCAW_~$LGe?tiQ zWQtLXvKA@wtKoRfc6c#T3Dc3>OG8Au#JD+H^f|3#`0=1j(XZ7SFdx-fztq?QGrT)# zPh#XPW1qo#?uEYDD(ox3B3rz#E2yfX^GC>Z#$yn|v%*a1khhf&-A^EVlm&u9!UF9J zGEpV~TKC=`yjb%wdEPydLJ_KDX8O6Og1(D`Tj}x?DZ86Re^qsuT2g8HkWakaw48c2 z?fPn@Nk{ z7P0ekDy78>Y)3fj*gSw4h|?&2*fIkG^)}EKqe{|p=wJ5j2+OMh2Nv93c%Y0af~gnh z3&QDxqC@Rgut`^}0A`XWnn$#q;PvAI6u&|>0HUfQB8ABNvv2g2;xixcoN>#<>HJq_ zYKDR-qvw<%c{vBJbJ$!rX}`oSZJMhvCClf2kCKq|v8x1I+{J(_9NKxWpDY6GEzE?Z zI_K=l)IrWGjVbgE9!o_}Ga9D%WVK`jots@yi`5-tEeD(A@SD`K z78lu$g+hmb$@W)l$4Bq)6nK>i%6qvd6CkPyS16(i3vve{*!Nj$9oS@aA%-X;Y%&vb*ni5M>-^ddZ&u1>GhuDnk07Vu z581jjjM)ZkR<#pY-If+Db>pUHt?y4KO)#19Gml9)Cj#gb5kg0{L^k_<$I9*#!44zg z3SKGET}vaKLtT#|yEUe+rPAu9R@!ComD=4uA}j?@&>pG5D=M82r{97z{q)d=-r8FF zC03B`Vg8Pdt4!p$1=g+~clg8g7S2ORbbKMhDxM> zS#utgB$x{IpAVX+x5RX>|gT8)$-&%~naT zHxw9d^1(WD66%8K^eJp~SF-$l&!)_xo9O%ocQ}0Q-C-hXz6a#eOSDbNyP3?|VT}S< zBIJOY{6W&Q-~%806ZQz z2KSL+!f(yw1tp=H1|bFEX4go?saQY9hRn8k8%Br%lAHIXXxFXASD7y*;Y7Wj3Bt~u zj!%_tS>-MLUsr)-mLU)0aOq~*U2K18a|Hm@B(?b?m`kLs83k8%6uUV&Lstu8*3@Js-yM(>a_qxccECN^9=IMq8(CXhW zd?msClm4K!w6MMxomn7~{1W9q%i4=xIFhhJjj}~Yg&V@soMepc8ZrlBPdy~qpkaWD zu&2!Ys`4gAhh#=M@=dU}I$;Q*YMV8kR#SN72wkD939;`pR$@qlUM<{P6^e~TlTmBu z2Xl*c1r8v$>hKkmKh z-n!LmnlZ5ZzIo&CjVboRKNT>Iz{@( z;K#cc)7LTkmUc_dol&J+xqnM7pkk(d)i@e<H7%PN#EuI_f z#qpG+c3+%ed_4XpSaHaX2Hge>P{mho!dhF+X2U2tk;|g}4*V?oG0>rH*L2fHK0W?< zUsRMlhS!S1`(Uz?$XtAOl0%}I;LDAFe>T#mlSbrGWnj1|1hPkRTDgJ)3d%}d{d2R$G~JoDwnbT)rGRD>4m~)e z>tpu~p2UNZa5+3u1L-+HvMPD}Ue+7Ds<2=!%82OeJ)7$Wf2n%Dc}g(pAf|q-;8!F0 ziCFi9yx(#9)xXKVB77#J?*yjg6%_4eqBX%arZh-wP#F_M==a)DQ!^86w{u9LxESe*xFO;T#h(;gj zPMg5MxW0@Q&^j36g)9&#Fsuw?ll*Xx<;V!TSGksxMpu+kR}>1>QpEhx z<5}foR6m5mBNavG8)LFSNav^uB{FPnDRy*ytZCr&k;CvyKg(7ITyc+{MVW$e9)?c% zsD00Y623JCDNijC+O9vxObf|Y(g4a!`(}jb`_aik^kh;hU`=&gps1G_%htpGr;E-$ zMoV?;GS4}d^)8;5fdnMz?k67iOodQ~EV<0Xg1YG$O@O@VGDg9)>URYvpHvyQdFnpp z4b-cftOTQK4{mt{oIH*WeA_i%*qy(9{(csimF6B=5vl>vC$1oUT4>7}NDXObV})c` zyBXw%+~jYGKw?eyD;YS$iDJr9TWyuQ$I*zAj8DMlPH}gEOA^|>dRb-4hSvE0YLzY3 z0=GlWUWGQah$fy(cDsRt^ug$v|!7J`T6i7awmvgEEaE;kL)+4_m ztEk+V4b@?oFS2hlXp-y0S@ufbX?oRIw0EB-QSFe=_zQv6tmOB(ntbd|_+IKxGYo#srPI-wcsZH#OvvG{wMptPT=|I z!Oq?eZ60y8j`Ai1Fx!gB5k(g^`|{jyFzr!eu1^8CvV#s-*STrev3UD~vN%q`LIZgw zax`Rk<1+8QM)$VWgahU>^(SSLDhpghc5i$$Tt@5?$8+l~ zK%Xu26wZb@-oDAG+bSmwM(8ADQ$YJtvnOz-+D!$fNfReTgvn%SEttdWSpHqHx#Pj) zQR@`*&#ygKr61CA#wPNXclTPGg3$4EMp2pb&Mtlkr0a!Vo?hBFCqD^k+$_;>Ed{W| zm@hi?*=3}(rqB2HQJ@kscRq}!O5fm9fE=C#-lgVIb?2{>GU~0}E41}62dQjSR9N4H zkmwYmzn-*ZKVuu~7dT>^K%bRPrfqyLZ6W%sRmpfw zEi;3!O&)Xbbz`$sG8IM#Cjak%?pL2&s~PKh_o>;D=P>4`%tjG~_}M)O) zS=e6>Y@|!?GM)2IbVv*xj_4|_|Kx0#=?}O^+luyShpG8hcU+a;Wyc)$XC$;LP^$rM zjA)POLPNDZrD=x4&9ky5S2wtM{|tcWsv48j-J0l;Az}b*w z4|VyRMJ@ySffK7ERuk4|;RLT+(4mg+aG^iKkrycZQ5~-w2sqw9!WLW3Rb+CVK+}bh z%oD-ZzJ5hNHAmQPF$CT&`VNRg zQN2!}{O!vs61?I)4hv%WIVrUblN%I$9hDs|=GaBlse%c{9tjAYj4q!`!z-3{(* z>q{v^)Y$j99HCGjOqi9L;o^urzS%!KI`d^90}1DBbI0UL8q>QlZ;(L-6=;$<{o^~x zG&)1FAi}>@qZf!_f(-#VvLI{uzGq$J^4uaSvA11V&2I%L@z>`1v*ONWx}uG@Bc+{v ziuIxp!(J)1j{x$MeH^_TjmY8F$m!!-9bWoEzhiH2@&>>#bg%~!mf%(Hl8P! zuXA7cJlGujL$laHS3_|e2NG-J{tfWd^vEe}bG?^-soap#`EbQg68QPXG7b@i4L!hzx! zrU+xbEfzMRxKJpQ{H`W*qY>+9guOh-fK(V@I*ICB_p&M6O=P&jdt%PyUT*2^c$C(4 zmF)V(q;yXF5wQzkG{e&gYGhfD)aif?$UO|iwOIJW4uZgbLFo2eadgtw`+yTcDjo@% zRflAYwc(1?@J;1q>2#gEi&?<&TVfEDzmX=Ie$RglP4qYLvJwUm-)u7Hf-+JqELF0 z#OGdfVHTI3PaO;uT`6$dOhNRPg)Mp31(xG&|Up8UlGyO9Dnb1@g~g+=6?>W$gMl!d$+P&L=o z;sjiz>l}V6HXGGW92BO2D0SR%`GoAlUNmrYmf_v{39?fMF7+0!aAFb7h(E+FF z!2_eslpL)Bs_Vi z(JClq#X)`06*&&)f`Zkoma7>f6`lx=)L8E zx=Pli*%1j0W5IYXa$E8w{Q}W&>6z&YPW8Zw(^)94^Xz&+f0t#H(#;e87YU0=nn!u7 zwkHzQW07)bZ3@yy0Y`ETuM7edYY!=qRx(w0LIj0A!HVldnd)aG#6$bUx=Z|St6S9Y zu4}ZBa=T#g*=&1p99Xcb43f}6G5yNQ;>2wJl0Y2;^GRDKr;k$^S*TnJBN<~f4KM)V zl%)Nn^Z5df?3;(ZbLV|&DV*FbG;!+J91zjiUUl|#LrLwipFNGGQ(S7_Q;+xbwi-u* zK7JZqJ@E51LY%2TP&ON0qxHL5!3*Uvsf2`gCc7eVQ%YLMH$cq>c{>I=bwCexAaCSo zY_l)ppttd_tP3sU1wkNxp2x(BMa93+be%uqd2vYZa`qfe@O{A z(Ton<-%$_KpDI*h=bFIcd-pVpBm1t!--YM>ty%qcB6)L0bJub%C1vtHgl~_f7^qqu zX^}P%6GS)NFw^Uyoa5#8eGk17Wh(au&56gn5H6(INh1{|;(X$^`qs~VQWdxI*c#R}m zB9DV-d^R*Ws~SvDlL{`z{Q`v7Jco$(>Xh6t@+1J`#Fsav2#EcbW?a7!lD}b6i_LXtFupGRUNr}oR(kq_NHf*f0f*h zGqx36J=u2x4`&SI@~3o&%~RiE{rU@DVtpW9XHIU^ie2usbxDJV&^?e3>oG}K4lNA% zJ$#-GIX6NZUo#W3{#XvcQ4Bfy(>q6(Qd@ zVa|v`I^npl&^`6=&F0R_O>PQ{tB79?9~v`~tKWhbf>n&YPC>-V+w?pC$j zvoDNyZ8jUP`U@L8cJdl~sJ}$s6>hi8qfGMT{exHL)OJr=(Ik2>w?i+~nt&U;n9wW1 zypi;5{~Ym)^h_&o@N+-X5*7rAGhzfI2qVuc__K^kt%wRX>Vqr} zk}%-eH`iWrV4=8yHy>7eSQO-~zRhb#Ytf@=Nq?X?zvLT0yK>{TvmYf;ApdJAa!X%S z&f#HJgR!}z$Vp|LgTTb{%17%#35i77kBnO=#KKHx#2jv9p_laQQ; zHNc?$Z)YtX%wos|G()9MXU#PEgjxox;ggzRqi)aoZ)hIF0Bzs!-=0nGf}M-doImv? zP;yKk&{o#~(c?@OpA}+r#&2o|KIjzOvgYFcoBMSdT`!u~G;tk$p0v)bflcXb>c?+`yx=&${?;lH zQpz7$P4=(}def@mUkr$GT)07Tpt`jhCf!gMx$*^`vn8AbFCkt7iRUyp-mzbbeaf_P z;W4)+@HtP%^-*#=IpNIgOFMTp>tj>KE1or07A^1ZREg-$Ffv~$nYr^DLotH4CbDH%p<7-l42B^ulynemee} z*sK-somdNXoO}fpYV-L?-6$bDYos%|1d;hFfBj+*pV` zLX0z7V8~tcr3^|2n$*@Ydj@VS2az`kT#Ps$|EO1pYg*`MM8RniTOq!cIo=kS-Z`t@ z3Y9{BB7uSnwofpAG+YAj4^w*Y-~;Y#zI*|?o2OketCvI2Z=*d)4shpltcui_HByT<7TfdUdFNRS1{R+#-wEAp%smKXUuvlcq zZkY^o<#HMI#97x3lt^h0D$2jpMTchr(6I@PtU}+C!37J+*vwRBO{Ga{hRoOEjiQ(i z{t_CI0(@rkQW*+RunH#f@Mb!{n&*ytl?aoSAsWB2B++Dy$Ky#8E0@FxD7fvQqheDz z#b&8dLq-#aj2TATT4pnfv$Il9lm$anIR0fNfltye84Z;Wy=@eyg*AavlVv+*T_%F2 zCE!W|BTja)3#7u3ZjSqwXB1E}Yuaxu=9H#O4Fu=2lwwFqK>jS1WHMJ2=!Gm*I%f=3 zRt7aO4N9WYzn10Nol;UA^E&31$QB+Mcf7n?gx;2VSWx~wa^8wov-!Gm#K?5Zr_EDm zdi{KVeSP~k)4}~@s1lDvPbBfY$Yw1Ve^X{rf=>!!N55TM9_((0zny*{qCk8@M&l(S zA1L3HNL>hySV4l4z9KDpbB$3z8(Z*er%G50X)8(3+ZrSkl%UDH3<4ZE4 z+eNT?%A2y#Ik`SUwkK~-7K6Nfu?1gM`ctIF?0mU_?yPi15C2=^^SvpX-d*D_y09r+@;j}=^`;F61^pb)RiHdo>Y3f5A~Sm>gJ29K^;L_>=f_V{C#BfGWD^W&j`sx6a&FVmg# zbwAUmxzsc}b8xWZ{A>mGs_?_tec>zc-4hPa!D=(d5IQtT@9oUK^8H7+U|Q=Ix>Bgr zisI61&EcN~E~b8`_$FAA>_o;9WLJ7X(K2^bovIdp#!5=~xS2GYNnPcS80uMnlNe@}GDqiOdt;a64n2pH@_mxjd(!n#Y*6-HYhoux| zj;wLHQLvjz@vf7V$9!oty?R6ZQ5&C5a$@#VKUDLp&xW*n_L0N*%vWH?wvps}`>+r{ z>mGBkZr=jPY$xAz43#5kD7rHpb$i8B`Z#yrPV0nXY><5IUl%i)mYk{)(bYVQZ~A6B zb9(%np{WJT+UEFETB+n#aXSyre)x2oCrW3!=ac?k^%j3B%po)i$A&QNWceKo+p_L9 zeir+lM=JN#+wPZNS9t=JMeE_7y?K(U8p(kkeY87zj#Xrt#L~Jt`8p=LpA(alFWjEO zR0ATCvuJ5+*bv}O2VX6Qo6K=YmwTNIxe2$YjX7Ym27OCYX`{7I5HDd@OmJmCj_nd- z;d4woSD#l$Cv$#W?l7vlfN$U?C6b~`gCnb!kO>T~+g;jdr2;;5H~5u*TP!9wsh4PA z+r?Xa8`m^u+I-Yf*!#RmVD;@tW(q`l)yZ6x#DcX9P!Npi>UH&Q$DR1}V?_jK5ncA5 zqG-8KZSSjd&T2-O72*e_>5leo&U9+S`A60!aR-fRRriYtFx9AGI1X`u7L3%2F7Z0j zJ(uxfr%qPiI>1nH9L{f9czWEKXyOj}{I50k7?Ze7>sT9c%tAGqPOQwcH9?9xU!cm2 zYHEMnd8J9h7%%ZuI+{kV7~@{)i1GIh+P@vweh?j!UEr5sI#8X0Ix_BTUi;X+Y!gko zt7SF%-+Ujie9Z~;neaa=g2mJN`U~8_L}?%E{_zXr5Cbj{7va{m)nk5{mnv18TFv&k z)7v@JZaPSG(j`ui$v@8Fl#(%)eMom78#_pmm8tMvG=$QAre2TUZC`r9{%DCWHS2-6r*OYF9(-vi`bgB<_a$^cWPs_VXN*yuVx53VUmj%}!V3ngJE) zG;6k6JQ&9?Tbsqi2#XhWsjR8IjfyQoK((DPl)=rd-KLsN^z&*?Td&-Yx}H0FIl{<( zIy#D4{L%BWx|7O~OM%;yrBBPFgN}IPvJl4Yh6FVA-{kst>E#fHON_Jdn%_mRepPVD zxoe0*x@F4%2c_YBP~6(RnZI>*zdwB5t}j8s<<5Kti|3}5V5TcFI??RO0*(KCo> zc$%NT6>|D(DW1Aqcw*58U`{>!FnGPZ#)>>>MTyl46B)e1`W62jf~Dtt@VNiy#USuJ zm<33fU7*biY<7D4xk@T!cRtCcp!z+jurq+0Xrk|=nvG_d8_o_IzdHr6>|NfA>;A5G zNaYm;a*>fXd0*a_qk6LO3A)vl$HbFhX8%GD8VaY%mNyLd4=nQ4Lpv@AY3o2giOLS9 z9}dduM$*(F<4q5~h(4UA?j%YKqpx4dqyyU1RKlnEeF{19GYENQ2_wc8B#AGwdQ78suQY^e|jP2g_II8iqvEwdk5>@{xVqkz(n zGk0&Qji`rP2k{TTi^1>D{5^W%@J`bV)s`zBsWMkWK94*5B7hv%Q|hC~sm}}-;`kL+lv|(b%^u^D6`Mx{=xSCwL6s`TF?6Z_FGb5xGLB@-1~fh%@93^Yt~Sy1ME%%Kd=z=LsCddwdML|I9IpAS zxt(THvW`Rjlg0I2Y4v91is=fHXuXGpUB-m*#$H1$$+?vuy8f-C zVf<)5HD&NF+S=%2MNsZli)(jWn(g%tw{@}E-N*Iy^iCa58H&?9Af);(&_9*YjC9;)rnE>T0`)@dR za9$!!zNu+$O74wY(V3t^mcwFSYrgpH! zS)Ucwi<6~f=K^pkjP(xY(VFq~)grr9nify78x<;rUSoC%m`7_v2qY7o2FtB-Bs}z)@rX#lNX}O z{+xXpOOl4NE0qgx&#N(BYuKnu(nIZaL**+@XxljTlGa^`6OfjIPyw&*!O(=@aO7i8 zPq08V0BXTy$n+ZFt4E^!%c(iwC)3Hko* zTikOk*K*T&3#A_kUbgXMqrAPuhF0B?+CTEe@B04rg$fYim9(E;k_K^}O*o{c*5BI)3c{q`#S#!9Ro??B~=+&?5cA&~G}{#U7Aa z+6O2@E+};n&Da18kQ5U`NdG}CF~QsFEkkUMmOpE2mhw5RFrWXwM}R~%B&2bO?K?qe zW|@nMUf>-Oj5Dxvw7^tyLPzvQy2Qa$^2(S)sOK1A1SDJNiGFLICU9C?%K(_5 znvh{?Yfvve6Eqd$xcL~+LPN#6hPS06_sSe%K`LhPb4)T4%9lmCp12l$w0G=#o$2x< zPwd)y+rL4^Yj+XFzc@j|()M%0TC(U&u}S}?+-E{&E05Q2B8U28(+bTJ8jp_PZja7^ zP>W45ZP*ps0`}Of6o5eN-PGaH@aeL9M7_#iX#cnr;Arjw%&SYRk~PylFotzJ*a*Mu za69>R2=8#u19I`im02I#jIu<7a`F24pfp3x(1KSP6Y_;xW31+KnUJWvQi4FnYi$z% z+wsM}2Sbdtg5FooDtr*N*c$UhYeTFvHVXqFpZ;Lmtz~HViPwx{bLnF8Z;$vqe%*%L zwF>y^W4fH6EJ@)GwW5UnADTHrSxP^m1@cc4Twf8=y4zf5d`v z@P_rE)KBxHpibDLH!Lsh`X!-o+l5~0_nOrEd1_tyIU}KXIHC(I|J~hV`|SStR0_R+ z+9-?H6|lGOZfgt3#=D;|77P04?LX-)?vO+6%#}}&edZs@>aYLuvEE699SPjif7v#P zK=E0?B(wqU6HQBOq(ub*y1%t8gt+emQV^wZ?|%pladM=#rh(m4vM_QnGtw|JQ7|%6 zP{A?CIhcwXxtbACigGhEGJn1pIhg*XlDd!%mJG`Dub>eQD?1#6xRtYuD-kOr+y4$7 zF|l)|=4F6U1DbL_@HoD7y=WXR@v-tGN&12a`TuM~f0K-FE+GjMGlqaSZ4g@IyWLE6 zko_?joy3QI{|Ep2NJ-=><`Ol92p?Q(_5xjbfDM|yH6;#bkt-?Xt|ORStjC6k1Y0{# zbB;uYNZuKlI9S?xI!#OyC=`q_+(J$Tiwee097@JF2B>i*g2W>(%ZT?Y%P{0CCF4_( zb>yK((PuJPCB)LNM7FN2x8fi?%wWq5A)h_U#lZJbJ%x64!O@({zLu1xjiaW?a7pI( z;&>#No{)=`dN}Kqzblfjpx%WYB02ga%gpO)va7}D&N+~QCXvNZyY0~u7NV+GfZ9OV zcWFC-0Q+z!U>4H~Lk6`Ao}N$FjxVe0{Z~}4Z|e#rJN@toeg)&B+T|@<#^sC{7S@dO z8-?;2T^zKRR+FOE)=SV1n*{bi8_+87G@;Ss5vEZCb!jaM;VgtEJDrYQyekWqD3N7! z4T1sAfaX8qdzT4X8U7cPJq=&5>2UHC0p156*cNBxlGX{9BELMAJKzXdL?vmxW)B@r zZh@7JtxPd;Hf4(wBP3E+_>b`rT65rY9i~ws8@C&ugn=ny2yAr<^U~?!b~L#q;)qH ziyMrsgsESO#}LBg@kFUeh3%|r>a`0+)Qx}!uO0{j)!6Ryr&-KG<3{elOhT`dW&g_T zppWM2NxLkNy5w`}@f8yo%P}D}cTy!&OkU@g*Yqi7;@|Cl{tXgXaoQ^Vmzt*$;}7WJu44^#D5wI4pSdV zLl9KCJz^ZEM!*3_-l0cBRKcD2lV{Ng#6A;#20gRgR3Z`#7>`8_*^@3UY})~aq~#-Z zbfzCBoCyuL2!fM7pbq=h)}C)$ZB3a`fg`Q{Tmmhd1z#I9paZ_OY#C_Wrk$@^tqkYy z4LvuSER}@Dffrd^wIesU%1P1yuJ*}`Mmlekiz`X~7E+a8d!!hPzk zj6yvobutbVKCMOMCMPbW($P><9guz!2Av^@hUk_A+XM3)w}t4cWAXlI;kHpm3r+k= zu&91DBd1`s;Sp*mbbbE)gOJT&?7Df1pekD}QA)YN786Q6foI9gRqhKeH9;prJ?1 zx$ZLEthd)kGxYRuwBoja*C)`uf3prK{PD)7kMD>C&*Cx0M;pCXCT}>OS+R(%uDgkI zTf1e5a;cVyCg_wlvuVsp1OY{}P^zn!?5?9_!|Z%aiu#V)r`!vJ^FEVeobC z9rJVmmau!-hn-Vk6ZrEn2uFSoCsLX~onrKx@ zjX(JY_*uL(pR(FpyJp{04g0$GTYoqB_b7TYr%F>Q2N5u~WwDlf+Jz)wP`hr4ALXPl zVTk+3%EGfTZO0+`_#h!P6Da*?TK$YPE=MGGO^yFGf2&YaGu;B*KzpSo1qNYmClf>e zgAC@ncV(?mKSb1OoR-Z7pXWXZD$!$r-w%>|RWX$Ev@gpyD8)Y`z zxXM7bEF2b{>S-C;w~4~_QyKshsy&uAvt(46derA!Qho8R;X(RYV4r?d( z!}l|E`1L~GBbQKs&>mR)K6k*&YM|Ya#dGmT?si;W%9sC04_Z^1{EuBGa@!lE;t4aA{{0o79yrkp|XR6 z>!*;Nh>=JcjzP-a+~Jes_zzB$NQYNUj7e08Q<#NSh?#{&gqc-{m6@GgR9r+M;x2mgPrqs)m00@MK6f=u+f3V!dEi#?BBNzKtMpKnj`*)89SarugJmAXvR ziVT&Q6e((iWNp%GAOQ~#3JNOEkC^bK$mDNEMkd6@Hw_Iac>6B4{9F%A2mF4UTpv2p zR3C`ab~<$ktI$Q!N!sRq)an>JO-XfrF8*9O9u%=#^bfzAGU{;qcqf2iuxMjqOe3Zg zL#mg^C#F@KZMSqE*>OgHn8~fVc;*2PHK|fkRa9!&RMe9d!OM^NBX0Sv5oHC&BsyKYTK_N; zjuZkF4ECRaj!tkZFm}Q0!vCQa%`~~_ymQuF_8*uq^z=gi1;7pnX0QKofy{8aphWgXI08$SUaR|5P=Z7}ehJEOMZZLkA?w>9dLc8_JwG zt_fk)UX{UfF_=_9;qlDY|M`UfM6oMm+E3Tscq;q{=M#p2@V~)i*%ctZDI7FuZ+tKN z)H)3){eQK9e{AkFn+=Q%pfy+cmAi5FP}RGp&5wJ;A;E|g8^85m-1BcF3;wXP?eLg3 zf7djJUdX&=P*YkjcjYjY?Ety$5N2xzqMlq>SY1KN6#Lz0g`@xUOGh`H^I=f@0b-rRj!*`@g)ALn_Khb2DqQk*Zvsk z&ZrKGs<}(n&(L47!nR{sF>m~BnK{i~U?;vB-Hh{(0Qg5Dy<(PT%`$IZ{#!M(j9ta9 zcty4;{r?s`tBTE%!r3JNNf36?@*YfdYM>7^fGeS)>hD|dHwzA*b zwpZNtm$_}9x^2JvX)md}F_yat-1l0nm)agge!%6vz>mqd@)cS~AV>edFWD}xM$WFD W&SvItOdOmXpEo09WMT^9aQ_RRQ}p=& delta 57999 zcmZttWl&wq^F0m|I5-4%heL38cXxMpcMlxg9fAaR5AN>4Ex5b8`#<;I&$sG%s($ZI z?K-n(rhDz4)oZ%j{NX#t;c6oN!P%LUj07k_jaqsRtL*5$kJZx!2(+0`Bud`Y__-4+ zc7tsI?CD}8=kX;`)8#pdx|hYv8<`kQ!57Q8e8l77+gO;Hnm7U*Q&h)bfdIb8{8M z0DLpk80r6&nZ#;)vqK!2f7+JZPFkJKHdBKdLwhTn_C>?1mA;VDN94wie|TGjeC!$W z>-p~~?}4||iLm9>jO9%h1eOSJ{2G9m6MkVDspsmJJ&#O#(-+!_>9 zgyJa98dmT9nF>+R0PPl)gwg}GI65+j?Ez9a6Bw>S1mKB7ff>F%s>g-Kse#i4uKTq5g)cv} zQ;JH5Lr^pL3~|#`%iCSl{BaaQMaJ*+FE7MHfshC^VD`FSR-BXYDNQl>DE z6+%VtsEM-%jp)DuJTS!Q$f-~9iM?|G@ zlyK+S35+Mo$bV!uuUvehCP|)Kn;ZU+s{k;&-}PAT+<1IB3Muris!xY8oAmOVLyd}E z$89ym25)*rTP9C;}hqwUu6TjD_XPOn7RS^9NW#(gevS`i~l`$Wq>p-AY`^;$IP|xG=*oxxBMzNC1vF zq2Li?Vx|D-a4l6-v0b_RsvOgMFrOu+wvmj20d_TIup(8)14m|0Mi|1At66jMbKh#| z{OuBUhQi_(PDJlyHyiMvkC(2FO|{jI1W@}-7!VxpicQu0TY^?Yyz{X1bTg|Iamsv^ z`r}G=yS?h|(AQ=zHFFng$xg?X#i^5q&YG+?1oWFXE`&UszNM*R7$_(-qKq3s$6IXi zv`SgEu-$Q3v`?%)+8c4zgIrx1bFx%rsv+*#v5q0BBt)6p^g`%DFJ)NH803OA84xU| zlGmMozoDjHXh?(7{DP1&6a{Q|M4?fQr zOMMg=PmR8JYujihT8eAb@P9sX5aWPpfGNPLgyV(N*SNigmH#+xPqzQ!{xq9VAG?Ps z)jCZ|NaNL^wg9xhchXG2aDCI#qQYRtI#dmjPBX7iLB}^lU zi{rWFM~aa~UR^aFGZ;LrA7Wh`5_$G2k5j9a7Q6xuKIYG_()K^LH7*b|fVRY8nh_lD z9Lii60kjw3zanHfKBfn!R!FxCA7OrP8)T|Km?Rnbqk(fVCOM^`fwQutUSoZs0!?Vi zIIOWFcfD#HQ4)6|OdTOM=0{E&@ z@cqVS9w!__Gek2eBQ^-gFK#c%5OSn~$x`xzZzYW*DNSbNd|mI^A{H1QnWT{?Oz#-A z7f>i*69ao)F!Tt0HwK9_#YlH4LA`g#P?|ELR3BdgOQ=v;Vfwsoz{5+jpPHSUxL@`& zuP8;dq-QPN@KBzpQpAP9+^O^Dx!WS8`ev>d=ZCGdT;2MTfFHM-vzlrg!=ghW-28dT zLnR`QiYM)P%cVonH7&n=cpB-mCLDr7*+}Wf`}8Y{nUHD*mdhr8{TUUG1DO+EC}YIT zZjPs&Y(%emw|gHH6#~ek!)C@PQPUu2@W8hD(!(_68H^+gXXi3LWW85kyUr>D26L}R z-x4PsUPOX-0I=TzVdPi>u@6XsLj^e%^ucP=rd1UOX2;Q}17SM(`I%<+e?Q+r&YEg8 z#Sl^2*mzctt&CdL7qZu)UqM-ym6%I&JSfpY4 zRKQbSVJCQA+uG~2E=5US19WqLJ8p6pKQhYfzAAy)Ez;;Up><+?Gk5WgHVKN`<*zP( z+gOTRilQL0&oPByfa5{pfhrK@C@)3Cw>|e4?6DiKBMxv(Cy6i%2MG!jXHqcuYtkaq zkAv@EB)+SLBu|{Du&0e|(W*D1#bzg8r-^4%2c1&}wPksm{q7&#;uT{)b39Zfjjbm( z0tk9a{9PtIghhD&LnY(Z(i%U#j5RgA-R8#G1zB8}a6}8} z^I=A+)wGC~8TgJEasBL*Ae;1tR*Xn?Eu7X5d-;0a$PQWn=~?^Gq$QRB8)dc#khVqz zHUV~Ljys9H4<^&uq`+>vy)ELhdV-rMd8)oAawxRU5G$9nWc@O}1w<5JJhvJ94aLvf zp`zCcn&)z-ct!20DqmQa`8hdfRNyX_X_MMDs(=4_FGViKipW!D$R};7h=&XQFA^&i ztZVj@g^%iAcnQ`rK#Xl^fRn_bbnEI{x%H2G zylaEqW6s9Ppz4f`xDW?5EW3G-Ko2)LB}i(HabepH2KP@z=)Z`=te0mF4ZiE7Hvhz1 z*c6fSDLnx)<2c9(Tar_;ezKP0!vtJqkBiWj{7@SBrf%S8(((ZP(RdCVyT>+-O`=;E zw|rgldx!NJbu zinIP_?ljL#l;HZZQBKG7rJo+1XwD%n!dsrx;Yer(8I&5Xn7zSm@jyO6!-Sw}!{HBEf+Kn8>jdh`Z>oLHi+BJB}T zPA+d9A9f(_>K6O;1X_Tc@7p`EVrZ%mfJFa;r#y2v#<>f4P5^ zrku+vJ96i%#+BrRDrJ{YHn-i+Kk^h#8J;WYD_qlPKekPLxoGO7K3`5r5Cd~5v`K$c=LVj|#@ z1rw=M%o$y=ce}jVj;(qnx({yD_Z4K+bB4eq-W7nXOgflP+(Awu*wFyY9CgsJ5a=!- z9Z{k$ll^r78&cIGn~WNMNZR_7e32?-K8Zo&GR$n`Uez%3gJ(puyrLa;Vz%fPh02iT z%N=JP^?cFm zj%rjBQ-b(V(hx(;5FlE!$2s5$P9@=$0u4TL3JBe0$Z+2pW^&PYeLP>QKisqucokW0 zYFnX|bMYrJj)_-77|A516+;kVLf%M3BNk@?Vu%G+^dR$*46Mz7?B$xRdFmI;8|kSP z76{GPJ{Qp8vAlVGPojq%!$D$d6S9kO8c4otKZ_fkEYJqQjfoc|jmT^Ja(%?{qX>IA|!VyzdZ29hI= z-*k@ad+xhhF^SeKBkpHOw)aLa)+5)+9jBaGvdBzPsDR_BjKWIU6{1}QXmcpEl?!K| zpQLYc!?bz)suEp0T1>|Rzrf&>?}AK{q$U*e{hu9gV4sq(_jJz4i5i~3UWx8j_`BF3 z<28|LpGX5YZ9%d+@(dICS1JeG<6bq6D{JlxdS8(vNaiFl#eh?s`LxnGCjX)$`dgw* zEc$kUWNYeggGce}6~6EA!=H(r$R!RvON@lq_kT+E_Tg(N%|9G-iyN`kgb?X zrMesCcwc(5GU0!_vC?ZfipP%@;Ge&u1QJM~47KCrg_o0Pm3G=c_;9UV=fbS;{!qiY z=`kTX<$*JHI0{)#c`HWvNhBIra+)gyF`>ct!{dA?S*)>%G+7Jf1}}?}nl`Kg6-7z4 zKAQL^NX#q@JKS)@DSXD31@7 z9)LANoX%usdG{o6f(7WLJ0iw{5Q#qD85sZrO2H4qmLvbMMM#>gi}1O1lwl`(n~6=Z zDtJ%Q<{9cpxL8p~UuLRWO$|k=`OlzH2TzO-RSflJb?&~L>k~tO9qP) z*OBy}39kSC$?6~~$ZaH&MsQ@<%a;V(R?>rE+unl_poskj-D+!QaL2md{$mKF00(j5 zC~z|Q^l|gBTE&U6>U8q4X&4~4wE806dZh!J-B(_`mBS!R_8TcVKmH~3O<3X2h2d+F z8DHZt7<4T9WD?brJK^ZS^H3%HX&xeez|l|fe!=*~^*E;B;U7GkZA1Y#P)U@JQT{Oh zZ1vo)RS;;>`?~aWG3I4;QUvOhR=aiQNEKKXww=MUu(s?BhN}31`lD&0b0&{~1Ab)7 z*=mio^m^-f%zD~q?-(2~qO|7}qejUi`p1_Joa7EUjx?H~gi<-r+`@$0;nd=yVrS!Y zRFU_w@2>87Wi0-@ukyRp8R6&LCZwFvcag@sL@=2sd-au@@@2P{C~rAM|0>c=cMvyE ziv!+iOqWK@9FFBm{=M8vuQc_RX03E_AW83=(VH0-2K;9Pa8(PO@Ik;+A$05i5!snJ z|BdV{Oiceq_Re&jzp4&)=;bX<0}oyr7a{p|`H?@H-Hh4!j*(!J{%;f$mjuhW#GiKE zU4@cxPR2S5oM)uh0ed!}?@w8lX_aVcy842jPlu4qag0K+a$ihoM^L^5&+h%|YIz1v znvL#MASUKjAbd@C>th6XiBH^8w{|@>R0rV5F;9Ytg7%U_)3YFir{Q5!iiydTP&X1; zfY~~z?)$_0gGdgHGMrmf@=5!?ffEultRu`&l=&ldvi9a_jEtXk-vwx8>@gAgmZ$2n z7+^^1hq)%r^V>3Zzc-;Y`rIlX>D2eWqjUSD)w`-w?hl>x2^=Tut-!6j`Zd06#Mo-s z=f(S-fFyYI8Av=-y_Z`^G6l+!;2h$p5$(tmf<34SWwU-}ccEPRaM3 zUwxRn>6t8^X54YI2sRcpB6Jfc1qPDuqRvgh3=xO%W+vQi;k2TUr^ri-QLI)GAPgX1&4pV-*1> zd3>}F^b}@nCh*}vrx&o;?{1MnIeKQKg>$R_T&r9_HJfJyngNQ$@`l&dW&UV*k0 z<+A`#!wlQ)_#XH8XQ^DeiS7&1_4H^J$oeB8{|3JML88EFzr5m%X}S-3lO}IHt&IUU zZ~l5q#tVu$S-iFg2O4O>WNY%TsgstKhlEnXo@8d89cz6$k2~jDSoJPh>NlQ)GYr;O zSSxtUwsHi;aRezR38>gt3DqjZDFHm<7{#}N-yjYtulj~>&0s?7V7HYBD?rxLGc5^j ziG^^{MX{%C?2-*(r>E^C14+=L&Rr$@EMQe4>Nj_o>i-go)6g#_rX<3c438-HXQr6^Y@Nm&WrXf0mY44ia(ssxopU} z=Q1;Uth`C;i<~$o0J8a{meHa$rn<)5@-RZup)fzQE%9F6^UiD+WF9{A4DB0GX{$$Ao3)h&a z8Zirqx`2%+4w*{)9C?M^{v$Cq73zef6XN}6k-jl1AE{G4d~VjDlD+AY6|@ZdGZ?II zM;bR6+n;lklkP?G|jp(qL8m;`vb9ypsqm$ju9nOXt|W4dZ-xaXTgrr`3_KrHk> zt47jRET&eZg3-2-Ms-?O?~~TeUi6eM+d6k7tGV57j48v2O*k(;zGl1&DJ^<@>XWQF z#8AsRNXQlsaT*@ZMdizMHriXrZmZp69yN7zWp&oTr}XpULqOQ5emYW%_a^T$^m1BL-r zLJKl_Ukr6SL@^W@8J8?h==n912Tg*o43;ArLSIs4-S=hfuuP)9VHOS^L+#BG2SPv@ zMx*tcJe7=vjT4v1?S^h3TzKa8^+s|KkY)z^AT^t~Z+9%h-$35pan z=e0sXJ(Z2j+>YXA%#K55Z49*frEJ*DbfQKxjuhi{y=;UvU;LhMM*~=N~-L!e#EOHkO~ou8(qdP zpj>GC2hpIe>Ubi@?bt8l+jW zbpohTj;|#no7}IyLbytK=F*W8Gej~o1Y#-fd9p?_4;MrPNC0NCg{F=agf5ATEtkS! z$0OY_&^0k4(4p$3IFW=;>xeJ%3CEA9&r74XYQKV!n=g2 za>0Z&wC+L$`7t}4oN8_LQ*A<-OD9yx^e%}!%+&3-E%55p54?ZeIl_33gIB;9$c!{3o?IFNF|bdjzY zX8mV*i}HwO8C7Xk4$Vg8Ylf5WXRL#|R5TCDk9rX>rOSM)Y-uzzo(Q;zqDzI(a7wi+ zBJ<5^m?0x3TjAMMZ}1t+$4{g{Af`pqhV(w;AqlF)rEhM=Unh&2jh$aUF&bK zuPoL&)E=%f(wZLm+1KVX*hCsO=q-}G%hvxw$pxz2?*nA-s+BgJ${LX`xLk7Cmb!Yv zMcs2B8a4ci+l^JAQC#(!$)b4n7VR-HbKxI&B{h*4nc9;&MM}{7-+PSe+)ub{ zUC#QP`35ZsaCa;x6ycb6T zJtKW*jlWk&8|`kkao4>JD8Wk-ZaMAj?Bv^%kzS15B)Gdf}Ar;Cr z>Y@UhFyvG;ZS&NpOe>Z+PbYL|N_Q36ovnyv36gw%As-D_ckuA(M7{06M_hqkgh{QZ z8h<}C085rA=#&@n)f-kuQTz~3$zoqq`gf*DvIDi_E*-hFgDfeb)mZbaCPOr_O8gSm zyTPb-RABz)lF0x6G6Cy6-;2 zw2!i9fE>CGvvwcBj_RJWM6I|n5})%dd#UZvrYvZK<)yzL{cx|g;TT@`WN8OFnV19Z z&*#x}SwJ^%n-+zxa@=yrmbt+O3j*MRri1v@xBd5uY9MZqx?j zAx$f}fh7_63lKnutNcyE!6{kRI+wxxNl@wkvgL0O$*^n0tAs#ifSDHZ;v>xy=U;xL z5<&qt%B%J;Iq~>=rZ^G3-7o~6BG*4-zP%oHbA8fKB*!(qYlu<|(MSMErrfu=4U=`=?5rC))9vnUy!`ov!woL7GNC zO;FWC=hzP`z3yF!uI2Poy`_01iS0UBQj)$(SQ%wet9Z8my)1N0G!m6bGs$)B=btw1 zI}L-j{8p>G9nm82%Q#9{7R+e4n&gI0 zTh$5kwLHm;{;1TicekjXIY4GC)AnM!OTc_!OZA^HZZV6W84o@GLAkZ4IkdYijQLh3 zBu6j8WBlNkKnSVr)Y&PIWx(s8>IETv!@)gGTv&UkS>3U2qO=~k%zCqp!QDBM+Hbu8$vrIwfos7S_o1>l^*%q87dU$Y!caIj{=5-T$vWGPsbT@pp1OGs>7@0`JN6# zD%M%b?50MVk~eZplq#TQG%?KVhP+5Qcd$7OslRq_tg46ey`XUui&Z8EvpM?uPJ&~h zC}VaVUnW%@ulYr(JtUv`UR~oOyjG@r<_Yc6!@a#i-SOALgo{qq`eb91chwkZ$j-XL zQFhf4L4H0;C-FkAd1hik?%GmX1S;O!>ZavsYdr^w93) zddhs$Sd^GpAmAi*b)!SU-QMZCo4?Dv!FJ;cIq|q zbv=f5;xdz@!g(C&_q7$?m-h10RM;QFxtXP$!s)`KNlL-uf&iIfk^?ljxPd?@;riQP zsNu46YmHxc!~0uUo9^G^vW{V^1?q=YR0wkDS9(siJOungmbBDX)Ukep1`(X(3;yb{ zg2D0F&i@FV8iM2BGz1kfq@kQ|Y3yx$@ku%z)|cH}WXlOq290XiIt3Nz6U2 z=hlxuhI!tXwBwpyM$sQ{S2z_W=Bf%hS;;<~H0VmDGwe^obe5BI8&iLf074f@c9VN4 z6JsfS?7j%hMzGMlzGo+6`1E^<_5r)lNS)`rclI+r#~TKpy6Lrlo_JoR ze3Hup!DTg8`|Q_Zuj$k@)|7`}RoN3*Rp3;`?$wNm1Ig!*Z>>Tz)AsVJ^Pf(wLapt( z!=zw#4!=LYDzO~|zN1{vm-?cjPaEBPAFCP4VMIPWGv|V|pzakh$%E)$^GM4}P$B*W zHys2VY)yV%N&dk7BC3+3Jdc%W$n$*}N*>$gHXx|u2}3vq z$*g8oy0OXMi+}9K=$|yTY7Qnu3!S4ac91ASvf$5m{Lhh#P*{>>d9m%&G4L2Tg`;H5 zB*|Fk8>#c}gZ#%{o9(>k2nAS@CIA$i#blD;d|GO531R@xHxzRgsAH-L5fO1Ste3T^ zbETv`ss`D~=^3ZRms1-JyGAR}_yzE>t=?Dtpq<%KTtf`D-JqE9bhPXDH%>^EUGJ@G z37GRbslR7z+{<5d=Y^u~umt zsK|l(QjC)U#6qa{-zX%{CI`BIG%s3oclOgN3p$M#oQA<wxZ$o3XbRUn zdA}RsW2|Sg!0b-JOYK-Y#foLRN4cOc!lQ38jPE-;$-_kC+;9_UTm7;N8Une>nY2mH z4ZgLK2R7MhnD?iCuK1{NAv}ZbHeGB6&A#Wf>qsq{wzZIFa-3hjGrZh-Pn~YT5C`^0 z7hy|=2Sy{tU$La8q&d)j>-2ZQByd;93R3d+d@i~;DVK%Q`)aQDV9w_*HhZ8}mL)c} z_v>-oz<{rsD9Js&y3HJW;m6kqzwgQv6YfaH-kK@8)g>QPBCAxJ!op*qKLWY6SuSD8 z7F9D6SN^D_jNC1O{!$pi65+uazhP19kGJ|N38;7pJQ)ZK+Gg;sg-5(wp4B<*l#8n- zqk6AYIsERM)U>KPorv7lt3C7{F!?ix=C`{--mH3bc$AP_^UMi3*U?SVNF2L9zt86$re&2$7J zpdf_s?h93X#jte;v;#8SHk$hN+FFl2?!V!YA;jHRNp;j^2b6;v&R@tWm*mW$5!%|L zT4W7nfx$AsyM#)ACWIPOBV=T7^a3Gdz<-Mj15@f*O1rGh0TUvUEiZmw#utRiB^JV0 zOkz)8qYVy1hUG{?1Dd-Cas@#^6R1t@<3N*FO3LTXNhdNSp>-0mPz-+*_BDPbV9=@_f zd@ewdv_Gu}ff(D>|L-gRfzcGe^uSY$5A6g5>L1t@?^)Wn$he;aP=7NJ^^PF67!K0aFtmuD&nZf@L)zWoC9L zC&FZj;Qvf~07eCQr75E>K^-f#Dy zqX0H71mdrwbr9&^m7J7~J`&&|Vtx74XO{Up>zNv8G%`NpaQM7G&n7j_nW<2ln>&~N z08MPnNx*EDqzBUGLTDG3<}Bk5wgK!nf)S27$^VOsNmL{7;Ee45Re}8%5v$V2oc~hT z&__QwcL};+R1R=mvo#zFs}ddIV?u5o-c7ir+|GZGD>}3Kqj%|+9P*g_jmFLIBXv*@^Euz?<0#}Wb&v$ z$e>f|Hr`hojUY$Ni0A?UXzs5xphGHs9RN^J0b0fn7>;;Ip6Tpt{!>EG-Sv%kR52xy zwXH1*<_%qjjD#~0HttvC5%;$%h^W6c1Vd&s_lXozZ$k5p1pJZx`h*_lKEEaF$YnAv0Bt2gDS6{wA*}f1}z^W-(yK6@hcd!pN^P9<2Uc zb^X?fBZ?oa6YvL;cpH2gHbpjHG?6g{01i7WL<<4zAfE%F-fHatoWyx$xH9%zshVxD z0r*)G#lxIBFJ>VI?#rq_f-Ks8yuUd>SP2iwHRJByZlU~@m#fE{=yftEZ_f3ysu%i` z-A$5_pof=l$8W|0mt05`#Z6&ekzpN18U}11GG!#q34I?(U;Z9kMhb1IU&}n=4!j_7 zS|@$a+zLQ(t6w&Iw8xF#2XKa~W0TAfm~d0Vxuj4+`>29#{ezoLwT=0gfnv;PU+Lr$ zv1atsPN2$9s>qiv?Cgt%5e2}#r$w^No7)II9$rb{4jb(kKMsz-K^4wAR%RpI`x{C> zjaGD8&i991{j74zgADw8cXcw%07aa%Vl2JL2|^$$ZqYs+W;&+TFW6!HB950}Py3zc zyLTK_)I`~Oid8r5F_!>uc1$MDNQPe5?bCfXgUSBU03yE1Ej#Jo=gkBU2Cs^_x{&nQ zYFk01!C76^HAre~?)&{WE(Rn^$MhSt3^YaT!vz2nSXMC3aVZ!f+EQ&K#VdDN>hZ_; z;6RYqfDg>`$ACcbV*v4!urIT)t?rqFn+ltm!C+Jg^+((WMYNLIhL=YJT1UniSD_C8 zL2mbYTCfOU(_hDr0V}O=m`Zg<@)uV6JBQ}JaCEd}>Cg%k!a*{d?7)7WYCGl7V8(~h zuao)&pTXmtH|y4R0?qyI3pUMLL^?W@Dj=aDfl|n9uG#BzxiUe|jV?|AA^`Z68eI4v zbfDG?ttQgT4Uzi4M-%7^lQOLE3kj%udr%lushZsBL-y<-$t^-VEM{W0bzD}vJ__?) z?QumF#ZxqKCg3Zfro6*Wk1?YP^Z#e!99}d@$%BzwoV|6S+_-&rht+{<&{!|YWb=6t z`(u-B#W2|RGvDU3h{RJ1pHd%+G>SrB9ZN6%b}l19?$&vlYMLCf#oERV;4r7K%E9ty z%8!k*KK^v}O+JzK_Pjtup~uHIy9M^1MQwtxP;&9Ic%f-W>#5tI(vZIwti|L9E*WsptLtDM^0>$>l`mVVT|=3mH_%TL_BqRMd z6hqk8a|xb_X8*yFUB;4;N}f;J>&?@&WHHYK!R#6@gD!_;b;Qj$N|oFBY8|@A~zqFKU{KrVb`_5>8}J>m@{mW zNJh}Vu>D&)z{bk?Z|Ojz*8dfS)o@r93rl@Z;B`BR$Y53Rn60JY8sM5H0UC`XVG!^B z>F$CD&o$G*95nQXmBRJ}-feWPi|tsV4bH6AFX?@9$!y_DNQrVq;zZR26AN42HS)i{ zk;h6ut-ex<1E=y|jm!y{@C#^*91X-YEIV-!nlyr3BY7Z=1dZw!WBGa2POL@fQX}+VT4}#w zM5yG0L@~j(J^o3=O<}+5C3+{2LyJ9Fr6-8NgJ@BV?*M&@MV5Z-5Uxt9^WuMmS1jrz zcF~%Y4P~sCZKJG_amH!wIfO4WXFCMj$z+GHou0ge)An%0IHyF?5CR-|xV{*(CuOn( z*RQceATfL8QHw}#;$&b;w&3oddUl-72N6)i+d1%4F7y7*!Xwzh{MLom+`KX~zmzBN z1tiboH=-mKzwjc?9HEC#q}c?^ZFOWp zE`FuO_)tsb;h@e^T4{ax$gKe<6}r;MuU!^1D60=nq8oqwE#tYQs%xa6c4Qau9RQ6o z&P)kihwKM|yvARQVqQPSDQ4=b`O?+F{$I>gu)fR7sQjJa1$g6v23PX-P+sbjt6tt&`J?JGZ%W2J(rsQzZ$ zobrVY_3FV@+Vdl_WU#Ry_)uh_?v4CnSPCDyU;AaqZmkghuP@Cl`Xhl6j z(I0l-V1c7(46X%$eD-5^dQS)NI+<6$6LgN9!~JwjYJ*J{H_?cO&lnQsme-z-wwV zI~wo8;$t0^Myfh=Ja0J}Z1yZZ2xYSG=^JdMlp+ll_`eXzaiAPFmgU_%M(FwxzwxKN zs-$nCylCnsl!NIPq1cU64Vl`VHdX=qknq=J!A?}=l390S#{W;MjSd# zggN#$=DyW+`Se~l3yQVJVhv9{PN0I4{4DWyJgWrn0ZAWhz!lG5c@0E|-)A!5d=Qe9 zrOvx&YP(gXZfIyb_tqn0x2KQCmHtP(&EjiZ;BP(eWf@1P1`Ydk-{cFtL65~ZDy>%p zbv3{FR!tFpREAjawYY8eQl2En-siT%^W)Rj2;{ljPy!wtdP&SZwZA^2m_x27hxVmP zuGpRet!c-a!ZSbt|D_NQc|*$P(47bt7Ef7crv4**W;#2^I0GYc zJPW38XNJpDx;f9TxeDS2}J}pwgS2zbuchvuxF{2H57#>66 z#L3mF@ZI{pXGr37U2UfNzD=!>h`~Ph7m7z#6!#(?NY`zsN^LV5GSKa{xK=Ernq~ep z^k*6>*OeiC06kafpHC;S?kFn%=Vay3oomUjMEr?7kB}QV^w%Q#BRL8{dU~HT8ME_J~lWi;g!Sq5dJn#IIM7$eHhNU)L095 z;rBIQd-t=7!%OjX7v@9I8P&)SD-5NOQDOTS+H1luu8Ui~kf$Ii3=XJJC$^5sU#y2c zH`sJv{RW)RqPITXnbE6v0Cbz=}D^^JHSNwp`GU+_|dr>7bi>YvU%D>-`=@RYTz07{vFs9v1aVV`Z0L7va6Bp%R3c1D zhwzSt%#H?EI^p|mb|g@&tdKc<1NcW_&@4Y$Q1~fD%bCkqHTQb)czB{_&zY2wFsp6< z$7iscnJ#}eF^R+=S)6Fh63Jq8QFvk$y5m%A`6y=5^zeo~hkP}z*QXlIl^dNoZ=1kzrwv|p zclPY~1T7wwlm~m-JxZP)G@C-k=n~m?FvuO)zhzY&!p4jWWsK6WalG^T**S9A83c)w zOTUgc+>?h?z1Usm8vWQ8cOx9w>FRyr+9wZ{_7{agDsT~L#f-_aK6K#1u<_w3cQObP z$0g{;P(>eTlt=@1WSl-`|ODU8#?QBauvEP^CUGu4b~&bOPTiwMbc0zzI%p*fC` zN(*k8IE(9wfApqg?SHzS51A&bHE|+u3L_VC1{hf&I4OEUpyv~G!u8j_;xc2P@bo)0 zU)cMDvgmc>;it-+2lJg`^C5jT81RO&Y#Q`RnupA6Z1lxP=Fpuq!CcPanX-fxg@O4b z@2EjW&IZQP;$*VS#G!UD>P;9U<|2oKFLB=(Bk2OH^920P@2PN;BvJVWZQk0YrF zcle)#+(U3!qNs^4zjc-VN=yDV0Rvs?5@NUw8hfWuRu0OA0Ok|CqxR>8akC1R_~a2S za(uB38mG-+D4-~AnQT&iRQK!Z184aZX&aVoxZvHTIDA&!6Kkq_-s|sgJJ(cgE=FJ= zI@RM)%=o?e5ahw|@7c~ zwb@vCyVqvJf;~;JK`izt`cBO)JV;tOSY^5#E+IU$0o6fA8pXgu+Q0@H${7s884_v|AQKU@7hcvtcn;`*X=sydgq~g{8x# z{>S?)emBP40n1ebrm&}Xl%#HhR73{I&U;5)2VxD|WO-?ztSY1i-w6Rg4K|Y&{<{&y z4BOrkU_2{04nUO= zf*~^tU$!j%`sE;9SeCK+%N5{ub_visqmAts0!CE8WtOS;Pi}DU}@p zB^=L&OS6qesmbJ^wYC)MP$y`qsGSR=Gg^mmEUCr##XXpq1{d078uGv&ERgjon3lVT zm!#z@Bg5O=mxH1Ru`))0uHCLEyeB;O=lw4m~bgO-IX)%dSXH>lV_; zv5tn6cNma>K1ywImDw3DMp6rBI+Llh0J))fNfouFppp_j0+p7)!Y0R#{hc94S zn-4X5L(nxcaz@Z9pz;`>i`a24kuc_mwLd&8{_5bMyXVG6v1dW2b(jGN^h!W=oKF?u z(d)UGv(XJ|=JR>T(X6}Rm(dirB*%A}qAoYwP9>zALUk7nQ<2u3vv-_YoOXY@K6sxv zKSfH_82;wb?dj#kP2f9%VP5i zF{h}Kf}(ifvQ9L@J3V0=`CW##2<%^2hFDdi!YGwYDcKj#L0W_PL3k)0*M`UUu>M=W zzc0pTuoL1hpSDCCYbC5kWuKs=Wuvvp@tN-o8Z=+JRaEPn6;{p<7BiSq5rmb|JZmUs z;PI#m=`p$I4OR!$n9jslm*pDyS~6iQ;fC?ku}pdz4q3&q?%uWGTpD$!DV1yXKGdd` z6WF#nomZ{eW8`8p zo}mLPQ&6oX!-Uh+uHC}gkuh)Tk8^vSJYL%0&1SNZOlSrg>VQz6@FaR2jOH!~9c-tH=`)tZRcFRXIbGPPrac6$5w-A$oenzv~x(qpgvL2IR*S*=} zVQp~7{ZKn96@ltNyMq;=b5Xf3OwvEs#&*#5ScRuWcr3oqhWB?jEi)cpCEAz1uTUHD z;gjn`^=kg@I4BHeyZX9=i>XuKL>Jn@7ONKZMe6i|ZNl;CyrZ7u!q{k7h;5^xU;>w6 z*IQz+Bf&_E`hZTMTzMkHkmUaZS3s!0e`1@X>H|9}e~%s2>03Lh{J$Ol0R|^VK9k|u z6c;rhFd%PYY6?6&3NK7$ZfA68AU8QSAd_J~D1W7wJ#X7E5QcaE3U;Fhv-~DC6$qLJ z$Pl!RfedK|HDYWcwya8agZkH(6e&CYiY;&=B^_Vs?l~S$7)$^LHv=5UKLg@M3<%&7 zVIhDf3^C5Z^L+65ghAn+-2#kQromflIR}Dbq!oAb8Mq((*wvtd1Cs=tlyaJwfN=pHA>$YgpT@VF zW#DlnFoxIl?3WASYH8|nIAV;!du#lJ!y<3Ood;XLR<$kHwhA}?4ylHVGMHVNa=jRi zsDg#f*KRF=l`T#AseRq`_=vN@eQQ5w4}Y<7f`q0?*b*cZp;U)Ey9Hu5>YHV3VaseTOza1A1jV(+E_usr)F6NVZRaN0xxoa8CoU(#byIxe&+LYFO zISms_#QTNd0H)L$5kXR5rbt9jE|BOAQ%a}FvW_M_O7qT?rP<$oEL;ali^eW(yMOD< z4##NwZa)Tg`aQeT8&L}8#zc0EBbBrd6BIf>&MNc=N1PK`w6N^H}F#SG@FmoT|&Pk+r-y_lJ5Xt=${AJY4S}Th|_t{bvd?)oMPX@;hT-y(lfM^W*N7x?}vu2%keEyyJcfr^g7F% z#x^mn>OHn@#mrB)h)<*C8AVE@vE_;*PIU^~6p7B0^Af$;a^W_ad+9@ulj}}fd zBv{%n6(_C+ZQIFx&pG#ex4x=d`_IH!wZmBZ3+@eN1&U!x03}6 z0}CTFFF;LFRe_m@nHj*$$OcD2A?9LX;%4Ir6gP3R;00(|m;=--oB%AW09IyZ7B~uk zn4^=oi;b1F8-Rb>jOO1#fTq2Pxs8L33qaG+-qFLx%o@P&;o%|d;p)!l;x5Se&muJo z3xJ!o1;EnA-U1+|q^u*OCnY#|11Z( zx$^#lwlH&X_*ZrG0Js0K>`ng1GW{Rxf3hxK|FIb1SXcn&HfC-BQwu8_ARN=bbdv#E zIs!QV6E=5u`futVLazVt15p2?Aq~LX!tx)qyS=@liGu}zTFlYG$=%Ju1)$()Zs7t1 zs5&~B0RMlBGI6l6_x}Il{x3qt&Ey{)M1WTRD98Mtkd3RPjhBVFvW?q6Qd^qX|0B`A z<60K~ITr;Ba~pSu|E#z8M~{C_)Z7th@BM$b{HKh6Z^)#qswpZhNB2Js@b9z)(9F@? z251FPbNfe26BqOU3H&>)Y-01D?)fj7|8^6A<^O;A3MOtYHeLWdX2yTD2lKz4|5^tB z4<#z<=;h16!ovw*U}I$kuy8Q50(jWC{QnnSGj|sk3!vM-1^=g(|N8%KBnt~K3p2RY zB}X&9Fx!llh{^zo;+ZN)x|u0s4Z0;3(B;XVYdut(0c&%kgG;S*`3xuQbi8zudgC)F|QnD(mfZEMBmBQDCD8nlVOM$rklIRDVS)WCeOFdiF_-L>yUL)j3+WoC+Xx1A$ zv%T215Sksyhwm5;yY8)!ZCczwM#?deMI9-M!qAE=PU(QE-5M79%1kP`oA8WpQ~5v zZ|YntsFW5&x^Sb)uz!XFrJOL;M3z+S$NFm5YBrUgf}{(M5`Q~kX(_e9yNz$3KJ4eR zjNcc)s+Oy`J{0&aKm*LtO+dG_7ifP|hfPW08sYC6tBUN-{V_53(NoRnTYeL1ki8iq z_0(+Yz{B<0^cbpb%YiKaeX*>~H|1mbLCVDHHS!aaFDmh^*>Ox5cD*q=7T+O>QI97} zpyOS5cEd50CW7c?D3vAG9T(;~uv101IN(gf(P&rH3MP(auHVyA+2r zxe%adg4RkPiQXqR*wI@J8^X;JW)H6s}54}Ck z`Bd!|lx!nas=_Y*aHi*3-TY*=ftPDPdp(~I;r=wqJm-XDfyz~biAjH~PFZ4ojpBMu zc9vp+d`Kb${r;Vfx{W=8|I-M#gvOiW96E;$MIG1`SqLGjk+@f;FZtvY8)@COXT9Ru5uzl*_AOvc)r}D6=OPD$QZbd+EaJYx6b%^x4Y_gvdw z8G;1YZgBZ4?n{{+V+2EgD%#xXfV~F1lvodzCvAZ!u+^nUjkoeQF}0?5-uJi~Y1>u$9*F-F4lrXcY40Vc2E?xGg8j`fpobwD_ z%wGaS@}n{CxLcYf)WKpSnjJWJ>>u6cXro;H)=Q2cPf35-&s$y%p$I7L?hUoE2BZvq;_7YjeE2Lj{)_|Rf6_78CQP}6CJ?>Ims9M*rI&gA%T_a2!y5kY^g<9|{-z1R6cy4_1w%+^ENDlu!l z2hjsfggTm`u;5231r_)8yLKe$Or;fJ4~(D)kLU_aHb6nn3E z*L@#9US^R&NM`N$?5GSJ8-uH&sneOvA0*9ZYcyt;M?*=Zj@G8M`LPf;Uf z7TkZsj9rWZ?uEDq!}`04U!ayShw58zjqw>+M%`QK5{o~fQf5TH-<50dzoEHyv5D27 z9L@}z3*Uo5kI(N1l+y~T^67FQbNRFfEHb0ooH;dZqf?rYTp75R-S+<`To`=oB=f6( z_Cnz-x%WC{rHP2zKlr-8*(|$QHGo7!fh@BlYw?7Gu3ql~Ct{IQr*taniFiU^t z+#6?zIB-pqDtKb>YddaDh1}{ItVR;TaC3rX+!g`xl*SF*u*>8O8g1BQf9m!T%SLWJ z(XQ(;X9pdk{cK%J!N@JwYu~DlYoRESzR_Xk=gCyte;sfgtqR089+~3Eq$EnALV3U# zhJ!%B^-{XGZo=OtMBOLp=Kke^i=OcXJKxFa4QV-QTbb@jku7 zSS)uGEf)J{2Df5y3{XpadaGsKHmP(+1g8rFkJ3pj#}uKWGqbn0^fA=98di9P6&n6s zt!#lbp2r&5xGA7`NsW&Sgp7aWHGsllVMoQLU?|Q{rla4ThX0Y!>0RhgryFj6MtwK` zkJ(w@-fp1*q*l^&$RrY640VV z^KqxZz~vbf{T2ukqIChTaY%a+PB}0_?;WtRdP2*dOCD=V#bJizcr-cxfypO2ScF+?+g^@8<+?c-_)g$&V$6l38*RFO8wp89 z*?f<|UF+=bsKT7|ix%{)W2TZMj-yYVHR2Ukzg$~q_Yr?NmRE5|9F~P!MDttAk9O-) z*&D0jYKm0+v&!q>tpQdzSSz{tPDM8BJ}kLQsmd%V9+w}tIq&p&DajUkzTnc(n2H8ABNmRr6oISJBv z{~|&{YbJjIacGGk%8p+o`bF+NQZPo$YSZV)K%T)_bB>lX9%c7WhbeghhZlS^_S9L2 zWhF4!zlYju68y-{IzIz(EhjI7eW=@ca=lEi;jMM?IQmRP8L(J~_nB=EzZj^y!NzY3 z-G|sm5~e$PSeev1Z}BR*VRp^1kXmUS!MHww*S=uxxRp&4(D3kngCiv;Z=QTSy7Z6P=MP4_Hr_-A7@G zv$b1Am8g>g{3B8v)g-WISw&<(V(1`-WH)mT8YlbFbg3@RUvB@(j;yhvyu-y_uxSBj?61AC(ztw?|L3cN}A^e52h-V$#faS_S@bwYh6keDuV z$TqqTR`j{5TBIncEllCc5Tu0fDqd$3m8augMho%Ou-gm#v-#BX3`0g4e1u~Wf?Adb zt|&qn8D>bPsq5s|wSwiE404;tKyt-vy5^3E(|GTeR)0L{+l%lT2IDs_<|~oQiJE`g zjuN6DHPu2Y*{tL|-}U@S7o`o=@pB@-nDQK1;_bcWUHwFVhey5Z!Sh#oWzT>$AN(%w z>@kgse(lB+c5RQAoaT7fBef+_O=gJOp;ghNprE0yf_XZ1{qJfJn^LTh=_i(r8Q zbXHe$_sa42zvAbXBXi<{AW#SZLK=Q=O-&MkL-y-ja z2uJ^lc|cYTRx*pcFOrYo^x%KD5Kc4bb2r^7D6Hq$Jn5K5yfAKrL!8q|G$#2<6k&3| zvuSC_M9SNnvVHsDW?{JSBUkeHiy00+*VlLtpV1(@UPFdc0G1l_rW0WGuY18y!g}#h zM!zwK#c>x);=ZP-X0EXAi`yt=O{QSDiozDy8`h?o;q<;FCptT0cA|gv{T*5Sr_KH= z?7QQ>W3DAv zwwZm;TOpRolI&3f88+5k^tKW%C!T>dD+8xw1P6zxqXu#&mqeB2Q}sSwJnFCINRLu67BE_#6IG4=p+M#G_QQoeTk*YFLJ%*^M-Xo#t^|^?M)@ujh^Mlo zF=GfSdFAY&`AvU?L9V&-mqoeXM;BB|F*00<|0wJGbJ)bltvY%c>KdAgSLKL4l@1X@ z6{qIq+1`2QSj50(PE`X{lqozB^objQ^;Q2i02jjQdM)FCLKjgyHoQLvJ)uRtq!-?V zcKi*prRy%tmy8p61^Vt0x5lA>8?v;2H-f-#4?MinG!cL63!B7TMIRl?bHR0)&irGU zj>%eaUa+voSD(z`|7Ng;b*YWt*?)zp z$}<1wv~2YSYj9^fC|MeoX*!`FIuvG&7!=jpkk-7;W@kLsJmMt;=SB%z(24;Z}}g?fVIcw1Q*(0uxP`l zajdZK-N`NcqqfsS-Bev!jgmO28;e#bR?br4%&T$CZy$&Wu#kC}UawfVfLp26ukszh zxg>vkWm?S+DdiBO4Qb`HL`@E}G__V}|CGvnsOHuhsL;hzd<)LAR7bEGf-m z!y0)-L@AIZAy?23U~ z?fv7$fsX`qX`w9^vo;(CYfGuC5Vcs42lsz0Cfl8`CT(#QEK?Pr2z9|)&JE@w^#TR` z!oR&$lO!ggt4>)$cJhNysyC`L30gT|^YRjgx15zSDf|f}*{@_dNf!1mF$Qe5on6w_ zXCHbleGuPo@4>;~rH>Q1z6jTcho_;eWiH>!BYS2BfyYPDB?@6w7;+3uZ9`_oUp#-Y z%s5|mN?gc+CPVK9TnH`y%$sbN{Zeq{GH-f&70;{Z`%k4i9b$If6d^f+yp7j-+8&gp zw`=SmxgFYnB)*u(pJ2<{?xWpYCP4cb?rc{~l9cOn6%P_IvVstr{+^{$IP~ReMbvOO zjPo_h=Za)G70gJzo5u2W{bl`>S*(9X_rnM_&ZZcnifCwE$Y!-RuA}9ktUo$x#mof{ z!=$8TjR-~@czBZ;=wFu(;g5-`a-z<+q)KtrZN_z0=!?Tv;Jq0YCS*_WjQqw$Z3T zK}B4gRo;x&<`|}pOT3_IN8DsgQ~F$bo3r!j=EbcNzJc%FatU{dR0&*_UOMdf-83X2 zE#~S}j(lk;-|I&jRNy`7(BP5f5E?Y=$iGT4U^s;EnEsSePcjxX`+0w&a>T+UuINAi z`U-a_fMs;_HR-@O#dK+klf>8a7SQq(Dt%ey=27x!>F)*Mm`;=?Bd6Yf?JLl-LIYQ$ z`B1M?uI}KE9G7$vl3D&1$nI^bCJTp1j^0>4`dSIuB-Px;x#YIeL3fPMj4W~4Rw3`pJGMsayx@?_q+e})>g1v&ym#9s#dtT!s4yLZT zQ{Y>mZXjV4Gb9bE*3IH;PnCx;EnO$rr|FiSt%Zr%%Ns$1wcCH=#j*)#XdI`Tp&E6t zdBa&x*UODF5|v1u7}^sb!cVUh&%`~YY3FT6Xz$h5nH=s7ej21VE48r!y*{c#3t)y; zxS@xT1a11g{IKQ69*|Y63TJ~DYy{ftl&5?;7~Oe0{u0kZVdSzGB5$lKT%>H6$5Mkp zb6dW>90k`#)7F3FQVyom=((_x*)t@e+`_M}(J;@(^ zE1r;PnN=aFddw!)rZ-nbK|hDe{p12ra`J~|thvtziZL`wt{;WYM7{awm$koseazrv zVl0rUl#|(frlTr2JLjN=@6y>DK!(+JUhvCC0=tG}`D}j+x^IIW`}ImH00r1@c1h{h z>#@c=uinN3er%J5ecZ2(7)-;|Wvvv?<~7SMYsh+fF8Ej~3%SQXd(NBxR%5(pce3@T zsPify`bC~EFmAUu>c^`E?^URw%LW~(8%;Dtex5hp7x!uOWP(L$ZOI>3!y+Yb&_lA6 zRnn34bh>|$fNdwJGdDm(6ym~bN5m+seIMDQi22DlrWn|VY1Q{fW2yms!%Yf{C`Ok|RP?@$2Z)h0y%b!FrtzML( zy;qZ<f4bO>lOPx;PQVl1HIMuZ6uT+UXy=+V1eOOdJ1YWt{a|>AHj`;~ zg-d2Dn3Tojza|?8KF2wl<*S$nvB8O068>Qz^=T0z&TJOAON5|6|5&9i#UTCzWc_P7d`qeWT=Yk^w|<`@e&b^7ag)( zRLp-O7M7(mduF0?yL4z~q%D$JG1LYhg6f$V*#%D4XfUgPrvY`6%cVO4jMP+5an+-@ zE=}VKW`s&@=p@Rfnf|q_X#U|Bwa=-CT6wLU@Z$`Ea!!)Aw zsM2IiO+A(K9-KOo)#)%6XsR5iIQWn7`-gv-io0N&&ZZl(b*5(ehvk1DKvr5Y&>ju} zRKKu4Bj@k$uMH@%w4BEdXazhH>($o_6vWhb7sxj2)dkR(nowS@j9;RgprB6ELaZ!!%wGHL`Sh^sITwWeo?r2O~36Ymnw`DN1*(r@9_s%kCf@0)+% zvnlbD`-9XtBX<0vbjQQijVo?w+3EKC4M9p&>Q0FaUj^4W`X%?;+=ucno0c%}t4J~C zZYfMzS-I>bbw@uQmc!jIYrXMA=dem#e1am<$*rjIAE$Cx%p_4iZ&U@RpP+mt+8VRS z6>ioeEFI+nDx3`AFG*)G&zl!I^{an(QI>%*Zk__iPZ}WJY4XwpoEc|mca}2D5lM&q zT{6jThafgjl8VKZPgBE#vz*>6Vk5#adQBcP@bvQ;mE`oPH2o6Jgm5K5MQ+-P zq=1OAxdd(jM#*Fe@IUZg47YzE*7wr%I!rlob$#hJ&XHu_?*JLhxs=(w?}g*SdLL_> zj6?d1qG^LWe;?(m=g8yqpXoj*&C=Yg(-CH13twVlp#32(o0LS2A*j!k)qAO~(RR`! zk&T5&t+2UCi*6_5_>_{GqquJ)Sz6M=B&@upQ^m7vh2-LPW$R_zi^P9B)+FfyLD%NE zEqSkbJ8`Vj&`6N>xn2aOcyciZKQ?hJ4D}X>mK75Iiap_C5=JFuIxXBA-~7`u+J2J7a5yEBSA{ZZU1xTy`|Y#WJIe_r>c+!^EirCOCo1FQEqzJ0&LD4L6-P^o zvrK97$LkQ=ltRS+BybGq+vnJKl3g`Sr6LedHx6X$ifrfM@8XYs7Xr6v5B5!*V=D~t z%7_q_0S-vqcnLTkO&FbuoTtSIQF@Xr&8>FeYa!Srod2lH;E=bO@Aen?^Kj*#1)nS# zcA!)F<>8sUeyxARxR{_v!8usE@>@ptX{><-uZmIqoC$Ic&;HihjM)+#8HWuGTQl%6 z*g>-L0d+e6f$fnEQ>EZw(vjLCTaj0rKMnVnE$*i8FD^WP@{lw|f}Kvl(6l~ap~FUhT25m$-?N}etayMpt(QRGiY3vI6xO06T zw?3Zgf&z3df!@b1^P9W{@GM6BZ`ZruW>WLZir6@@loCr4OB4tpKI-1oZ^y5gcG+2j zTtG1FZ`+VRGwR%3B7AkHnUV9=d?h#vZyQ19;ib-@qO=)axEnFQ#@zyE=<|WIZ~xw>$x9+|oV(kj+ZG@44y93y(#smA zTV-{9Of-TrAS^3QYD^H zmIPXM->@C}V7)j!DlKocUNlyv;1M6eA?7YqDK;lPi6%&14tC{N=YlKGiOZ?bCxkHH5gXlV?kWP7HkiT9p#FC8?ysQKkot@ut>Ump@g-&B(s z+36?UIfyOF<*_(MeGXWjlV$6iz(E=V+ej_nVtKx1>}KZ&RJ`B*izjX5aZ7 z|Frq%O${F?5h>ORx-^NiYQUSp$jE<@AJqb{Y<2d9opj~kVg$uk)H2fXXT3NIc`?yp zI0z964^LYfqOt6H@hvw8engVsR!fSvk{{C{;!VcN#u^a2$7G&!*BaP-EiWKd`$y1% zZds=@HG7o|Kt@bDxb>vq?>y_D9j#VMQ^S*-%;yEDL`DnQxVBLMve&v}$!mWZu-4ko zg(UNrXuPT3vz70OT>dlMcAg3>CI}vnRt+g3L7WbcW~ZnLzK^%?(=buNL3X$N0=A8p zWix$-dXi)e{$-zxuabgqHz489ZvbjfB!JZ0y)K-(G2DDWkH2^~S)vN14+{RGu-8Wm z1qG@+u+XGnlx;)SGQWn2rnrC7LRc3Paz7{8Z$l(~{1O3D???sP??JPm=Hj$`$gNK9 ztN}kMl+lJCz5{nj*a`!jyE~6~{frLwLHThOuB|5$_nB)zq`!|x;m-`;*Gp0&uCp4Y z3VikAEsFa!c5Wf?KPxrZ$K;oU#o(MS*5(z-uL?69&#e9F=icP>-NJvE7sah7f%h<0 zW1t2+UX|47ig}yq=kIccg{RwD&_x|wt0b0##N_Ytj|3#_FR0R%iw3eL_LXx(ha{#6 zSHYQ+w(m;YYsGZ)eU(0;JG}{9=_zWM>2JN&YXuV9JTbGwGJ66DomN)kG!W&kwSghx z8tM+LZp}mzJwxFuOQwHr31%W;?fB;(9AWGeMIbrY}w0 z6n18g=v3BJKW`0T8oi<@ZcGWvj>eB?ClIze)uR_pwP!gpjSX(CRNw1kyQ5$wUXkGy z9NEn8otFBCXzobF9{1X>3?Y;$Vww87zRFg3iS+V~mZc~T zZj9nGes=CDk*=t1d-E9zQ0V!TrtX)Tw#O!BKW3D}yUBsxlv=cPl7Q`G*Yl>d%-Eo3 z;}nr1dA2B?p{9QeqWwV&!rC8O7Bq5esGasF-hL53Er(tIAN%X_QP4>%(u}zouAv}8 z1P~?SZ?c!f^^)UhUk|HQa~%+Fmc_jA+fH_lC1y7gfpjI=Ec&p%s6Xf4PmpFhZrm-| zKZa>Sc;;_-xA;(tO21x1s_}CUds#pF88)#6Ti=Xr{fx)r@SLTkbeCFLk*Y%1Y z#M0a$+xbY^e&%TSBrjNW_~0Lj01K+F4Ccp^J)KQ3HMc?ZPL=kdhlqBgcx6?(CS7$v zfw;s&#V|{*>9T^1pt?2nsoC*kD4j)FR;&*f~R3u^Yiq; z%ZJUAdahJj&<%3Utlmfh5419+#T-{P8?ms>FfhDsrnXRSNavF`1 zJ>WAxGuV+sv_VJ{zL#5^IVl}MAUi2~#i6tEpk~*L@5MA!y|Vh3%)}$59%FItW<}Jv_uoZuP&VgoWTS^FCG`zC38^K*THD*Ue#(co& z!&k#Hr~L)R!VTQnW#C@Gr+(doNhKJzj34UXU6U?!k{ZVG>o@^cW2*g`TkWzH+}jMX z=!;`8)1_V*P*%<9Q2$uHD7S}wCVPLjk5T$miUbRT-X_OgdM+B2$fK-IuwG_%J5F<^ zWZu=$YKAZ{xnW(}nes9d4fW_X<@MivtEQxm!g-3=q%z&ikHneGMEPQznO+i&G>cMM zrhUAW+L@vsyy~IEMq^1|20Uw)i5P^k$%W*EHjWCO>iaD1aGnrnRr}Dil_P(}DXiJB z#oxFwwyDsOglkwXBf$1xwFclOQ7z~H!R&>$B+^BA2F*@~+S}#}zsplj4-=>9<};$2 zFvI*S9+8^b1Qsh|oRJDHkMoFVl@-5eyDHg)XEi^c6CVuQ8AjRuXi(|egxW5#c@UJp z(y@RG`Dbn4Esv#dcX~2!Fl~Q9GP?pwD8PUl%mZwKg07l_r1>`A)WOznhmCj)?dJC) zS&w}WX-++uh=+z8K)s@9Ch;^4 zS5eaC!t+DF)cfV1IPI{`3=OYQZ`&JGpUNr&{60y-*3cA_vR%M~_#^#VUA4jStpKHi zPUa@s*8LjsV!>bfyU?rn1{=ObZ@d@z@KVqz)wD`G;6VvRSr) zk(Q^1kM&Jv&<_Gg_0)mqUrBnP63E1HD~kg~g3K_o1yATD66I?V{As9XK7pe@1)*ek zC_*=#h;L@T{oz5oqC39thfY4n*>!&Rx$)R%{l)ns#aahJ(bj)9X^5Z6&-l(%ZVk_6 zz6{nwIGeSjD5urRDM*cn!?@;&0l)VC^}p|$M_+o(JGse*TW~(=tOPmDPza*+?g|Sf zVBqGSG>pzC`)c#B3u5sas@b`7(fBe`r67ffT1W$Uh?%8|%qmy45a$@Q=fe4YH~9V@ zSCiNBA77~X3s!%$eVb(?k-uFGqV&|TR_THzNaY@iZCwO!8CaudB)C9%_@d z-m7H%i&Sg6o6RJ?B~x<+=kW&8tGsDeva>-tT*}B~xDA&kAOG;$e!N+um1i4>bi!JzcvTzpZ z3p*{F;A#(|S;pgC!#Fz|%li(7*y+-nXTxLx+ykK#Gnz{z@3yTHx#mAy+G;fV-PfjP zTyB3gb*rZd8$Dc+v)`e+@khGn!q1Kj^G&WMCTr>uxfZDKG-$TJ0v{D%cHSX(OQJX;a%T{bXGa>F(NSw9Pg&OTwm+3`td=m|Y(d z7U2;h{MWn6;XWZ>Os4+x@0zOtTM*T-wElmPT@R68fwH8WVY+Jdz)OY|y4-x;x;rc! znEQ(VJtMS}^?Yrlj;MfAj~5itQV`9y54T>KECmbm(Jwm-u*1yyHQm2^wapK}N)1 zxr$=$J9gvAr?=jf7nLd6|?%NOstOK$GKwHjPifWgsk`y z{({1V$Qnm22?tCsR@R{o?pasgA~&%+!X3#h6u3N;htmr=P^CDY$a7JOq+SM8etW-7 z>eq0as)U>|Pvoh@6zd;zY6l1W2G$yjuWVAuMqi7rixBBXFG7h3vew+f*XaUbibS|C zhv_DnPjyKXx0kO=7v1*TH+RK{f>C7cAVDX9 zJZLr>k&=dC+L^#PSo}t_zh6`x z%yZ6um>jpV1aqm!7+5HiJ+kfjF1loyrz#e#4&BK%4>1@fr|Ojrj2 z_V2%18`y@O$D%>1eP)@Dq+d5ePy&kvkZvrfrY8_aam1k?)-`|p2xTk=E0}kERL(A3 zOAGBZg=sl=AEACDZgV^xLXT#kW#1iw#!fe9?sq9|Gr770QQ$kSBzWWf}Ry*b;T&%v4TF7f~O zmrP@2dU;3U`rm&GkzTNl!?g-#FuV?zieO4*%qjs}mK|2|!|yfZcE;WDKMqr5Zv1e5 zDm|dz5f_A5>F|e@gZ(OW7*=Bnyj(Jtgf1j!N&R_1m?FaGHW>dmI|*^x_BDk4czK^5 zgsQ48%2yuLB3_Z2)Gg-hAfCa-o%TyK&~~wzTz7=^VR(OK=P7G~&NQ2)NaNR!mv zVggrRo9CtBL1dirjjj2!#4681b)h?UEwl;uoFX-1|OZg#`*uCjFXY*I> z1!~q`^d^6D%2%ZWy&|egxJ8N^tG?v^$NofvvGvIz1cWGYh={>l18^#Dcd5^%{>z=% zjZru8#|3-){ctx~8ysV>a6xCGlT6nsspifUjMSL`<%-|aLd|YU833cdbbJGLaLVl@ z$oncNY+1~wvn%LMf>88yceoet-zr3Sddf16o{WFj<#I|q`cbc#zaoE3qIfhh#?!Y& zbpcOzu^GeC^mGt>ap#>I{|s$S2lC*UZ)XcTl(`|rOxfAGmi&5gjum8{hEDY18uff~ zQ&E*y_JCDw{C=Y!T>FA~X-N=Q`&7T7Tna};h33cX!l^6GDS|kWav@LHA41hj3le$R zc&C4#Q+;-ZH5;4hkt-nJhP9z{G~M13eY^~ZIb!kEJ^D|6XLdYHRk0I|J9_e45j&6J4pB>y z!F{Y9gmQTGT5u3&hy2J7>%GJG+sjGlot#|}upf<1hX!b%j*+fWBhups-6~)h=~GXl zG|Uxg{GnDLP|?P@m5V4MR<(4YXT-s1g8i!~h+U>TE0l4;vmpz?o4 zd!xJYpik41ZITHeWjr}tmCt%?DM;8C>VYd4b3NNv>+u?O^c}H;w>10|1uWtyfucJiwCr3_jJ1w z)H4Vqzm`6Vq9HRpar&Z&6vI?1A0U6d~C3*sN zL>9+WTD=ozMl#NHJYOVy5d4aY}7!pr>=iNiOk+l<$AE`GQfp zNf>sLV|MfpjC^b)wiYVLogH7uP97z(=h-Q35cU>-!9)&_Ekww%LFrq!K#T=sl|DAM z`mNS*{h})ELE6N;vmnisiUnIvR>_t?W4!k>sd;g>cAXz2@m;d)I(8VE)X$gfjlqo< z)nxy6MSZi;{j9GxU23wY8qR;W9kt_vQv(S59ZbboGV3UT$NMc#abz7y{lRg&UvVa~ zt|}dl^B!UZ1O~Czhoe;x4>$Ac2hxfhID+mvl%mG;^sYG`Ij-cu0|;I8ne3gNj)tD~5Y=<~-LU11UQgahXTQ(v zH;1-iKpvf2wUr(ule19z4DI0WBRGJUl`XrA=)W zDHq<6wo7^p12!O}Yl4x+&kPWW(Any&So9nllyqU-mXC)UM_c*r1|J|vc&)WKpBx0z z&x4kLWp0DTaB9mZR}DsO_Naqi-;n8V>k^vh_PAc{kf$K?(MEsxHBL3O*&+5zFQj6v zCcRU`nSnA1P=DjLYuCQ~iv9ASYk@5G%_h7r6`fQXvOJJ8s}RAJERCdN25D5=5B7pD zJjH6?ks*z!@hthX-6yf~(lucv3pV7PpX$(I{Xmk9*C`zDWR^(&TcA1s(_JV4GRxsU zMLvVtg&yKIi+_LDwVXV1xr^rGL^Xc=EJ`y813c;z9Y17J1`op$?vt=F(~AnGE&lsh z)l~9wX}zE9DOA?sY7J~EZQ9)W3lk4RIr>~+)b=+xIQK0RJG8SWFX@;fA%h;2CQq_> z&NRtYX_H3iuYi26pFE|y?v;Ihj>U^!cHw{YlA^J%Sx#%JHrxA00mszk z3)3OAfbUuvtK6b<-hTSPdJ1fWM~h*ooP{l)Nu3pJsih9fP+$q%{O2_>&o%N;4(yvi zw;=o8dKOmupy~x_&3(VGB{#h`N)z4cG(Pk0QOAR&#!37pnujHVkTpYt|6o=%7{q-| z^WGvc)7*bA%6Z#Exu;vkxGw&j4-XLnh-4O0>HUod(pecn&{E7Ru2)G%y=~11I&9+so z78A6K0i@}4e23luMnJj0{;sIb;gZE|{APk;`?ag#TJCgFSwSdU#Dv&mFE0a`0jacz zGpIb#^d6L+>h@W?DBFw2TG1~xq&?M-?Ryu{P=pJQ3yX)DLNS&q@;IsO zu)0j)h1J1FdbjvU+70kd*_aAu7f>u7Q(146@T#%ue$n8Ks02Ks(wO7Ni+jd6V_+nqpa2K0>Eek?a z9QYEU@RYCdh(nR2lkk(!U0Cr@<>33yWs!W(M)VwZ|84u)Uzqu=#nY!?gtHlgw;(cu zv9VHzMTi6sxgPzIR7^;-4Z=5h%JrdoyqX|_@9MJZ2- zS2wi?=7ru^p?$-CLswNqUks|WJ=JbjjUPnCTd=SrrJ4$oq=bn>tPryRh0XTmU9$_f zQtySo?}~@5Cy_?MR|OMT;o}7EsLG1}yd5pUVO-Y#Dy04NQ6@p#Z>%D*0Su;^7i2V<{#CVk zKGWPp5LXaUCAIBo3mdEew&R9K_2jd43e)S*(RQ4JHkh?{+epw5@PblWlpXAUlMx8A zZ1R_tc80hetHC!03LPDrb5=5{XaBsgQn`K4X2q7(ZbCqt%JV}m@7T8lUk2Z&^(Hfg z#f~SmLK6%`v9p|0@*2EGDG^Up7?=R*uRnE73e^d$&@`{?tbvI*3sRMj$+OkPnvZeS z&Wk=&_QLkxLsJ8&N5g^w-(O9C?W3p)p>mWiKuGtf#`O_+bSq~7_ywtn)QnZe;34_X zOXV09F&x_G#qqz6tCab}>^v+G`o=Q~Drx>dw8RAm=$;g@yv9!By^(VhQfw#4QOZKC zz)}NC(P{XF{o0of#;RW=g5ffwENOoXh2+acn~@cpchw7UH*?Rw0o?|F$+sk@ukt1gqu^~q@t&T)=jg8&N2s2LKl+Ev^m#jwYy%g=d#X6@B2B#T69w{`&p zA%?s3oRX&-IU1xMWe|>@OIxBuVkxmugkTLBS(nNCDSHwO+67gNoh{^&Cd5f0uh_(136{ttoEA(rt19NOHQCPzNsm*7}$?6NC38u>nLb%hMAUC_K$XE zZJkC(04uHtcC(Ow%0+vV5uYo_Zu)6jm9#N8Q$9C~0u7tGFDFp2G$F?S(d$-yeezj$P%-I+MVLY$UvYWmf0zrTj zS4Kzj5zq!8bSxiZw)v#>!p^8=tEG$WF|t_pF}Ql?B64UmeBm>_i&yBY7vm=hCyQC$PAR88pO{d;Tv^)lEN~#SX zr?&xH=*RhlGAvZW6@7*V@7f6YLH{KxI;oWMA_yE3ltNwvTg%$$wHL48%nMc=gQ-I_ zi)ucP-?hHYE3`w-E&=j^K{ARkyV&~V4v)j1D3%U?{`6g@P|3s$b@+elDY=DvZ` z_EjfboY4Wkx=lD4s~XszqhKplho4ZJOX)?;qgjFFiZ*mWD@RC+)?H#Y?MeoloBD7@ ze$S|~W7N8jyVN88MYG;?!d-udr)sRD^bomF0iLhN9HZW4P%AQ>RmT-51;_a)Q6QOB`K z+t1q!h_Wjnu9e8HqKv+1CmZ;1NG!rUW{i}adqwX~+X7=2gv=K5^dq+8fh6>j)B~LV&JKXb{BO$ zFIj2~0=CD1^}XsgqJ(snrKEOo+)(HayaLOBKivv<`J9qYu#L3ud5)>JsD3Aihn)3) zMv6?(Tqt_kI%7f~ESeP^Kn&9sy~3g>@fWZ$a6jIOQb;-1JRnZx7DhoIYANUboc|~6 zaxqd2sCHRmzjxnlq`jN$QnbZ~ftGAx`|1UL`y2t%C z-n;R9JAo)ySsHL&a0RxU;&Nv=ExhHmnFL}9a)?LRtdOuIT1qlkn15!wkY*2Dni-|Z zXx7So$U_WPB+DNMT6{h9Z2@cr#JZAV3B&GrL;8YwXI){Icvt#|Vxq|hxGZdc@|BdC zxTE&x`&YFDKEi%!2F5YCfep;fl+EM>5jJB)KgUFmaBFrD!DIo*7iqaoRPp~*2&zt- zl^vp;qvPquV8y)0Vk)ix&s@UVAFlT4Ahc83O%DEheJ3C`uM^LVxRc>-FP)Jqkv@tI zDF#Y+V{>uUP@qHO+cO1%x9C!T7;c-L7;ge1EZy}+Ca&Y~mW>xbct17bIhf>Rx!f&p z1RpAAqWAz2b0X-6wm|LQzstn{FWuO`0oWBM+EB)$95f5V2PlxSdep&;3DZCM0?-)` zyJj@M1v0~Ioh;PIqOUZsekaThL@bSpOTY8msACmoOp&R{AW#^|COYeX7G0WeEk3mN zuM^1N`xuDo3}uHH)^3AfLmy%0>Wjh5+9V1s=$609J5rcp=7w1bSX zr#;z+lCvjly{Zsl5X9sRL<>X9gY}JNM+lH`TpUgZS4f|9%(A!h3Uj4L`;1zDpu7R0 z!YlwbrYJ*N$)%8qie*|O)>M_g6?L`){4yg4T>O|8yz_FKygNbvl7}UJFMKhmBw$d5dJHnyy(CN zh2{i?1c90-Q}E_zvvtAjK-d8X6;PQ8;zv@@Z|1CVSuxFCqw~{$7WH43GY|MVso?!a z+7RMZn~>!%_9-LLsHnkaoZxK~ddr@7O`4glDizzV%z7$8{F!wBR$^3^{q$)nsd5C! z8-Td(_{6IYc97S9d&CYeY%5Ng6GK7)4l0seKDuM1K^*}W1ND}nZ-ZgIz~vNk3cH(Y zog?Xk0B_lNI+@rb#6UubNCE!@Tbzo;g(uOB`Mm!j{aHKbg42js0Go@A-yGLqEKQ^7 zGDz%_4>Y_l(j8)Aqg+wY zy9i`u_T(SC>K4BNfs%b83<|f%ilcoG4^zgYxxeK4koir>f+-#}4vi9VPQinnq@Wnb zCQTnDeA6e^FkVM*=bS=q6FUORO-K=o`H>OWe&|Mj!e)#i{T8l|uELF05|!goYq>nM z(W@T?(3Thy^Ik}9E^_0Abzgq47yJsm$3Ysg(Ov*S3i;{+fbLxL-VSP_Bs~}#DAGQe z#T&X(UcqLxhU1s6>#+e67c73yTHA`>l!0P+L=Di?FIi{{OAmZsvgGC`@E$}$S5yaH zgigMHdnHI9AoWUmn9e$|)jr*b&b-<-5&kF=Of=wk1@nv+i2NPVf0qH@by5O#UK9rs ze~Ss*S2uF5$@mA3DAPqu$}|<@l{|Lgm;vRUIUDb@e-qF<4 zNaj7g;P}=cBIIro3Qu*QxJ}ywX%%fAk?#m+;(z2XR^HxfI!G(BrE0)I3@f zvDt#v8u|fxsEg>T3edg$l6~_xdX>?}tniDza6HrRY{9^u_5~vcBFklY$`0p+r6T_M zzrojG(cDiB#s$>vWjJy)E#8f);8n~Q=tE2Q6OmW9c)|kxAi$NKo*PJiUr_UK zyPEN+U*aP}+hpM@3~t)%Osi!KP)7nluugSwAihSMQqLoa0N^*&wS~@2ZIOkh0Yw1f zB~AhtJUr%Ajsb*6WY^D}3nPgTJ@saHp3@gw3j`0sw^8}uugm5Aod-#W+^i<`zqI`! zGPxQVu#YTX-FLl%K+NbID~!s2wq?$M+eOa$^1N7he50r)->(-Ba-jQsC8vrW_#a+e z?L7F`hi0t<{OE^}MXw7a=VsK}pBUt!6-e2gz~51lr%dvHiRcVv;+dt} z&-ge3@k!1IzU}-mQtI8Cly&>%(tL)KIPPfvzvc}44XS9)!$)Z4eLEx#vH@;cAawYp60E0R?W#$s{Ytim{W zn==~qGauqfiEyAqDo%l#;|u9WIfY2?Z0fI{tzd%}1dS8a z@v56wyDSp0yniYaIVzo|E?SARh+aPK#$g>@<+U6Q%7}&8Dv|F9o&uS04)aNFwtg8& z$nm0P$&00yiuv4s$DQ|iDZY3&cNBmH3up)UQ2DuJz#!*mk={f7&r6I|G`eKw4TBq- z&=kNz=*SQnuYD*K?yt_;PHuo2fBwCyW(OsI&Q=JwOc`kfj` zwYbe6-QhSK+?Je%7H9Uiq!SUAO2tZ)J9q(56_X5a&3-%Q^D4WyN5hUo8$PZ)Y!Ge& zYs$uO+u^5wW!rSBYp)0b67`3|W5`LV_m!11UVhkNWM?7{DDQl6Kw?>B$~c z94RQxkj@G9XcegYJ)P0xB2jPRKj4m>GZjbWJ8`U}Dw3BT&vRA8=6s*oki7u(%I8yavyKl!X*-#ZoU|2j7Y$I z;ltTCC+8PQ(>L4}8VnmnNiQmOXVnQuO64r+Adw!O3-%lCdwn z_mcV!6o{QLvt^gV%TuYP;hRX*@)_Qvndw3QHJ_w?}#z^O=p zxWHckvj(89A-HHQD5JAr(TbQk3$o4QWGAkc)~KcSLXNRD&s1KIT*FBV3S@!g7+^2I zy44KX!?!QHdosGvE06q)WK&%6IwqpmxS|t(K42&Ov4M&medX6_6}4`%%?@Io19I~o zF~ijCn4oi&f@0ejsrRU37Qjn)DEeGC23nw1kQM!#ups;*6=ydBkBhWtfe`1i%~T%K z5ki#6V)s6*j`k36KdJ0)Q12^ksCCG8okAJ+;4=41n25en`?bs?`uH+LM|{^sv#B$G zr}SqBA*0DZ@)ON<;;58y<%sE55gS^BQL)>>`Aek7D56a6EIG;b$CAigbQ^}N8bQ(v zphKqIdQMuU9@#l81BzB_Ld>Juj7AOba464I2bOVV+@*iSj&H44kSzf!*;BrkQ;W*G zO>=Qs0{lWgiAHAE_^A-C)PqBN2Kv)~(R3W@U6fIng5OyK-Wdp4OAmGFiDA=jH(vDQ z3c+z4CMSnxUl4PoD{n)?GdG#0TSoByEhk0QIe`|-X5pZv1v%qtf?(X$S9^rvf62BxJo(QTMq;J5Y&>G!+hPB$czRai#hwgXqBXA@nF|<2?b-6b`0;KQDdu zF%5uN(hkGWTASZW z!&cdNAYl$cEQEEuY_{1h7g0WcK;h)1jpP4;=PE8Fy(^Ws)tU|Lf%ddG-Py85?tM8_ zHZ9p&%u%rvK!(5>zoJF95wYTZsU}Z4GTSlt5NV1Oqa0tqz;RHK7yTbMz(=+E-pOT_ zwxqer8^05*DjMi0(o*7#xHoBQl3y4|WJ|gFB9+OeLZ>vwRqXp8MUaGl%U&2ct`3hS ztab1*6kqQ&U<;*&XSG@34$IsKT*GmL}5Se)rpWBV)tNtiST#Ag4^Vp{^$ z`h}KdcmN+aW$06nnkjd8^ykGve!3~o6dB~wFaukU&ErAhmid9}UO3!myL0lwCXHR+ zcb){YMN^uv_yp&^di#$-GP|tC3q=kJAeULDhVtrmi@TaE?|HF*Nyb>^qnNCqPyh*p z1X|YqH}LFG5ko*cr(rc}pwHv3+rS;^>YT86=xd%w4-vserU;V1#PW!|gqPJb3TB_n z57GZ=(hs~Y?DTm%9`aZSQ5776P+Hs}j7Bk_6-m8EVLJI?=NC-3nJK-JIKbTWf!_GP z?u&ax{I;DZsC5p1(CM4DXD(x^ZIBh+`^aKo%|J(JrC09FtF3PY9`O59h`${d(w}fHNlkOk-Wu@B6q}739^Jk8B$}y5`hXi2CEzr@ z$2D_LcNSHDNr+RJ>Rz?v#y46(spZeO9Ly2vf&tMDvh!PJu4#IZxV4zaXT+^`P;tSO zZ>!A*?ioJ4oCi^paQGI4_fMZKC?8C9)N%qENYO-vFc*e>q+k^Y154#TP|NVdyvYIa z-kr2B(8l2Tr1DDFyDF;Js9;$nFR7ZEF8ggyM3!!UbV6uz-4z1Gsn;$Wknn!B`~R+Z zTE_eT4kO6kP*!kTH0y%Mq8a1?<@Y{(4WfbxrpAUw{ZvFOX3(&me6MS6=fyFu`FPA0 z6yPH6`mGkMdKGPDJV4mBlSM;J#G`JY7FR=jF6R8{JOk>XPBVz)o@C~a($BWTE>XMI z!KZ(Jub;m{JH|Ra>8J*M_e#2&7m&r1Tk0J_Pstl#apr>^EivdN zP{Zm_P(Vy5y`iFPz=G}8ibT6sL!w}p^lzZTBg~9NS|p0w|8#xNazPa~AG#Xb==xjX zW0hF>aR&q2%+_V>Dj)=TuMiurGB58z?vHVQGG?4=UOsZQ`6Pk!^B5>^z}v>k(`~HN zs|gdkdaf&TSUDnT;+SW@BW*i~$17b~^vu7}fNKP^&#yttw|+QQd|ZCv20bflLsGC4c%R6)?_ zm+A7>VW1zJQp1(`Q2daAcp3Gs{DqQb?{%C zE5{HK^aL^n`WI@vrM^L!7U&gm3bD?AA?_kT@Dcm2r~(e>&EmFiA#sERjy=j6E!<3x z!Td8e6KTwSk%lld6yo?B7EDawD1c2mY5hIj#HFs&cOXYmdxohhen}G`kSD~ByY*232_c5VdwLir+AW%FoN2G)?lww`j>z}ii zMh(psUuCR(q=#W9?M6~!xNaha8d=Uj2ob(B9da8ⓈU~n~0SqU}U2uU0;6&8z~pC z2T@}#4MyhAnyUJOz(<~Z9$Be5v!^@K`DzbXC@w=i5aVQgb=Zy(4G(931njCjntpuR zH>W=T;puo#eJ|-sYB2}h-4TaW-T9@=Xf?_jnx6Zuvo1;tNa$pgyBX>j@>E0l^Ypgi zXXta|A*0uq0V`~2wDT0+km7S@E&DV8K%(H7u^#0M)Hz%x2+B*G3i6HsO(?*McCNFK zr?_N+Rf118(V0z!Mc-(D-pG(Ox@YqE%{eWX3-%P}aIWGB7KRT^R9P=Qq&!ez4IVnWb(Wr(3CV-OiOr(YICMlhg($cEmC@t$avKEq=BOz>haRefadjJ)kC*t?qG-$kIv2==m5W8+#6dh428aGRa zt@^TjBjhaIW^q?=Xcg{pyHhISLd78#6ZdV4RpW0a4e#*McYNR^3B?+}-fDR-w9x#$ zrm5VyRa`}22F{r66#N>}tsv|UfPEI@Q2!2++l=%1v5pA^ffIi=-WcPB){T4MFZ$34 zhC4EyhfZYG0B{C>Hd7I+3@8`XdrQZTet(;?g!vU$mbyX&>yZ-x_~JJ<`dl;GF{vUR zks4A07Gr#O)Koz;90qi1PihRDZyGS z{b`!Iit4|*a{#6)(b)V)ak})=OxEE%1y_3_6-`*j#iYBFX?Q6e=B zF>-9g0#@^o8%KcZ2{To0 zfDuQk2j#ned_I!4$jK|i6{3<0UP+?@sIJf3u$ugXK!U*m=4lFx)P?PUaNpSpU~ab;E+s_=dejBS zmY!#QtBvDumg9<>%$#Gcl6e>e-co(F>{&#z*Sp|LvP>|N<@b#uz&a$flwlm4EN?rqUE0(M5ZJ=O;@Cy8 zqLOFx(E8jeLyx(NE;7?Ja>4S|NH2Q4mKEfAFgo`1KJ3G zTW7n-AnB+>?Na4O4kpTH5`#hw0AG|2tewOh#8OC+?F*5k_v{vAw3r$manfiMfQoOa zt-1K1ox*0VF}b?b4*=*;e>H--Cd1WuV>dC zTWnl>@9OW_-{&roA{cETV3qc^iE7`Ik+mLK}62l%PmqNv)T01eO<;VTJ$rBDGQ z^aYBA`hKo08uZpc1titNX-KST z(F`GhP4vVR0JreUwV~ss$v#kjz0lu83ktWms#1ph()b#wwGC0$F(EO$6sYC?xrY!D zTcv4k6eyY?#-93&KrV(HBTHE!ac+OkmOB5LStSpnpF%(}pOwX~0>m@Yg5ypI|489Y z=H3XI71+XcXrJjSR*Ul z%HYqva?7^`%s!3@E5b)dx8w4pZkc1%L0_0)`&&+--TleCJt!+gzjkE=(A9VlaH`1? z!A}ZR9J@7l#sDlR>7N97P%n>Tky~sOfrH;u#q}YsqgY?)Q3p70=SO1!(WQ|*OQpJ> z%mHFbs$u&%E4BxJ3+4=DbSoBV99e1fw7}!Hz{+B!<~ci4q-A=pn(Y$&8U?LwsalnV zlr791SBmJX3=Dvew?mF`?p;l`LAZluupwyo6h^5SY7zP;0$pae@okGFIhc}kvQ?e` zM@f$*4TipGu(E1xDGqAg-@9h`Sjch@RCJmMx4h~%p7&ROZ}hUAk5E}^Wt7=$>tz`k zkZ{caspDvHlh2X9#!Avq6ruhtZM!}EZjblT!PGNhWxK?H(cxzWS6lWB4C}>(xrE6! zSkKpS^0Kp?K)~v)^j6~6sGkBgw%C#_<1WNbWqTKaP3~3c)uC3|C|D&H+{S0p3n}nHOS^Lec$cmfx@h zGp7iK@F&W^6@!;-nKH8(88l0#>tP0+cxAAx-thf@QiOYzUP{1u!?C_3&Uql&LYvlB z=Rg6$^PT#^^TkQseGb_#!FpZ=Uwkl{(N|AFUlQ|>DASh|<#jq0yR{uwYS#AWWUBV$2j2?|OcI^z zcM#PXk9;4Wfj7<<4mgv$p)$b1-4X8vGhCP2B~SpR-Dx%0p91)oia-tkD?coLR*Azi zFCx>9)QPe+PL-83)FU9x5Y0J63!#+k`BkiceyJp)d=8k45hgFB-!_rZmGYg>&buip zjgPorY3|HtU9*U4j;S3tO2auuOP7Q2ATMa2sY7AXiEG6L{jjjs4EE78ud%wfk7wn?%zn*hMRk5rBSd$~?}wxZOXnyVHKx0+o01N+6`bFJ%GfhcvQ(NN zy79N+Gy_xWwgr4~xZ?%S*z0*7nRtqQy?f=Qr@GI-bWQNm0P|GO91{tmIU_26c<^D^ zme5q54stBh+ek|wOn|;UJ`^qQQ<2~+aC!dYcD0R80ZptLy(mGN;k@DfJvJ-_yRk5X$nJ|VHaT; z-H|l7LK)~CQejn94i`40_WN|F`4jBq2+m1>&NLSK=>6yaoUq2<(PwYhi~C#K0x2(e zhoqdIY>?1JU7?NO+y-KQM=mBifQdsu+8_H%P(IpqleXK0*-Z=F%CM=OE@(2Ev$?I1 zE*0`OWq88{Fb-Z3{qlJ&p)kl$dOvwwAy5Ye8zu=9^|6-R#!zsnnhn&J&nr8i-AL#2 z#K6hUMyA-H+P(Rn1p^w*ze|GO$Ej(}QT?3khvRshF{qnhD-BS8F$&T#7JORaja3Gh z3)eF$v1hU6+vCTQct)U~VN0e^qt8sj_r?6_{50tSVH=CBhvbt3@wjy=Pu3;^h_e&q zyXfHwXIp?a+oTuDshS1pJL_AF-mL7hOZW5#489kBzz_E!C{1x#oSy1%iV;62S>+P# zVrqB$3F4!TotdYMcD zCMS@D0*U5tSrQ<_W+}G5DDLwL7mu1flgxb3HuIWw5z>#$LI8CsHS+V>MB(GCTbGr0 zig^7}M}CZq?Vy2y`_5;;17&u%05c8zt%FT?-T!zWdNC@0r!1XVZHYU3G)Sm3l}C>Y zj1q|@6{@$>SWhG~R*}%r0;g?X?TsQPLAvV^3{qR}gJa}+0&@)T;-(n&UZb9qN6B8` zAgn*z32<}ivlR9q*^k^=`%+w&PdlpGzSivRkiKnO4vY+`p>)(%-JL##Bse5|CJmH8 zSd~U5MQ>q$hv-V9SY1K{Ra9ELs$+Tb6lClIFgFO*5I4zOAl^3b?=h~CIA%EVEPAZmqyb35(Suj=*>sM?*agS=Mr9x|=e$zDV*9{VIv ztVHw1#mlG-8$-Pt`3tX|u(Q=Gcx>nLSn968cZF8_fJfRqtiTLn(lSx@k^FAqk2edu zz!qYE@VSd7Zzm&blSQp0CAf2HpT1a^no_VT zG6T#%u-Dp#z#a*LyK}V+4e|zBg0{eiV1ODNu*RNAK{OUbuUI*w-jOKQ7$GFo5KuxL zRQv|D?U+fb`5M0O>uV8H_m#5zewsf5RkpoP6>e!mB7knbvqx>jJVwtl)OAwYVpV)k??PYYo-YVAKe$3ro6fyVj640{l}7j zPJoB#j)p2s1dS(_OxoEZu}M^+mKHz$(%iBI&puD=#eK#-s7`#!+nneb)8c;1zNCvR z!+%-c&Y{ZvcG}#MG2O70WWGb4A)dzGsc-SdGUn{o*B6UB|G{ju5sQ(ayR@G&G>46&Piwv62d#R^QGbck@Ad2>gg$=U`^zGLHOH? zR-=bY32VX5T+M%geA}`nX*0uV8RtB0dLlIX@*-Q`NX%)F2Aj_`f47pM6~t^o9sMu1 zeq5t8+|ZIHv8t0a4v$7u4KQqZu^NxC`)D!GhG*oR8bZQabGH%b_5e|&3jG6ro{jx{ zfPrG}{No~}u{UXMiUL+FoErPI?k!L(fab?A_3{{X&1wA#I7J?1@Q~vdaKolMw#$6E z7T_T6QE|kZC-rX+CMZMs+x8~3rLvzx3ovi7EGv5n!7$WhNVtfF3$d6D_F3w4fji0f zU#4Zo+#DN%^<72*a>#K-uVLzcl3=@r$CBhdwJA!k76%1xe!E@@@#$vzci9Lygj*8g z{ZTM(Ee6&a&84S6zAe}(;nqTAgoID!k0m*1r|_AtuW;mc@Hx3}Ikc+9qo#3bMb6m7 zv-UI+t6Xj`4v%C9mulmpwm9;QKW2rvmh5RS3WQJavpU3*q{$z$HU=Jliu}P8poXGZCl>=+iiaOyE^kPxEJKjgBU4HQ`RufjzcK_z z&eKxPtqIx+?n>ms%POSn53qCg>|A_#mMFMzmqT5-CWMEjrW#SySml%NouE3a29Sn} zaZV%9GWL>A(Zs^=Ld3m)25wG9GbQhZM6w9GNcl>*lKq!1S_#8)!w^9Ga-EtAskpid z!v-9#k~$9t_)MuVRI5B*7`Jc+My;~6Ej2hzyknJZ(7sHs}4 zF0&!P=G>YX*A+`*{UsWdboP`ss-2q>O9D>OBW}Q&(m@F;pSSja! zWh}hN>!fZOj~B;e$3%QAXw-zytgmxrm7{_@T65$}4XBF||NUs*a2I&z8*^l$(=)Q< zpV9)prrdUN2@Hfe+D-)!2@7tRw1RoZMHcN~4fg3?P#gY#05Ck-6r09AKlMHCT;^jP zXA#?k!%<^Q1i0JVNBq~VtRGH{|E-Gb1P^O21bso*WKrE~-}6SMY?BpuZq{|rXQ@4S zJS(jQr=lwOD;IUtOS7n(+U-fzUKYX{yewM|6y}&Wo<{~HlG9qp7{4>sjhl>I9mJ(H zfA;6j!~fcUes_2qKKD;F$J=rReEAork};yAO*D61UTyrB6W+5Bo_XG`;$om8cwL5k zcryE|w-kos^YY(f=u8=ENVP0=Cr#1I$yv=@1q8`0zb*|il|JLN#S|<@VDX^uq+Mtp zN1WVqm)@_fcEoER##fJFwRtF|CMqF^?;WhR`=W(^;G7I4^KQ=T_dUKva07f##@11X zYV=r*1my>R#>8*YWo0ig5CWf1-_M2l|MrjFh=t&&biQY?jP%9{hW{4b5cyclxUz?) zT%7yxW6+T1^g_HKpyl-4T6)qcNfzS~E6dT?IR~nOXQ055M*_ah)==zEC(&G0(|a`1 zDnN06NcjEGI$75a*Y2^43CzspK|OCz&>3vCuM@#lJQXyc^bM6u=i^e)_g`Kcf;6Z8dpW5b)b_l?;eYBbMrT%DBm-Xb^y!NLN< z!Z)?dpvMSUhFl1#fbyBX4+@46LH3>U>B|;>*%bccJoH29y1p|qZEuh?ykLse%Gs4P z)&4;vuBuS>D9ymre=o}U^8U$5Uak02RgB8CsEyD*q=C2Oo8XBuX3f=J1tf$(Qht4W zbg4Nk5X#)6#!~!E)la`F3_sAv8E6Gr6&=fw>o(W}6bmrX?9(CdVW@jBU81its}MSW zJ$8_=RD5Z-DjCHJV=BteB5g0R1mG%^5P5wwab`g&z52yGuE|=u8lv9~!8Fdy6r#e{fyUb%y4LLBtm! zV>Cl?lEm?H{1a%w+BI3$l9paOYa6+LjL6B9{1?+&GBV28_HJ5sN_A98o@61Vh;)Yj z{&>dl{i>+VHU!rTrwOIvX1#p{4eQ_4pKZ+Cjj(Mo7|&pp-OD88-Tiu!yQOjIN4gPu z4mcW+A+o>mKrA~yjv}*3FO+iQ!Tb>Gecki{S-smn`N^276B;uT zr*(w;rDO1zh`SIaMQaaq8YbK4aLyxW`H**Z6F_EINW6#Q%4JV%760(Fv z+kFF^LmWaJ7F%`MJQiE@bcv4%vzf@9;nc&?$t`;W%?#mKHGpYRZuy`GzG%oMZaZzp zqLqW1q^;H=5m6D#XmRXKKdjGl`_?4bXS0sYRAdV|?oz576MpZo_)i3XOQE^`1BPQ# z@f>?G89(s59Y{re!2q8o$)^wj{J_;82M^#1`$_mOsZ}T0&HYV7@*u#SAUg*uGASLbmNscz8On#WFA zR?YJBs8a*n?dc0lP>4l;@Al#5rTUB$6sgv<^9Q@n@3-$LEbwQcJrJ^`>t`9961@i! z_2BoClDGVH0LFqq6R;(gJPAh(v*F6iSM)sKOoF6Nf85JH98xT6Dyw*kog-dCSG()Y zxhqU3QFuQD%i_Q3>Hrk1yDtPlFCzo2$nW!S=uhU`5J2=*c!&LeFq?cKzPS%$NEQ-i z-j@+SqI6s5OqK0>mfTPjj|pC2^f25Tih`i|l0l73#MeuP=9?iDLk98$0l>rhmABKE8g-kt=Ci75MMUG%l}e04A&00B+eD z&6#=tH?(ZbnEjD|*?lzm?u%=hv-~480DscK)=M3qqZi$JE^X3l_qW~M)Cn3DbOE_D z4Uj|tERDG2@dW>Bq>ujE@_Ee3TE__#Zm)vd28fW)islki8FqhWPtsLFb;~1-cElQi zr`y8C>j_A9A<@y4iyhZ;C%_QVY-#2Jn(*0B++^o^3)O~yR&;vezUqYdq(VwwRL1G= zXh|}24DNBEzaM@9NMk-`P7GHuSXpNmBFT$$_ivpDhineq)z7j8066@>i>~(TdvB1! z^5EEBAx6Y(u4Ii4>HZN!-xnBeI2Gy&D)jYcgqa*8_2>^W}|P z@+C5j5c;Xibdm z9XZoik5bUg7WE_Bk27(}C7dxZ7OL%Of$3+AV3o8Jdlyj4|47=`e@%4;h^fz64R>A1*R!h?=Gf!Gr5Sjnm}Qu%Xgr_JZVpSf5b2=ATf-y9i@{&Rs^t}3>GsHczlO1hZ~IC>NY zrtlrOJiMN!nibpwdSY!bhC^<9Di-7DyK?p*W+?ed#WCI-MzIH;Kawv*;BFVAyGG{KF!Iq1RAAk;_1uBso!lwxyF! zFhJGQ;L**YWRqOQR(WhG)k^c^wD6B5N1)|Dm12kUEe;xb2@%UvyWT|3+Vem`+g-WUJgLLsYhf>r{sk=3nCe~V73+){?Ed)VG<^p^mSJOpd zyo!Y(*R2g-8jq4CNOvmT;s>O*C6=;(-(#c}Pcba8Qy7r!h@+R4&gN@m@krso?YP*E z3qkWv9sI=%-L;Di%V$x0(WZwF43Pz>$uCT{=0u4vgbe>U?h#piB$79RK;)XXj>cQ$ zw3V_KA?|1-29MsQuEp&k(k~9HFsB@7%J0$`s^BxKPYJfyPapoPM0xI{x1|4y%>BEy&*&vhL~4y>%6yqnXzhKl!il;Ekw{ zp&KU*xeg8hmIUD+bL+}F|1aEnx!xym1`pR)HJx4 z4TQg#Puw7l#}O#(Ofml6LTmpt$(g!kxdDS~-&R#66L)?NwQa4Pq zE{$V`PSHb1|8c7r^Y5Ejx6>%|xaBp`lG3^QQg2K152w&K;$gproyvFSW$j=sJ4)ApyViu0n2P^&E@ZS)_5G6f*6?hmlNYN)7 zNKws(NGB)fr30az_~mP6HH5;Vnk3C}gbUB4Sy&%a=FB9TM3Wt+o0h2L^-l}$Vh+!u zL<{jhaoXZ{-A;1eDCPWlWwW&Fjm$raF(RoJcl(OY;wrux4qt_x1yGyKy2pzaX(^?p zID}%w9fB8kEv|(kh2j(~@D?j>L0ha)pcD!2?ocFna0~A4a?|g8=X~eR+?hL*P2Oks zm;du*_ub9RZnEooN=I@B_CZcl*?mlsqxDDMcl*%T;EH_>j2b#P62`E*Z0JW7`Dvbh zH(6i6SXt(YC)vWPWUhW0ZV3DKRJo4wX!Nkw`hhhkjyWT(5ip@kY(Is^s~YJE>lw~o zzU4Jg&}_aeEE~I+F?pKFG%|BhUFPO{_xV-3XwYrDAD(xnY{w032aWO3*9Tq0_M@>P zC;MXZs_&jJ0Dy!I16~&M1xPB07^})8kV{~~dxb8bt&G(G*Y~{>G(kXy(BbX*VMA;b zmF`H9W|p*VO0mb=#F|%_^{#k=4J0j`LQ`llUE?|Mle$J_%8nXp={OB$1C3-JmT&E@ zej6ux?YS(hZLj!Cf`TiGnKol4z|jQaIZfpnThBjo05Q1j$}`+@xfU$85T9E1d*b5u z-I*`%errcz?&`jP?9t4S(Tz^hYm<&Qyij=SFyZ+{@OzFe-_i#v*|MHJubTcz%Py-x zsdou!EE!iUiKTrc4!p~UL*_cq=X4|Thu4DYClF0C8 zoERRz*P!l=AONd{=wQ7Nu+a0S4Z?Wzw&7W^24`m{WIRh}E6ea_9Ch!rIQlEXYLs;p zH;be)BII%RkxHm@8ga2>3{M0&c>Z&!Jr6&3(jgHFBhaaD;6lbf;sY;E?#`=fLm8^k z&e1SgQQVew9GxYs#F}>ys$)o!uC|JyFwY9P z8xkR_3t%#55MxfSSg@EP+CdNoXNe}{NIy4P5!;qb;<-L7RUAy$&8Ul?WaGf=rZoLY1YR2E$E^!N6naq(e?-8FDogMW3Pg#YG8#XhlxRovThrf;mE`V&I{l^QjdB;y}>ERg5+$j=75Y*{0 z>9j-LsN^YncxQ`F2#kimLy4B}EwQ|`OPeiSyOYf*9jcp|C0H1?o^H+AuPHT_oy@d0 zcmUF86n)vp$((kHZigw`Pfr!pG#k@w&yF+?Uk)eL`{>;%O(WV6HRtv&>UPgN~U)PUlAL6y7=olfQ;cii- zl8tv%{XUB{fkndH8EC61mmxm*(mu;PHselCV5pk1-#bT4X?&5Z$>7x?p_tsFC?ucA zf;6QIqq}24;$^6=9jU)Z`_xDw zDPy{m(kBiaL^V|v)#RB7&Pw;z4DZu%d7$}|HS6I=;r*+1cHi|Cm5`rSA*N=@soOJs zeL4&#z^%MN*)z6?AGSU`&tMs{Q;=^l0)rDlV0FWnvQwn|OTEtVtgusT;F88JGX2$_ zSXZKVat%U)#*TrBJP8MJN?U`_Fs(>e1LLv5$E9j%>=H82p|BqsrVNP;R(8{;=P#1N zSIbj2I}6G7JOqu3t@=I~zdhazh~9R{DEhfg!rrg}-lSrK!sm2MTyG_nip{#(PgRq` z!;$~6Jco4G+?Q+v={|;lyMdb6y(f43;=U!r&WV>uy>YR}l5@>%wpd2di|W4cDW>A% z+U?$*0dKFWi^)dK*(FE{u`$(;@wjB;(&|k}aE~HYf5A|#f7g&CUYgE!C|#p!T*+L& zj96$e({>E&Qx~t)q9@jnEaa!Z2zBXRauxn?;?#bm|2vM8!lDldUq$~gb)Rimcege- zb6<}<$q`m< zs?jy#WycvK2T{OgW9f$v-KW(SXSrep0*z|;5n|EiQu%0{+LeG zfe+%AxpmZBjXV{rYi34K+jq{7CM~(mC>tv}Wef)vw}$T&47wQY4mGM-M7t@Y!nZTk z28elz6<=6zTp7siM&8dl?H^@;kU5kq*AYD6V4T)3!ddPCy{2UB|}>R0Jq@ z3NCft6w5LH}rv$>u`L!uBJOYSo z*zE=a2Dkx82Ax~3=D^LD-Pq$g!e@C+)n>+p{xcXbJi7hb1&R=#W@S(LSBB#xzh>I{ zeGQIuRfDzrU|F7PW89^~i54c5BB zq{ThyIU+}`Pw>d!LfPHbxkxr=cLs%`baB$YZHkCbdn& z>3_qq)BCjccWIl$TMAzh7XG$`GTUO5s=F~P`vr{DHRL~U!nOLk-}YfH>}gh8BVj#n zw=2NxJx-&k1h?yFZi!iEB@>$6DKV-Jf{#E3$xbM9y{(q2&1&s&5xo0o*Mx9agD#4% zvUa&>hkMnWYoxH7F}=K*wk}@-XB{1%0%ePV!*Let3OcyH_<)1;!Y#Aod2jJd-O4s! zG&PPdu;Q&fY+M=h;ijU!&u3gPY5#YTGH{Sy69ml-IZ4JZ(fVbm`P6j6T}q-bht-|v z+S%|t34zaH9n5)rk;BTjY$Kx+eWo+brD7ePR9D$*TZ$3B5%7C->N^-q(IUN~J?l5d z5~k2mN>R=1?&B~z>x`eaq}P4f?37q#g$GF@&+BLL{A>4)Qir~MagLb(26eWR`vFwW zbR;4mP17>j;}I+IJmaez*+MgFBYF%goG!~-1=P*4?dsOy`?qBe+;(r}+^h#s+Hmc@ z9!S{Dh1HXMeTEL_f$vVJ`kW_j4w#(ixq7gAJP*?Ha^6Ne7!!GEW8o}*jrKll?h8C@ zuC9naj5yP>)~1h#!h3z>Eb!J>Y#F%a`(?P?ubSpphrw`a(yL>%Q>4wD&S*SXE@MDY zSgAeJxird(zt62+eM+Fl&S|#hJuR!pvZYtLka$HUi;(8;Vv!Es3XqV)2@_chn#f{v zPJdS5^6G6G)%-;}9(}1Ax%$v5*bA$fx~P+06|zx&G5)l!Fg2cUr2((r>ob7K(w!q# z29(!smE0$0mJ=|^ez|lOEKsg*G~Ro4n_iswOFmdzF z*0nTsp^&KWXkxQ_lhEXHwbdQAe|u2fk`dC z>eYI?Q6kTiBj37?87TWzc<1=|NyP&X@cN8x#VK?D{c~9KdDHb~uLC*ElZJfyOMB-b zQJcCfdO30dbNjq+s_-6eTN|T(Sub^ZPNYgDCXC2SZ@iF7{LOF02e6~P(z&P=nai^UE39 zJ2szAp$W`@ija$(3#!-Qz7w$PzPo&rGp@SqJ%YK>V-pyq4{SJ1O#K&Ono9bQc6YZ6 z4L0=zsiy{nUj^w|01a2&g_DJbudu}Oaqsbr({phD!`t?|nLG9SntSMd(|sl|azA-L zc3)BHHJxz|yUMsWxiYEpX+CQ{ZRQpqwS%2(SymY;7cay^JjyQ|=bHCS7L1JtC;Abc z%g#L@JO%cpj0;e7wT!Beb$ zP51^F>~L+~ATp>Y6+CU$8X*>*QG+|&$mt4ip5y2X71d=7o+ba?@RB?@8NeWs%vc0 zM?3S!Ci_6z>Pu@?6bep^144FR2T)i?H?5Ud-SzXyqJ%jdl5K?b{ke=$!{o0Bc<@LD zc7{+}T5 z4pxQl6OZ5Q8Z_;aF4#1U8Edfrv`EVu+{m=iwYJ|{g`BRbNl^stHY`BpZjv2K$iX9J zO!0N(HXrh0A&%MQJBZXn$hBc&t&EG&3J&9HDzj6$$D=;1zH+LbuF|>ADXy>Y`^sI<^;c&0XJ#Z0v zT(NDa)zquqbUWv>&s;%qF27%Q?z*F2|MWGvZ+8h}+zp~cNOC$!t3KvE5d@Er7P^Q7 zxpH_tz0l!Ky{q-}Oj&NN8O0hLlr!T&ZnBtn?oVdAnH=)H0vL(mHdjL~S3D*8Mr!~Y ztS(Lcs@#W|qK{a!i^rarJ3b>dxlC@wq{J8cgv-LVtdr>R#V_IUBbD8R(q`RvjHAn1x?487q*=R`y%q6Vr-+oLK;Ld_oi`R}tXXD8OT zD!9u6K2^{R2i2!maBVnuy5G0o*fh;M36bvkkwNjl9GGc_LqT=qby*un8wy#>smroo zvzW6K_g|9xM7yzm#->U3QloDJ52I8yyv3E7#;xL+JUWt*qS8b&7A)EUNDp{=NON0q zam_J8iOn&v@&BEMlayVlK;F@Ak3W-FJl*64F^8Ox#t=$*J?_DI*@$T_9cvu0@s6f( zt%B=ACYp}AqeY8W{KY?s;DXpmPXq6}%g-NLXcT_=v=GJbOxP{x`4Bm_zpc;}0>EE2 za;5G3JscGa3Rw(m(IPN>TBG34YzB=WctAoG3S!ZwTO~cXv9^GJw&Hw~kS7}MrHNW9 zof!YxmbDJutk0aJHcUSnDs$TUM}(Nezb7|x10waArS>g1F@ZQZCP&(fA`2dQ(sukF z{(1d%;?vVVPa(+PpQDzd=>g8NP9R0L6ujXS%-8%XCRHC$(Dh^pP|6~D=s(dsc`ocU3UndfN#R{44!fbGI!5uhd|IETX{F#LT=)nQ?VV9xyIQthq zD9g@ABD@d2{6PjHyW>N{L5lehtg>vTP|r8dLuh&+Tr@Ae7UcGIeM@U=r5)*b<NT{Y5rpz>Y8CMj11yEd6P>4Z?W~0k5GKCq^JJLx>Yx|xGT(i4eKB(FqwA3d{@e}5c?Fd<9 z(;vpz%X((C$fodWQDuVyt=4YD)O+f$w%dcCWBrpJ^!Vv+XBCJXL!AzhWt? z=%HT`Ac2Lx%_$Zi7)M}OVLQ00y0qe6=>d9p04=t|T>pg;cF@fq)k;=h7Pp?V?Z zl?ju#?LsOAhH=F-(rU*0Ez;MvAYD}kqv(IqpW=PMMNIwW(TM7a@Dm zm;kn``S(HjJQ_4tmF2W>zZJSn_eU#y*p2VJecc3a2I;OTcl95%YQ_AM5{CG7H_e-< zs~LG9+uZ}KJpBB5E(BLSuhafi0kDr&CgGR-2#R zVS3MY%$rIDe?wal4sv)%*)yvC(NFjvW`dkPy6b%DZ)St~6>X_Y4DVnxe;KZMD4Ywu z)2`Ei*nhbyoAf~Dj;~nh{&D4{$qbH#y}8RrXA4uiKUWTBHaNWe0!bmIsAsRZczC$D zm^nBZI5-#>aoCj|%%w~}T7X_j33738As-z4oPQxnxn-ycC>%+`<)~yN+yXf4GS<#6 zA3@waT>qI40+Qt&$Ymb|?H@C5B@1+H=^L|tHU)*Si))(;V2EO!wcx)s zkl_(nSqN4gu#Wv^b3G=a$$Po=j
%G1Qp1d@g7>5MDjm9LW4%IS1tv#X&)m;juk zWr4M$#@ut0ntrGqi!<8m^G(PN<#!m{V2s2L0ZI>bRP0Jl;PXy~&p^yb-jRkRQ&50` z9I^K%HLuf?h?p*dCwYoJ$g^QzA$`IMdIqvEc!LdJG{wi1$`#^xl90cV^o>_H=qU;> znU>sB_Mdoq^u7uyHG)$f>R8661hwBUy+|cc*4fx_1RqwbG38XT zvAdERHwn+)IA0&0-~)GeYr)5xh;Z^g^&2VE+fOyguirT5D!g&am3iZotMV+>dPvjS z%cbUWa}O~$)$L1qzIL!RF7;LrliJ=eH;65TTl(8gp46ygt~V{C)$)#FYM-Jkx2-CL zUc#0)MC4L{b)`*^|X2mkgUlCwq&|ie~wP3K-e$88i zqzHKK&?c2;bnRl(GK00h7J7Ga>$R5tHrJwH9zO4HnSxi*6jKh=PDn@)Y#_C@EwMmlz2p+AM|f+9}#Q zh~d|!@L4ULO^sV~GwY-Pehn=eAK zLfjtE+GglPmK8DYurg!C*(+iO%2yLmE7c`nnU18P#`2^}g$k`fN{VQASZT51Oh?g8 zxop3Il)fm(btr1TCZuoAEmD}|R3&S-3s?)w=O9UxCj{u7N=cb@w#=lp>1~{tm|0b{ ze^@{Z?CDHob~mWx@K+OZ#QPB z&5#C|k50-nO!2sl$S0~Wq}f*6Gf=pbY^FM<6Ad*Pd0fr=$TqpLU~blY>IYG{N0MZ} z*$E@de#5fbik)<+SH490$<+s6rIdsu@Qtku=2A_;5QyCU8EK$PEtXG{58&1Ml9Q;j zT_)HtRDV~>MIcNEci2#2+eV({&6GYDe8CK04eC#$3QdEu^BIiUjYV-xbglW9(MG&y zZKIcEkepmGx1c56allT4<1SYgdU=!bCUqJ(yES;o56pv zxa~)~@2~7z6#AF4Tt!?dUG-nFz(ksxfa_hV^%-C=$%y#aWq;_|qwQKcSS}2z8cCV7 zX&k1Sbf`O;aGs}sdh(q=2$l#_?8xl(q`f9% zCG);MOJ+C5Nz7-c?)ymDZ#paSRpr1q8!X%<;hR?eS?`b$_HIaQ->m0c$xo+9BO!|o zKU5K!?~yDOw5C7BlxbASG*6-9RMNaQr(f(1-{;6!n{2c9xvEqKN|NcmhvV$>X3miG znAFnFd` zr)9ZoGPhAKof-6G8X{iL`2JZ?nwyi+BOyp-qeOcWt;qew>85zs+ua7XF0uP-Zyf%G zg>(n8*=DiX8~O#|KrYI4uU|iOXNpdX#TJ@{=59_MuTB?^gX>j)2Dc9sXQ*a0efrc? zfB*b<=jpv{e7*gD2Kh(-jPhWPB#}DQmjI^`4*MHxFAESam=lLx2V}qr;s*W6)NpY4 z2;$`jgE&AMIPCKFmJUcu;BQL`WFXAR$IZhFmXH98b4YP=fcZEixdo*7csV7wq&cOf zdALPE|36~%_@D0xE6&h=-u!56q|!w+b^1{Ma9K zw!FMwyUwjdZfapI;%>@+rlg-@-Dq@ER=btFU1Uef&VbyZSp~t8eqIG6KnH8SwK5fU zk@lOhPMC`Dy`|wZEk_%y6%1i*9JsUh?k$>JYlJP62(Q~6PexHphGOir*+)=WvY(lX z;k_ky%&V>iB8#1fWgVY)JCZY$gv5E^{PcdvUgJdyThue`+?;=L$B9SL99d4>oOwus zcqkl=TN9$_@igg>0J;dU8gB0{KI!sgM-VoI!4CwXh{r5TnD)yr>=DY|AwrF5!oq{~ z;h$=vs}CRSFB&xjJ1kP?dQe3lv~qEt`OMC2!n!l5mfi`sGKh8vNyt%W==vqZD9F)i zU#F$-@7CW_QrY$;@9u7mc>GFLQ+dS@|LiyRYKLXNDeeiu3@8S`m&2m9CBbbpG9eGn zr`052!cGWOkf6(isF{)bzm@<|$NVAsiNhP`?yksm)wU#UR1{MgcR9-$^TdR-_Czixv`#}m9uvp+Ef0YI!G>WxXanm zM{wfX+1YV%GywlQ*1yHBI@b9O0mS>!lPzCC^k%bsU=Kc%#nqQukVociMf)Tg1uROvzi-$=jV$ zTVARx)1_|hmfuGpGQKd(jd43%kqbNi1gsW+3Hea6FOm-;P}!CVj5{o5TNCDe0XK4h z5PraA+*;uAxELWg-$G!&NN>NyH{W4!LG1qNx_#B@Mr5{o#^ba50o#20d&Ix)N)*wK zci!eJr4w=UcPHUa4SXY7HZ+y7h$havhr=>ad|liYk2_+_(yxP1h_YU&aQMwo+)`mZ&13hC zPdO!5c{JC0^gkx>m;FY-lSo^+AXjTN-+H{lMxVFn+y64V^Rs#xbSXcU{XFeuZyq&t zRVqPc<_yr%jpR-4)GuuJvTc#EZTr)ys?5mN{%Q`~$)LH(fX2z-Tkz)qa9}&w{|9)2 nA>+>Ja{XdJ<67((`mF#tQYSrcK_$cG;zX_{)6q$*%HaGDdF)FR diff --git a/Einführung in die Neurowissenschaften - Fragenkatalog.tex b/Einführung in die Neurowissenschaften - Fragenkatalog.tex index 2009ded..9cdbdfc 100644 --- a/Einführung in die Neurowissenschaften - Fragenkatalog.tex +++ b/Einführung in die Neurowissenschaften - Fragenkatalog.tex @@ -685,7 +685,7 @@ \question Wie viele Hirnventrikel gibt es? \begin{solution} -5 + 5 \end{solution} \question Wo und durch welche Struktur wird das Nervenwasser gebildet? @@ -718,22 +718,22 @@ \question Wie viele Spinalnervenpaare gibt es? \begin{solution} -31 Paare + 31 Paare \end{solution} \question Was ist ein Dermatom? \begin{solution} -\begin{itemize*} - \item Assoziation zwischen Körperoberfläche und Spiralnerv/ Rückenmarkssegmente - \item Klinisch bedeutsam für diagnose von Schäden -\end{itemize*} + \begin{itemize*} + \item Assoziation zwischen Körperoberfläche und Spiralnerv/ Rückenmarkssegmente + \item Klinisch bedeutsam für diagnose von Schäden + \end{itemize*} \end{solution} \question Nennen Sie die drei versorgenden Arterien des Rückenmarks! \begin{solution} \begin{itemize*} \item A. spinales posterolateralis (paar) - \item A spinales anterior (unpaar) + \item A spinales anterior (unpaar) \end{itemize*} \end{solution} @@ -1008,9 +1008,9 @@ \question Welche Typen von Berührungs/Drucksensoren gibt es? \begin{solution} \begin{itemize*} - \item Langsam adaptierend: Druckwahrnehmung - \item Schnell adaptierend: Berührungswahrnehmung - \item Sehr schnell adaptierend: Vibrationswahrnehmung + \item Langsam adaptierend: Druckwahrnehmung + \item Schnell adaptierend: Berührungswahrnehmung + \item Sehr schnell adaptierend: Vibrationswahrnehmung \end{itemize*} \end{solution} @@ -1118,7 +1118,7 @@ \begin{solution} \begin{itemize*} \item Sammellinsen - Weitsichtigkeit - \item Zerstreuungslinsen - Kurzsichtigkeit + \item Zerstreuungslinsen - Kurzsichtigkeit \end{itemize*} \end{solution} @@ -1179,502 +1179,530 @@ \question Welche Auswirkungen hat laterale Inhibition in der Retina auf die Qualität der visuellen Information? \begin{solution} - + Kontrasterhöhung \end{solution} \question Wie heißt die Eintrittsstelle des Sehnervs in den Augapfel und wodurch ist diese gekennzeichnet? \begin{solution} - + Blinder Fleck, keine Fotorezeptoren, die Lichtreize aufnehmen können \end{solution} \question An welcher Stelle der Retina ist die Zapfendichte am höchsten? \begin{solution} - + Sehgrube \end{solution} \question In welchem Großhirnlappen befindet sich die primäre Sehrinde? \begin{solution} - + Primärer visueller Cortex \end{solution} \question Welche Auswirkungen hat die Durchtrennung des rechten Sehnerves? \begin{solution} - + Erblindung des Rechten Auges \end{solution} \question Welche Auswirkungen hat die Durchtrennung optischen Tracts? \begin{solution} - + Ausfall des linken/rechten Gesichtsfeldes beider Augen \end{solution} \question Welche Auswirkungen haben Läsionen im primären visuellen Cortex? \begin{solution} - + Skotome: blinde Stellen im Gesichtsfeld \end{solution} \question Was sind die Auswirkungen von Läsionen im posterioren Parietallappen auf die visuelle Wahrnehmung? \begin{solution} - + dass Patienten nicht mehr nach Dingen greifen können, die sie problemlos erkennen \end{solution} \question Was sind die Auswirkungen von Läsionen im inferioren Temporallappen auf die visuelle Wahrnehmung? \begin{solution} - + dass Patienten Dinge greifen können, die sie aber nicht beschreiben können \end{solution} \question Wozu dienen nach der alternativen Theorie von Logothetis und Steinberg die ventrale und die dorsale Bahn des visuellen Systems? \begin{solution} - + Dorsale Bahn dient der Verhaltensinteraktion der Objekte, ventrale Bahn der bewussten Wahrnehmung \end{solution} \question Was versteht man unter Propagnosie? \begin{solution} - + Unfähigkeit Gesichter zu erkennen \end{solution} \question In welchem Quadranten der primären Sehrinde wird die Information aus dem rechten unteren Quadranten des Gesichtsfelds des rechten Auges verarbeitet? \begin{solution} - + primärer visueller Cortex \end{solution} \question Aus welchen Beobachtungen resultiert die Farbtheorie von Young und Helmholtz? \begin{solution} - + Jede Farbe des sichtbaren Spektrums kann aus drei beliebigen unabhängigen Farben gemischt werden \end{solution} \question Aus welchen Beobachtungen resultiert die Farbtheorie von Hering? \begin{solution} - + Farben lassen sich nicht beliebig mischen (z.b. kein rötliche Grün), Schattenbilder nach Starren auf Farben \end{solution} \question Welche der beiden Farbtheorien ist tatsächlich im Gehirn implementiert? \begin{solution} - + Beide \end{solution} \question Wovon hängt die wahrgenommene Farbe einer Fläche ab? \begin{solution} - + reflektierte Wellenlänge, das benutzte Lichtspektrum, falls Fläche nicht isoliert: Umgebende Objekte \end{solution} \question Was sind die beiden möglichen Erklärungen für Blindsehen? \begin{solution} - + Primärer Visueller Cortex nicht vollständig zerstört; direkte Verbindung Mittelhirn und Thalamus zu höheren viusellen Gebieten \end{solution} \question Was sind die drei Abschnitte des Ohres? \begin{solution} - + Inneres, mittleres und äußeres Ohr \end{solution} \question Welche Struktur trennt äußeres Ohr von Mittelohr? \begin{solution} - + Trommelfell \end{solution} \question Welchen zwei Funktionen dient das äußere Ohr? \begin{solution} - + Fokussierung Schallrichtungswahrnehmung, Schalldruckverstärkung \end{solution} \question Was ist die Hauptfunktion des Mittelohrs? \begin{solution} - + Gesamtschalldruckverstärkung \end{solution} \question Welche Strukturmerkmale des Mittelohrs tragen zur Schalldruckverstärkung bei? \begin{solution} - + \begin{itemize*} + \item Flächenverhältnis Trommelfell-Steigbügelgrundplatte + \item Hebelarme des Gehörknöchelchen(Hammer/Amboss) + \item Hebelarm durch die Biegung des Trommelfells und unsymmetrische Anheftung des Hammers + \end{itemize*} \end{solution} \question Wie heißt die Knochenstruktur, in die das Innenohr eingebettet ist? \begin{solution} - + Felsenbein \end{solution} \question In welcher Struktur befinden sich die Hörsinneszellen und wie heißen diese? \begin{solution} - + Corti-Organ \end{solution} \question Mit welcher Membran ist das Corti-Organ fest verbunden? \begin{solution} - + membrana basilaris \end{solution} \question An welchem Ende ist die Cochlea empfindlich für hohe Frequenzen – am Helicotrema oder am ovalen Fenster? \begin{solution} - + ovalen Fenster \end{solution} \question Die Stereozilien welcher Haarzellen sind fest mit der Tectorialmembran verbunden? \begin{solution} - + äußere Haarzellen \end{solution} \question Was ist die Funktion der äußeren Haarzellen? \begin{solution} - + Rückkopplung zur Regulierung von Sensoroutput \end{solution} \question Welche beiden Hörbahnen kann man unterscheiden? \begin{solution} - + dorsale und ventrale Höhrbahn \end{solution} \question Was ist die Funktion der dorsalen Hörbahn? \begin{solution} - + verursacht bewusste Wahrnehmung \end{solution} \question Was ist die Funktion der ventralen Hörbahn? \begin{solution} - + verursacht akustische Reflexe \end{solution} \question Wo befinden sich die Zellkörper der 4 Neuronen der dorsalen Hörbahn (richtige Reihenfolge in Richtung des Hauptinformationsflusses) \begin{solution} - + \begin{enumerate*} + \item Neuron = 8er Hirnnerv(Hörnerv) + \item Neuron = Medulla(Dorsaler Cochleariskern) + \item Neuron = Mittelhirn(Colliculus inf.) + \item Neuron = Zwischenhirn(Innerer Kniehöcker) + \end{enumerate*} \end{solution} \question In welcher Hirnhälfte bezüglich des entsprechenden Ohres endet die dorsale Hörbahn? \begin{solution} - + Linke Hirnhälfte \end{solution} \question In welchem Hirnlappen findet die kortikale Verarbeitung auditorischer Information hauptsächlich statt? \begin{solution} - + Temporallapen \end{solution} \question Mit welchem Gerät kann man untersuchen, ob ein Patient an Mittel- oder Innenohrtaubheit leidet? \begin{solution} - + Stimmgabel \end{solution} \question Nennen Sie eine mögliche Ursache für Mittelohrtaubheit! \begin{solution} - + Riß im Trommelfell \end{solution} \question Nennen Sie eine mögliche Ursache für Innenohrtaubheit! \begin{solution} - + Verletzung Cochlea \end{solution} \question Womit kann Innenohrtaubheit therapiert werden? \begin{solution} - + Cochlea Implantate \end{solution} \question Aus welchen 5 flüssigkeitsgefüllten Hohlräumen besteht das Labyrinth-Organ? \begin{solution} - + Sacculus, Utriculus, anterior Kanal, posterior Kanal, horizontal Kanal \end{solution} \question Nennen Sie die 5 wichtigen Projektionsziele vestibulärer Nervenfasern! \begin{solution} - + Rückenmark, Thalamus, Retikuläre Formation, Cerebellum, auf die Kerne des 3,4,6 Hirnnervs \end{solution} \question Nennen Sie 2 häufige vestibuläre Störungen! \begin{solution} - + Neuritis Vestibularis, Gutartiger Lagerungschwindel \end{solution} \question Was ist die Ursache des gutartigen Lagerungsschwindels? \begin{solution} - + Ablösung Otholiten und ,,herumschlingern'' in den Bogengängen \end{solution} \question Was ist die Ursache der Neuritis vestibularis? \begin{solution} - + Entzündung des Vestibularnervs \end{solution} \question In welchem Teil des Gehirns endet der Riechnerv? \begin{solution} - + Riechhirn (Bulbus Olfactorius) \end{solution} \question Welche Arten von Neuronen im ZNS werden ständig erneuert? \begin{solution} - + Riechzellen \end{solution} \question Wodurch entstehen komplexe Geschmacksempfindungen? \begin{solution} - + Interaktion mit anderen Sinnen \end{solution} \question Auf welchem Teil der Zunge schmecken wir süß? \begin{solution} - + Zungenspitze \end{solution} \question Welche kognitive Funktion ist besonders mit dem Hippocampus verbunden? \begin{solution} - + Bildung von Erinnerungen \end{solution} \question In welchem Großhirnlappen befindet sich der Hippocampus? \begin{solution} - + Temporallappen \end{solution} \question An welche anderen limbischen Strukturen grenzt der Hippocampus. \begin{solution} - + Amygdala und entohirnaler Cortex \end{solution} \question Was ist die Haupteingangsstruktur für den Hippocampus? \begin{solution} - + Entohirnaler Cortex \end{solution} \question Aus welchem strukturellen Cortextyp besteht der Hippocampus? \begin{solution} - + Allocortex \end{solution} \question An welche andere limbische Struktur grenzt der Mandelkern unmittelbar? \begin{solution} - + Hippocampus \end{solution} \question Bei welcher kognitiven Funktion spielt die Amygdala eine herausragende Rolle? \begin{solution} - + Angst und Furcht \end{solution} \question Wie breiten sich die meisten Hormone aus? \begin{solution} - + Blutkreislauf \end{solution} \question Wo werden die meisten Hormone freigesetzt? \begin{solution} - + Gehirn/Hypothalamus \end{solution} \question Nennen Sie die drei wichtigsten chemischen Gruppen von Hormonen! \begin{solution} - + Peptide \& Proteine, Aminosäurederivate, Steroide \end{solution} \question Was sind Peptide? \begin{solution} - + Ketten von Aminosäuren \end{solution} \question Welcher Teil des Gehirns spielt eine zentrale Rolle bei der Hormonausschüttung? \begin{solution} - + Hypothalamus \end{solution} \question Welche Drüse spielt im hormonellen System eine übergeordnete Rolle? \begin{solution} - + Hypophyse \end{solution} \question Nennen Sie 5 wichtige Hormondrüsen! \begin{solution} - + Nebenniere, Schilddrüse, Hypothalamus, Bauchspeicheldrüse, Hoden/Eierstock \end{solution} \question Welcher Teil der Hypophyse wird direkt vom Hypothalamus innerviert? \begin{solution} - + Hypophysenhinterlappen \end{solution} \question Über welchen Signalweg wird die Information vom Hypothalamus zum Hypophysenvorderlappen übermittelt? \begin{solution} - + Hypothalamusneuronen zu hypothalamo-hypophysäre Pfortadersysten zu Hypophysenstiel \end{solution} \question Welche Hormone werden hauptsächlich durch den Hypophysenhinterlappen ausgeschüttet? \begin{solution} - + Oxytocin,Vasopressin \end{solution} \question Durch welche drei Mechanismen wird die Hormonfreisetzung geregelt und der Homonspiegel stabilisiert? \begin{solution} - + Nervensystem (=Innervierung meist durch autonomes Nervensystem), andere Hormone, nichthormonelle Substanzen \end{solution} \question Wo werden steroide Sexualhormone produziert? \begin{solution} - + Keimdrüsen(Gonaden:Hoden,Eierstock) \end{solution} \question Welche 3 Grundklassen von steroiden Sexualhormonen gibt es? \begin{solution} - + Androgene, Östrogene, Gestagene \end{solution} \question Wie erfolgt der Freisetzung von Sexualhormonen in Frauen und Männern? \begin{solution} - + Männer = Gleichmäßig, Frauen = Zyklisch; Verschiedene Dynamiken über Hypophyse vom Hypothalamus gesteuert \end{solution} \question Welches Hormon sorgt vor und unmittelbar nach der Geburt für eine männliche Entwicklung? \begin{solution} - + Testosteron \end{solution} \question Durch welche Hormone wird das weibliche Sexualverhalten beim Menschen maßgeblich gesteuert? \begin{solution} - + Androgene \end{solution} \question Welche Arten von Stresshormonen werden bei kurzfristigem und langfristigem Stress ausgeschüttet? \begin{solution} - + Kurzfristiger Stress: Katecholamine; \quad Langfristiger Stress: Glukokortikoide \end{solution} \question Nennen Sie ein typisches glukokortikoides Stresshormon! \begin{solution} - + Cortisol \end{solution} \question Welche beiden Hormone werden im Nebennierenmark ausgeschüttet? \begin{solution} - + Adrenalin (Epinephrin) und Noradrenalin (Norepinephrin) \end{solution} \question Welche beiden Gruppen von Hormonen werden in der Nebennierenrinde ausgeschüttet? \begin{solution} - + Glukokortikoiden und Androgenen \end{solution} \question Nennen Sie 2 wichtige Wirkungen von Glukokortikoiden! \begin{solution} - + \begin{itemize*} + \item Beeinflussung des Stoffwechsels: Neubildung von Kohlenhydraten aus Proteinen und Fetten + \item Beeinflussung von Wasser- und Elektrolythaushalt + \item Unterdrückung der Antikörperproduktion des Immunsystems, dadurch Entzündungshemmung + \end{itemize*} \end{solution} \question Welche beiden chemischen Elemente sind für die Bildung von Schilddrüsenhormonen von Bedeutung? \begin{solution} - + Iod und Eisen \end{solution} \question Was ist die Hauptwirkung der Schilddrüsenhormone? \begin{solution} - + Regelung des Grundumsatzes \end{solution} \question Wozu führt Schilddrüsenunterfunktion im Erwachsenenalter? \begin{solution} - + Stoffwechselverlangsamung, Verringerung der Leistungsfähigkeit \end{solution} \question Welches Hormon wird von der Geburtshilfemedizin im sogenannten „Wehentropf“ verwendet? \begin{solution} - + Oxytocin \end{solution} \question Wodurch wird die Ausschüttung von Oxytocin ausgelöst? \begin{solution} - + Angenehmer Hautkontakt (Kuschelhormon) \end{solution} \question Welche neuronalen Populationen haben Sympathikus und Parasympathikus und wo befinden sich diese? \begin{solution} - + \begin{itemize*} + \item Sympathikus = Ganglien Nahe der Wirbelsäule (?) + \item Parasympathicus = Ganglien nahe oder in den Organen (?) + \end{itemize*} \end{solution} \question Zu welchem Bestandteil des autonomen Nervensystems gehört der Grenzstrang? \begin{solution} - + Zentraler Teil (?) \end{solution} \question Wo befinden sich allgemein die autonomen Ganglien des Sympathikus und des Parasympathikus? \begin{solution} - + Zwischen Zentralnervensystem und inneren Organen \end{solution} \question Welcher Neurotransmitter wird durch die präganglionären Neuronen des Sympathikus ausgeschüttet? \begin{solution} - + Acetylcholin \end{solution} \question Welcher Neurotransmitter wird durch die postganglionären Neuronen des Sympathikus ausgeschüttet? \begin{solution} - + (Nor)Adrenalin \end{solution} \question Welcher Neurotransmitter wird durch die präganglionären Neuronen des Parasympathikus ausgeschüttet? \begin{solution} - + Acetylcholin \end{solution} \question Welcher Neurotransmitter wird durch die postganglionären Neuronen des Parasympathikus ausgeschüttet? \begin{solution} - + Acetylcholin \end{solution} \question Wo befinden sich die Zellkörper der präganglionären sympathischen Neuronen? \begin{solution} - + Brust und Lendenmark \end{solution} \question Wo befinden sich die Zellkörper der präganglionären parasympathischen Neuronen? \begin{solution} - + Hirnstamm, Mittelhirn, Sakralmark \end{solution} \question Über welchen Pfad übt der Sympathikus eine globale Wirkung auf den Organismus aus? \begin{solution} - + Grenzstrang (Truncus sympathicus) \end{solution} \question Was sind die grundsätzlichen Rollen von Sympathikus und Parasympathikus? \begin{solution} - + \begin{itemize*} + \item Symphatikus: Vorbereitung Flucht und Kampf + \item Parasymphatikus: Entspannung und Verdauung + \end{itemize*} \end{solution} \question Nennen Sie 4 Hauptwirkungen des Sympathikus! \begin{solution} - + Atemfrequenz steigern, Herzfrequenz steigern, Darmtätigkeit senken, Glykogenmetabolismus i.d. Leber steigern, Schwitzen, Pupillenerweiterung \end{solution} \question Nennen Sie 4 Hauptwirkungen des Parasympathikus! \begin{solution} - + Atemfrequenz senken, Herzfrequenz senken, Darmtätigkeit steigern, Pupillen verengen \end{solution} \question Nennen Sie 4 Funktionen des Hypothalamus! \begin{solution} - + Körpertemperaturregelung, Regelung Wasserhaushalt, Regelung Hormonsekretion in Hypophyse, Regelung physiologischer Reaktion auf Erregungszustände \end{solution} \question Nennen Sie die drei Phasen des Energiestoffwechsels und geben Sie an durch welche charakteristischen Hormonspiegel diese gekennzeichnet sind! \begin{solution} - + Cephalische Phase, Absortive Phase, Fastenphase; durch Insulin und Glukagonspiegel \end{solution} \question Nennen Sie 3 Merkmale der cephalischen und absorptiven Energiestoffwechselphasen! \begin{solution} - + niedriger Glukagonspiegel, hoher Insulinspiegel, fördert Nutzung Blutzucker(Glukose) als Energiequelle \end{solution} \question Nennen Sie 3 Merkmale der Fastenphase des Energiestoffwechsels! \begin{solution} - + Hoher Glukagonspiegel, niedriger Insulinspiegel, fördert Umwandlung Fette zu Fettsäuren, Nutzung freier Fettsäuren als Energiequelle \end{solution} \question Nennen Sie 3 Argumente die gegen die Sollwerthypothese der Nahrungsaufnahme sprechen! \begin{solution} - + \begin{itemize*} + \item Evolution: Nahrung musste aufgenommen werden, wenn sie verfügbar war + \item Experiment: Schwankungen in Körperfett und Blutzucker beeinflussen die Nahrungsaufnahme nur, wenn sie extrem sind + \item Nahrungsaufnahme wird durch vielerlei Faktoren bestimmt, wie visuelle und olfaktorische + Reize, Emotionen, Stress usw. + \end{itemize*} \end{solution} \question Was ist die Alternative zur Sollwerthypothese der Nahrungsaufnahme? \begin{solution} - + Positive Anreiztheorie \end{solution} \question Erläuterns Sie einen der wichtigen Mechanismen zur Regulierung von Hunger und Sättigung! \begin{solution} - +\begin{itemize*} + \item Magen-Darm-Trakt: Freisetzung von Peptiden, die an Neurorezeptoren im Gehirn (z.B. im Hypothalamus) binden und als Sättigungssignal wirken. + \item Serotonin: verringert Anziehungskraft schmackhafter Nahrung, reduziert die Aufnahme pro Mahlzeit, verlagert Präferenzen weg von fetthaltiger Nahrung. Appetitszügler sind häufig Serotoninagonisten. + \item Leptin, Insulin und andere: regulieren die Anlage von Fettdepots, Leptinmangel führt zu exzessiver Nahrungsaufnahme und Fettleibigkeit. Bei Insulinmangel isst man viel und bleibt schlank, da die Nahrung nicht in Fettdepots umgewandelt werden kann. +\end{itemize*} \end{solution} \question Wie viele Schlafphasen unterscheidet man und welche davon bezeichnet man als Slow-Wave-Sleep?